Civil Procedure - Casebook Connect

Réussis tes devoirs et examens dès maintenant avec Quizwiz!

Plaintiff, a citizen of State X, filed suit in federal district court against Defendant Car Co., incorporated in State Y with its principal place of business in State X. Plaintiff alleges a defect in the manufacturing of her car by Defendant caused her accident where she incurred property damage and severe bodily injuries. Due to the severity of her injuries, Plaintiff alleges her medical expenses alone exceed $1,000,000. After filing suit in federal district court in State Z, Defendant filed a motion to dismiss for lack of personal jurisdiction. The court denied the motion and Defendant filed its answer to Plaintiff's complaint, denying any liability for Plaintiff's losses. A month thereafter, Defendant filed a motion to dismiss for lack of subject matter jurisdiction. Plaintiff opposed the motion arguing that Defendant waived its right to object to the court's power when it failed to assert it with its previous motion. How should the court rule on Defendant's second motion to dismiss? (A) The district court should deny Defendant's motion, because Defendant waived its right to object to the court's power when it unreasonably delayed in asserting this defense. (B) The district court should grant Defendant's motion, because the court lacks jurisdiction over the parties because Plaintiff and Defendant are both from State X. (C) The district court should deny Defendant's motion, because Plaintiff and Defendant are from different states, State X and State Y, respectively. (D) The district court should grant Defendant's motion, because the court erred in its decision to deny Defendant's motion to dismiss for lack of personal jurisdiction.

B is correct. Federal courts have subject matter jurisdiction over state law claims where the adverse parties are diverse in citizenship (meaning no plaintiff is from the same state as any defendant) and the controversy is reasonably likely to exceed $75,000. Individuals are citizens in the state where they are domiciled, meaning where their true, fixed, and permanent place of residence is located. Corporations have dual citizenship — their state of incorporation and where their principal place of business is located. Here, Plaintiff is a citizen of State X. Defendant Car Co. has its principal place of business in State X. Since both Plaintiff and Defendant are from the same state, there is no diversity of citizenship. Therefore, the federal court lacks the power to adjudicate this controversy and it must dismiss. A is incorrect because the lack of subject matter jurisdiction cannot be waived. One of the fundamental principles in civil procedure is the court's power over the controversy. Subject matter jurisdiction, the court's power over the controversy, can never be waived. The lack of the court's subject matter jurisdiction can be asserted at any stage of the litigation, even for the first time on appeal. Rule 12(g)(2) requires parties to consolidate and bring forth as many defenses and objections the party has available or else they are waived, except for the lack of subject matter jurisdiction found in FRCP 12 (h)(3) C is incorrect because it misstates Car Co.'s citizenship. For purposes of diversity jurisdiction, individuals are citizens in the state where they are domiciled, meaning where their true, fixed, and permanent place of residence is located. Corporations have dual citizenship — their state of incorporation and where their principal place of business is located. Here, Car Co. is incorporated in State Y and has its principal place of business in State X. Thus, at all times, Car Co. is a citizen of both states, destroying diversity of citizenship. D is incorrect because an error in ruling on one motion does not give the court the power to "make up" for the error by incorrectly ruling on another motion. If the court erred in ruling on the first motion, the party against whom the court ruled can take up the matter on appeal at the appropriate time.

Pure Dairy, a food manufacturer, is sued in federal court under diversity jurisdiction by several consumers who experienced severe food poisoning after eating Pure Dairy's eggs. Pure Dairy's defense is that the consumers' food poisoning was caused by improper cooking techniques, not impurities in the eggs. To support its defense, Pure Dairy hired a professor of food science, with whom it frequently corresponded as the litigation progressed. The correspondence began with an engagement letter that specified the professor's fee and the scope of his work. That work included testifying at trial. Pure Dairy's attorney also wrote to the professor about facts learned in discovery, seeking the professor's opinion on how those facts shed light on the food poisoning's potential causes. Based on those facts, the professor wrote a draft report stating his opinion on what caused the food poisoning, which was revised before being finalized and turned over to the plaintiffs. Which of the following pieces of information would not be discoverable under Rule 26? (A) The professor's identity (B) The professor's draft report (C) The professor's fee (D) The facts that the attorney provided to the professor for purposes of getting the professor's opinion

B is correct. The food science professor is a testifying expert, and Rule 26(a)(2)(B) requires a testifying expert to produce a final report stating his or her expert opinions, the facts or data used to form them, the exhibits the expert will use, the expert's qualifications, a list of cases in which the expert has testified, and a statement of the expert's compensation. However, Rule 26(b)(4)(B) excludes from discovery drafts of any such report. As a result, any version that the food science professor drafted before his final report would not be discoverable. Ais incorrect because the expert's identity is a piece of information that must be in the testifying expert's final report. See 26(a)(2)(A). C is incorrect for the same reason—the expert's fee must also be in the final report. Moreover, Rule 26(b)(4)(C) confirms that a testifying expert's fee is discoverable information. D is incorrect because Rule 26(b)(4)(C)(ii) states that facts the attorney provided and the expert considered are discoverable—even if shared through attorney-expert communications.

Passenger, a citizen of State L, was riding in a car driven by his friend Driver, a citizen of State T. They crashed into a car driven by Plaintiff, a State T citizen. Plaintiff and Passenger filed an action in federal court in State T in which each of the plaintiffs asserted a $100,000 tort action against Driver. Driver filed a motion to dismiss for lack of jurisdiction, which the court granted. Did the court err in granting the motion? (A) Yes, because jurisdiction is proper under diversity and supplemental jurisdiction. (B) No, because Driver and Plaintiff are both citizens of State T. (C) Yes, because each claim is for more than $75,000. (D) No, because the action was filed in State T.

B is correct. The presence of one non-diverse plaintiff (here, Plaintiff) destroys the complete diversity required by 28 U.S.C. §1332. Thus, the lack of diversity between Plaintiff and Driver precludes Passenger from using diversity jurisdiction as the basis for exercising original jurisdiction over any claim in the case. Thus, the court must dismiss both claims. A is incorrect. With no original jurisdiction over either claim (this being state law claims lacking proper diversity jurisdiction), there is no claim upon which to "anchor" the claims in exercising supplemental jurisdiction. So even though the claim by Passenger against Driver would have met the requirements of diversity jurisdiction under §1332 had it been brought by itself, the claim by the non-diverse Plaintiff prevents the court from exercising supplemental jurisdiction. C is incorrect. In order to properly assert federal subject matter jurisdiction based on diversity, the Plaintiff must satisfy both the diversity requirement and the amount-in-controversy requirement. Here, the fact that both claims exceed the amount in controversy requirement is irrelevant since Plaintiff and Driver are not completely diverse in citizenship. D is incorrect. The fact that the action was filed in State T is irrelevant.

The plaintiff, a citizen of State A, files a claim in state court alleging that the defendant, a Delaware corporation, violated her rights under the federal Civil Rights Act of 1964 and seeks damages in the amount of $50,000. Does the state court have subject matter jurisdiction? (A) Yes, because the parties are diverse. (B) Yes, because the Civil Rights Act does not provide for exclusive federal court jurisdiction. (C) No, because this claim arises under federal law. (D) No, because the plaintiff is seeking only $50,000.

B is correct. The question here is whether a state court would have subject matter jurisdiction over a federal statutory claim. State courts have concurrent jurisdiction with federal courts over claims arising under all federal statutes except those few that vest federal courts with exclusive jurisdiction. The Civil Rights Act is not one of those few statutes. A is incorrect because diversity jurisdiction relates to the subject matter jurisdiction of a federal court under § 1332, not a state court. State courts are courts of general jurisdiction. C is incorrect because it does not go far enough. The fact that a claim arises under federal law does not always mean that the state court has subject matter jurisdiction because some minority of federal statutes provide for exclusive jurisdiction in the federal courts. D is incorrect because there is no amount in controversy requirement for federal claims.

An industrial accident injured Worker while using a machine for work. Worker was using the machine as intended while working for Shipping Co., incorporated in State A and principal place of business in State B. Worker worked in State B but was a citizen of State C. Worker was severally injured requiring months of hospitalization and rehabilitation. After Worker recovered he filed suit in State B state court against his employer Shipping Co. under the State B workman's compensation statute. Before filing its answer, Shipping Co. removed the case from state court to federal court in State B. Worker filed a motion to remand. How should the court rule on the Worker's motion? (A) The court should grant the motion because Shipping Co.'s removal was untimely. (B) The court should grant the motion because the federal district court does not have jurisdiction over the Worker's claim and because Shipping Co. is a citizen of State B. (C) The court should deny the motion because the federal district court has jurisdiction over the Worker's claim. (D) The court should deny the motion because diversity jurisdiction is satisfied where the Worker and Shipping Co. are citizens of different states and the amount in controversy is reasonably likely to exceed $75,000.

B is the correct answer. 28 U.S.C. §1445 excludes certain cases, which would otherwise be removable, from being removed. Under §1445(c), cases arising under state workman's compensation statutes are barred from removal, even if they otherwise satisfy the requirements for diversity jurisdiction under §1332. Therefore, B is correct and Answer Cis incorrect. Further, because the attempted removal would be based on §1332, removal is impermissible if the defendant is a citizen of the state in which the action was brought (§1441(b)(2)). Here, the facts indicate that Shipping Co. has its principal place of business in State B, which would make it a citizen of State B for purposes of diversity jurisdiction (§1332(c)(1)). Therefore answer D is incorrect. Answer A is incorrect because Shipping Co. removed the case before the answer was filed. A defendant has the right to remove a case from state court to federal court if the case could have originally been filed in federal court (28 U.S.C. §1441) within 30 days of service of the complaint (28 U.S.C. §1446). Here, Shipping Co. removed the case before it filed its answer, which is due 21 days after service (FRCP 12(a)), therefore, the removal was presumably timely. Answer D is incorrect because the elements of diversity jurisdiction under 28 U.S.C. §1332 are satisfied where the Worker is from State C and Shipping Co. from States A and B, and with extended hospital and rehabilitation stays the amount in controversy is reasonably likely to exceed $75,000. However, while §1332 is satisfied, for the reasons stated above, §1441(b)(2) and §1445(c) explicitly preclude this case from being removed.

James University is a for-profit educational institution incorporated in the state of California. Its main campus is also in California. The University's only other campus is a slightly smaller one in the state of Nevada. Both campuses have operated for over 50 years. A student from Arizona who attends class at the California campus sues James University for sex discrimination by one of her professors in class. The student brings this suit in a Nevada federal court and then serves process on the University in California. If the University moves to dismiss for lack of personal jurisdiction under Rule 12(b)(2), which doctrine of personal jurisdiction would provide the strongest argument for maintaining jurisdiction in Nevada? (A) General jurisdiction based on consent to the Nevada court's power (B) General jurisdiction based on the University's activity in Nevada (C) In rem jurisdiction (D) General jurisdiction based on in-forum service

B is the correct answer. Daimler supplies the strongest argument given James University's continuous 50-year operation in Nevada and its lack of presence anywhere else besides California. In effect, Nevada is James University's second home (its first is California by virtue of its incorporation and campus there). Even though James University's Nevada campus is smaller than its California operations, the relative size difference is slight. In contrast, the Supreme Court's recent decisions have rejected general jurisdiction in cases where the defendant's activity in the forum state was a much smaller percentage of the defendant's overall activity. A is incorrect. Nothing in the question indicates that James University has consented to Nevada's jurisdiction. Simply having a campus in Nevada does not constitute consent. C is incorrect; in rem jurisdiction doesn't apply because this is not a case where the ownership of property is at stake and service of process was not affected by attaching the University's property. D is incorrect. The University was not served in Nevada, so Burnham's service-of-process basis for general jurisdiction does not apply.

Hercules Hardware Company (HHC) is a high-tech home product retailer. HHC's operations are in Minnesota and it is incorporated there. HHC also maintains a retail sales website with national reach. One of HHC's best-selling products is a snow-melting stair mat. As a result of aggressive marketing aimed at Minnesota and North Dakota, HHC sells an unusually high number of mats in those two states. Minnesota resident Hector Diaz bought an HHC mat at one of HHC's Minnesota retail outlets. After deciding to move to the Pacific Northwest, Hector resold the mat to a hardware store in Montana on the drive to his new home. The store sold the mat to Montana resident Darnell Reynolds. Darnell was severely burned at his Montana home when the mat's electrical components malfunctioned. Darnell sued HHC for product liability in a North Dakota state court, hoping to take advantage of favorable product liability laws there. What is the most likely reason a North Dakota court would refuse specific personal jurisdiction over HHC? (A) HHC did not target the forum of North Dakota. (B) There is an insufficient relationship between the forum, the injury, and HHC's activities. (C) HHC's operations are entirely based in Minnesota. (D) HHC did not sell enough mats in North Dakota.

B is the correct answer. Darnell's purchase was two steps removed from HHC's original sale, which happened in Minnesota, not North Dakota; he did not buy the mat in North Dakota, and he was not injured in North Dakota. Under the U.S. Supreme Court's decision in Bristol Myers-Squibb v. California Superior Court, the "arising out of" requirement in specific jurisdiction demands "an affiliation between the forum and the underlying controversy, principally, [an] activity or an occurrence that takes place in the forum State." Under these facts, this affiliation is lacking. A is incorrect because purposeful availment is not the problem here; this standard would likely be met under either one of the competing stream of commerce approaches in J. McIntyre Machinery v. Nicastro. Even under the Kennedy plurality's view, HHC's aggressive marketing to North Dakotans would suffice to target the forum. Ginsburg's dissenting view would also be satisfied with HHC's robust sales to North Dakotans, combined with regularly selling nationally through its website. C is incorrect because a company's operations do not need to be in a state for that state to exercise specific jurisdiction over it. D is incorrect for the same reasons that Ais incorrect.

A Texas citizen was given a two-year contract by a Middle Eastern oil company to work in one of its plants in that country. After six months, the plaintiff was fired. She returned to the United States and filed a claim in federal court in Dallas alleging that she was terminated on the basis of her sex in violation of Title VII of the federal Civil Rights Act of 1964. The defendant moved to dismiss the action on the ground that although it was subject to personal jurisdiction in Texas because of its past business dealings there (including recruiting the plaintiff from Texas), maintaining the suit there would result in extreme inconvenience to it since all the alleged acts of discrimination occurred in the middle eastern country, all the witnesses to the events in question were there, and defending in the United States would be extremely inconvenient. The plaintiff opposed the defense motion on the ground that a Middle Eastern court would apply foreign law to this dispute and that law does not prohibit discrimination on the basis of sex. Should the court dismiss the case? (A) Yes, under the doctrine of forum non conveniens (B) No, because under these circumstances dismissal would not further the interests of justice. (C) Yes, because the defendant is a foreign company. (D) No, because the plaintiff is an American citizen.

B is the correct answer. In Piper Aircraft v. Reyno, the Supreme Court indicated that, in determining whether an alternative forum exists, if the remedy provided by the alternative forum is so clearly inadequate or unsatisfactory that it is no remedy at all, the unfavorable change in law may be given substantial weight and the district court may conclude that dismissal would not be in the interests of justice. But the Court also stated that this standard is not met merely when the remedy provided by the alternate forum is either more difficult to obtain or not as lavish as that provided by the law that would be applied by the initially chosen forum. On the other hand, in this problem, the plaintiff would be completely divested of a right of action under Middle Eastern law and this would result in no remedy at all. Consequently, the court would deny the motion to dismiss on forum non conveniens ground because even if the chosen Texas forum is manifestly and grossly inconvenient, under these circumstances, dismissal is not in the interests of justice. Answer A is incorrect. The defendant here is relying on the doctrine of forum non conveniens as the basis for dismissing an action over which the court clearly has subject matter jurisdiction, in which venue lies, and with respect to a defendant over which it clearly has personal jurisdiction. Courts will grant this motion upon a showing not only that the chosen forum is extremely inconvenient but that a distinctly more convenient forum exists in which the court would have subject matter jurisdiction, venue, and personal jurisdiction. However, in this problem, the plaintiff would be completely divested of a right of action under the foreign law and this would result in no remedy at all. Consequently, the court would deny the motion to dismiss on forum non conveniens ground because even if the chosen Texas forum is manifestly and grossly inconvenient, under these circumstances, dismissal is not in the interests of justice. In Piper Aircraft v. Reyno, the Supreme Court indicated that, in determining whether an alternative forum exists, if the remedy provided by the alternative forum is so clearly inadequate or unsatisfactory that it is no remedy at all, the unfavorable change in law may be given substantial weight and the district court may conclude that dismissal would not be in the interests of justice. Answer C is incorrect because this fact, standing alone, is not a sufficient basis for invoking the doctrine of forum non conveniens. Answer D is also incorrect because this is not a sufficient basis for refusing to invoke the doctrine of forum non conveniens.

A Texas plaintiff files a federal question claim against a Nevada defendant in federal district court in Houston, Texas. The events that gave rise to that action occurred exclusively in Nevada. The defendant has no contacts of any kind with Texas and so he moves to transfer the case under §1404 to the federal district court in Reno, Nevada. Nevada has one federal judicial district. Should the court grant the motion? (A) No, because the Texas court does not have personal jurisdiction over the defendant. (B) Yes, because the action could have been brought in Reno and transfer there is in the interests of justice. (C) No, because venue does not lie in Houston. (D) Yes, because the Texas court lacks subject matter jurisdiction over the claim.

B is the correct answer. The crucial factor in this problem is that the transferor court in Houston does not have personal jurisdiction over the defendant. So we must determine whether this defect is fatal to the court's ability to transfer. In Goldlawr v. Heiman, the Court ruled that a court in which venue was improperly laid could still transfer a case under §1406 even if it also lacked personal jurisdiction over the defendant. The Court also stated that this result would promote the congressional objective of removing obstacles that might "impede an expeditious and orderly adjudication of cases and controversies." In the instant hypothetical, the transferor court does not have venue since the Nevada defendant does not reside in that Texas district and the facts giving rise to the claim did not occur in Texas, but in Nevada. Additionally, the facts indicate that the Nevada defendant would not be subject to the personal jurisdiction of the Texas court. But under Goldlawr, the Texas court can cure both of these defects by transferring the case under §1406 to a district in which it "could have been brought." Clearly, as both venue and personal jurisdiction would be available in the transferee court in Nevada, the interests of justice will be served by transferring the case there. The defendant here sought to transfer the case to Nevada under §1404. Nevertheless, to avoid unjust dismissals, the courts extend the rationale of Goldlawr to §1404-based transfers. Thus, since the action could have been brought by the plaintiff in Nevada since venue would lie there and the defendant is subject to personal jurisdiction there, the court should grant the motion. Answer A is incorrect because the ruling in Goldlawr has been extended to make this fact non-fatal to a §1404-based transfer motion. Answer C is incorrect because the ruling in Goldlawr has been extended to make this fact non-fatal to a §1404-based transfer motion. Answer D is incorrect because a motion under §1404 deals with change of venue where, for the convenience of parties and witnesses, and in the interest of justice, a district court may transfer any civil action to any other district or division where it might have been brought or to any district or division to which all parties have consented. If a federal court lacks subject matter jurisdiction in one state, it lacks jurisdiction in all states and a §1404 transfer will not cure this fatal defect.

An Arkansas citizen brought a $100,000 tort action in Arkansas state court against a restaurant located in Alaska alleging that the plaintiff had developed food poisoning while eating in that restaurant because of the unhygienic conditions present in the kitchen. The defendant removed the case. Thereafter, the restaurant filed a motion with the federal trial judge requesting that the court dismiss the case for lack of personal jurisdiction over it. The plaintiff acknowledges that the applicable long-arm statute does not provide jurisdiction over this defendant. Should the court grant this motion? (A) No, because by removing the case the defendant consented to that court's exercise of personal jurisdictional over it. (B) Yes, because the defendant is not subject to personal jurisdiction. (C) No, because the claim meets all the requirements of diversity jurisdiction under U.S.C. §1332. (D) Yes, because defendant is not a citizen of the forum state.

B is the correct answer. The defendant has not waived any defenses by removing the case to federal court and can still challenge the lack of personal jurisdiction under the long-arm statute. Answer A is incorrect because the prevailing rule is that removal does not operate to waive any defenses, including an objection to jurisdiction or sufficiency of service. The case is treated, for these purposes, as if it were originally brought in federal court. Answer C is incorrect because the requirements of diversity jurisdiction go to subject matter jurisdiction, rather than personal jurisdiction. Answer D is incorrect because being a noncitizen is not, by itself, fatal to the exercise of personal jurisdiction if a noncitizen can constitutionally be subject to a state long-arm statute.

An Arizona citizen brought a $70,000 tort action in Arizona state court against a restaurant located in Florida alleging that she had developed food poisoning while eating in that restaurant because of the unhygienic conditions present in the kitchen. The defendant removed the case. Thereafter, the defendant filed a motion with the federal trial judge requesting that the court dismiss the case for lack of personal jurisdiction over it. The plaintiff acknowledges that the applicable long-arm statute does not provide jurisdiction over this defendant. Should the court grant this motion? (A) No, because by removing the case the defendant consented to that court's exercise of personal jurisdictional over it. (B) Yes, because the defendant is not subject to personal jurisdiction. (C) No, because the claim is for $70,000. (D) Yes, because the defendant is not a citizen of the forum state.

B is the correct answer. The defendant has removed a state law claim between diverse parties that does not satisfy the amount in controversy requirement of §1332. Consequently, the federal district court does not have subject matter jurisdiction. Since objections to subject matter jurisdiction are not waivable and, in fact, can be raised by the court sua sponte, this problem raises the question of whether this court can adjudicate the personal jurisdiction challenge prior to determining the existence of subject matter jurisdiction. Obviously, resolution of the subject matter jurisdiction question in this instance pretermits consideration of the defendant's personal jurisdiction challenge and the case will be remanded to the state court since the claim is not removable because it does not meet the amount in controversy requirement of §1332. This, in turn, means that, to the extent the defendant wants to adjudicate the personal jurisdiction issue, a state, rather than a federal, judge will resolve it. While that result might, in some cases, frustrate the defendant's desire to have a federal judge resolve that question, since many personal jurisdiction questions involve the construction of state long-arm statutes, it promotes interests of federalism/comity by leaving the interpretation of state law to the state court. In cases where the personal jurisdiction inquiry is either straightforward (as here) or turns primarily on a federal constitutional issue, the federal district court does not abuse its discretion by turning first to personal jurisdiction, particularly when the subject matter jurisdictional question is either complex or novel. Therefore, this court can consider the personal jurisdiction question even though it does not have subject matter jurisdiction over this claim. Therefore, Answer C is incorrect. Answer A is incorrect because the prevailing rule is that removal does not operate to waive any defenses, including an objection to personal jurisdiction. Answer D is incorrect because being a noncitizen is not, by itself, fatal to the exercise of personal jurisdiction if a noncitizen can constitutionally be subject to a state long-arm statute.

A New York-incorporated and -based company brought a tort claim seeking $1 million in damages against a rival company based in Chicago, Illinois, and incorporated under the laws of Delaware. After being advised by its attorney that Texas law would be most hospitable to this claim, the plaintiff filed this action in a federal court in Dallas, Texas. None of the events that gave rise to this action were connected to Texas. Assume the defendant is subject to personal jurisdiction in Texas. The plaintiff subsequently moved to transfer the case to the district court in Chicago. Should the court grant this motion? (A) No, because plaintiffs cannot transfer cases under §1404. (B) Yes, because the action could have been brought in Chicago. (C) No, because this would allow the plaintiff to go forum shopping for the most advantageous law. (D) Yes, because the federal judges in Illinois can determine the content of Texas law.

B is the correct answer. The only question is whether Chicago, the transferee court, is a district in which the action might have been brought, i.e., was the defendant subject to personal jurisdiction there and does venue lie there. Clearly the answer to that is yes since the defendant resides there. Thus, the motion to transfer should be granted. Answer A is incorrect. The question is whether a plaintiff is ever allowed to transfer a case from the forum that it chose. The text of §1404 contains no defendant-specific language. And the courts uniformly have construed this language as not precluding plaintiffs from seeking a transfer. Answer C is incorrect. In Van Dusen v. Barrack, the Supreme Court held that when a case is transferred per §1404, transfer should have no impact on the substantive law to be applied to the case since the original court was an appropriate venue under the governing venue rules. Thus, the transferee court must apply the substantive law, including the choice of law rules, which the transferor court would have applied in the absence of a transfer. This problem raises the question of whether the plaintiff can take advantage of this doctrine by filing the action in the district that would apply the most advantageous law and then seeking to transfer the case to a more convenient district. This issue of forum shopping was placed directly before the Supreme Court in Ferens v. John Deere Co., where the Court rejected the defendant's call for an exception to the Van Dusen v. Barrack rule in plaintiff-initiated transfers under §1404. Instead, it held the transferee court must apply the law of the transferor forum regardless of which party filed the §1404 motion. Answer D is incorrect because it is irrelevant to this problem.

In an action filed in federal district court, the defendant refuses to waive service. The defendant owns homes in New York, California, and Florida, spending several months in each place at different times of the year. The plaintiff serves the defendant's wife at the Florida home while the defendant is staying at the California home. She forgets to transfer the process to the defendant. Is this service proper under the Federal Rules of Civil Procedure? (A) No, because the defendant was in California. (B) Yes, because the Florida home qualifies as defendant's dwelling house or usual place of abode. (C) No, because the defendant did not receive the process. (D) Yes, because the recipient is defendant's wife and service upon a legal spouse or domestic partner in any location is sufficient to meet the Federal Rule.

B is the correct answer. This assumes that the house is the defendant's "dwelling house or usual place of abode." Under Fed. R. Civ. P. 4(e)(2)(B), service is sufficient if it is delivered to the defendant's "dwelling house or usual place of abode" with some person of suitable age and discretion residing therein. Here, the defendant owns many homes. The courts have concluded that a defendant can have more than one dwelling house or usual place of abode. A home will fit this bill if there are indicia of permanence such as the presence of furnishings or other manifestations of repeated presence. Answer A is incorrect because the fact that the defendant was not personally served nor was present at the time of service is not fatal. Answer C is incorrect because the courts have ruled that as long as the formal requirements of service have been met, actual receipt by the defendant is not required. Answer D is incorrect because mere service on the defendant's wife is not enough to meet the requirements of Rule 4(e)(2)(B).

Family, from State A, was in the process of selling their home and purchasing a new home, also in State A. After selling their old home, they decided to move into their in-laws' guestroom in their State A home until they found their dream house. Since the in-law's house was only a few blocks away from their old home, the family's 2 children were able to stay in the same school but the couple made sure to change the children's address on file to ensure proper delivery of all communications. Three weeks after moving in, a diversity action was filed against the couple relating to a car accident they had been involved in 2 years prior. While the family was visiting the local zoo, a process server went to the in-laws' house where the family was staying and left the paperwork with the mother-in-law. The process server gave explicit instruction to the mother-in-law to hand deliver the envelope to the couple upon their return home. The mother-in-law left the envelope with the rest of the mail and did not hand it to the couple until a week later. The following week, the couple filed a motion to dismiss. How should the court rule on the motion? (A) The court should deny the motion because the couple was served in State A, the state in which they are domiciled. (B) The court should deny the motion because service of process was made in accordance with the rules. (C) The court should grant the motion because service of process was not made in accordance with the rules. (D) The court should grant the motion because the couple did not receive the envelope with process until a week after it had been delivered to the in-laws' house.

B is the correct answer. Under FRCP 4(e), Serving an Individual Within a Judicial District of the United States. Unless federal law provides otherwise, an individual—other than a minor, an incompetent person, or a person whose waiver has been filed—may be served in a judicial district of the United States by: (1) following state law for serving a summons in an action brought in courts of general jurisdiction in the state where the district court is located or where service is made; or (2) doing any of the following: (A) delivering a copy of the summons and of the complaint to the individual personally; (B) leaving a copy of each at the individual's dwelling or usual place of abode with someone of suitable age and discretion who resides there; or (C) delivering a copy of each to an agent authorized by appointment or by law to receive service of process. Here, the couple and their children are living in the in-laws' house indefinitely as they sold their old home and have not purchased another one yet. The court will likely find this is their "dwelling or usual place of abode." Furthermore, the mother-in-law is a person of suitable age and discretion who resides there, thus satisfying the requirements for "substituted service" under FRCP 4(e)(2)(B). Therefore, B is correct and Answer C is incorrect. Answer A is incorrect because, while State A is the state in which the couple is domiciled, the FRCP and the Constitution must still be satisfied when effectuating service of process. Answer D is incorrect because when the couple received the envelope with process after it was properly served is not relevant to the determination for the motion to dismiss for insufficient service of process under FRCP 12(b)(5).

In a breach of contract action filed in federal district court by a Florida plaintiff against a Louisiana defendant, the defendant refuses to waive service. The plaintiff serves the attorney who had represented the defendant in his previous divorce, at her law office. Is this service proper under the Federal Rules of Civil Procedure? (A) Yes, because she is the defendant's attorney. (B) No, because she was not authorized to receive service. (C) No, because she was served at her office and not at the defendant's home. (D) No, because she does not live in the defendant's home.

B is the correct answer. Under Fed. R. Civ. P. 4(e)(2)(C), service can be made upon an agent authorized by appointment to receive service of process. The issue here is whether the defendant's attorney meets that test, i.e., whether the attorney was appointed for the purpose of receiving service of process. Answer A is incorrect because the mere existence of a prior attorney/client relationship is not sufficient to deem the attorney the agent for service of process purposes. Answers C and D are incorrect because the fact that she was not served at the defendant's home is not fatal as Fed. R. Civ. P 4. (e)(2)(C) allows for service on an authorized agent.

Mandy, a resident of New York City, was vacationing in California when the taxi she was riding in collided with an automobile owned by Charles, a resident of San Francisco and driven by Mike, a resident of Los Angeles. Mandy brought a tort action seeking $300,000 in damages against both Charles and Mike in the federal district court for the Southern District of New York, which is located in New York City. Does venue lie in the chosen forum? (A) Yes, because the plaintiff chose this forum. (B) No, because the plaintiff's residence is irrelevant for venue purposes and because the accident occurred and the defendants reside in California. (C) Yes, because the defendants are subject to personal jurisdiction in New York. (D) No, because New York law would govern the resolution of this case.

B is the correct answer. Venue in diversity cases is governed by 28 U.S.C. §1391. Proper venue here lies in the district in which any defendant resides if they all reside in the same state under §1391(b)(1), a district in which a substantial part of the events giving rise to the claim occurred under §1391(b)(2), or a district in which any defendant is subject to personal jurisdiction if there is no other district in the United States that would qualify under subsections (1) or (2) under §1391(b)(3). None of the defendants resides in New York so defendant residence under §1391(b)(1) does not provide venue. The events giving rise to the claim occurred in California, not New York, so that does not provide venue in New York under §1391(b)(2). Even if the defendants would be subject to personal jurisdiction in New York, that is not a basis for venue because that fall-back provision is only available if there is no other district in the United States that would meet either of the prior two tests. In other words, since venue options are available in judicial districts in California, Mandy cannot use §1391(b)(3) to obtain venue in New York. Answer C is incorrect since venue would lie in any judicial district in California, and therefore we cannot rely on the fall-back provision in § 1391(b)(3). Therefore, venue does not lie in New York City. The fact that the plaintiff resides there is irrelevant because the plaintiff's residence is irrelevant to a venue determination. Answer A is incorrect because the plaintiff's choice of venue is not dispositive. Answer D is incorrect because this proposition is irrelevant to this problem. Venue in federal courts is governed by 28 U.S.C. §1391, not state law.

The plaintiff sued a car manufacturer, a tire manufacturer, and a tire valve manufacturer alleging that the three defendants' products were defectively manufactured, causing a car accident that resulted in severe injuries to the plaintiff. After the defendants removed the case to federal court, the tire valve manufacturer filed an answer denying that it manufactured the tire valve involved in the accident. But a reasonable investigation of the valve, as preserved after the accident, would have revealed unique identifying marks showing that the valve was made by this defendant. In response to the plaintiff's timely and proper Rule 11 motion, the court determines that the tire valve manufacturer's denial violated Rule 11. Which of the following sanctions would exceed the court's power under Rule 11? (A) Levying a fine against the tire valve defendant, payable from the company's assets (B) Ordering the tire valve defendant's attorney to conduct training at his firm on the proper investigation of pleadings (C) Ordering the tire valve defendant to pay costs and attorneys' fees in the amount that the plaintiff has spent on the litigation so far (D) Ordering the tire valve defendant's attorney to draft a letter of apology to the plaintiff

C is correct because it is the only option that clearly exceeds the court's authority to issue a sanction "limited to what suffices to deter repetition of the conduct." Rule 11(c)(4). Attorneys' fees are a permitted sanction, but only in an amount "directly resulting from the violation." Rule 11(c)(4). The sanction described in C goes well beyond reimbursing plaintiff for fees incurred in having to deal with the violation, which is the tire valve manufacturer's inaccurate denial. It extends to the fees plaintiff has incurred in suing all three defendants for every piece of attorney work along the way. Ais incorrect because Rule 11(c)(1) generally allows the court to impose a sanction on any "party," and the violation here is a lack of evidentiary support for the answer's denial under Rule 11(b)(3), not a frivolous legal contention under 11(b)(2). Although a Rule 11(b)(2) violation cannot result in a client's monetary sanction, see Rule 11(c)(5)(A), an 11(b)(3) violation can result in monetary sanctions on a client. And under 11(c)(4), one of the acceptable sanctions is "an order to pay a penalty into the court." B is incorrect because the training session the lawyer must hold is aimed at deterring the same conduct as the Rule 11 violation and so is "limited to what suffices to deter repetition of the conduct." Rule 11(c)(4). D is incorrect for the same reason that B is incorrect—a letter of apology is a discrete sanction that does not exceed what is needed for future deterrence.

Defendant is a cattle rancher and owns 1000 acres of grazing land in Montana. Plaintiff owns a strawberry farm that is adjacent to Defendant's land, but across the state line. Over the last five years, Defendant's cattle frequently wandered onto Plaintiff's land, destroying portions of Plaintiff's strawberry patches during the growing season and interfering with Plaintiff's ability to bring its produce to market. Plaintiff sues Defendant for trespass in federal district court, alleging diversity jurisdiction. During discovery, Defendant demands the right "to survey Plaintiff's property (through a land surveyor who will determine the terrestrial boundaries of Plaintiff's property) and to inspect the quality of the strawberries (to determine the quality) that Plaintiff harvests, upon such time as may be convenient to Plaintiff." Plaintiff opposes the request, stating that "Defendant may inspect the land, but may not survey the land." Should the court grant the request? (A) No, because Defendant has not demonstrated good cause for the request (B) No, because Defendant may not inspect the land for the purpose of surveying it (C) Yes, because the Defendant's request is relevant (D) Yes, because Defendant's request is relevant and Plaintiff must pay a fair share of the surveyor's fee

C is correct because the federal discovery rules permit a requesting party to enter "onto designated land or other property possessed or controlled by the responding party," FRCP 34(a)(2), provided the request is relevant to the claim or defense of any party to the action and seeks non-privileged matter. SeeFRCP 26(b)(1). The request to inspect must "describe with reasonable particularity each item or category of items to be inspected," FRCP 34(b), and in doing so, may not rely on boiler-plate language. See U.S. v. Territory of the Virgin Islands, 280 F.R.D. 232, 235 (D.V.I. 2012). Since the complaint alleges trespass, with consequent damage to Plaintiff's fruit, an inspection and survey of the land seems relevant to both the claims and possible defenses in the action. (A) is not correct because the federal discovery rules do not require good cause for a requesting party to enter on land for inspection or other purposes. Under the principle of proportionality, an inspection may be limited if it is unduly burdensome, see FRCP 26(b)(2), and the court may limit the inspection so that it does not go beyond what is relevant. See Hoewischer v. Deerwood Village Mall, LLC, 281 F.R.D. 665 (M.D. Fl. 2011). But a party may not refuse a request for an on-site inspection simply because good cause is not shown if the information sought is relevant to the claims and defenses in the suit. See New York State Ass'n for Retarded Children, Inc. v. Carey, 706 F.2d 956 (2d Cir. 1983). By contrast, good cause is required when a request is made for the physical or mental examination of a party. See FRCP 35(a)(2)(A), and examination in this situation is compulsory only if ordered by the court. See Schlagenhauf v. Holder, 379 U.S. 104 (1964). (B) is not correct because it misstates the federal discovery rule governing inspection of land. The rule permits a requesting party to enter land in order to "inspect, measure, survey, photograph, test, or sample the property or any designated object or operation on it." FRCP 34(a)(2). However, the requesting party is not permitted to use the inspection as an occasion for interviewing the responding party's representatives and the responding party is not required to make its representatives available to answer questions. See Curry v. Goodman, 2003 WL 22305161, at *1, (D. Conn. 2003) ("Consistent with Rule 34(a), defendant is not at liberty to refuse plaintiff's request for an on-site visit .... This does not permit plaintiff's expert 'basic access to managerial personnel to understand the operations of the [premises].'"). (D) is not correct because it misstates the rules for payment of expert fees and of litigation costs. The federal discovery rules permit discovery from an expert who has been retained by an opposing party. See FRCP 26(b)(4). A party seeking expert discovery from a testifying or non-testifying expert is required to pay the reasonable fees incurred in responding to the request unless manifest injustice is shown. See Gwin v. Am. River Transp. Co., 482 F.3d 969 (2007). However, a party is not required to pay the fees of an opposing party's expert; rather the general rule, known as the "American rule," is that each party bears its own costs of litigation. Whether witness fees and expenses will be taxed as costs is a matter of judicial discretion. See FRCP 54(d).

Plaintiff works for Defendant, an individual who supervises a rare books store owned by Company. Plaintiff holds a master's degree in foreign languages and accepted the position because she assumed it would lead to professional advancement. Believing she has been passed over for a promotion because of her race and gender, Plaintiff sued both Defendant and Company in federal district court alleging discrimination. The complaint survived a motion to dismiss. Plaintiff now wants to depose Defendant to ask questions about Company's promotion practices. Defendant does not live in the judicial district in which the court hearing the case is located. Can the Plaintiff depose the witness? (A) Yes, but will need leave of court to do so (B) Yes, but must first procure in nuance of a subpoena (C) Yes, but must provide reasonable written notice of the time and place of the deposition (D) Yes, but the deposition can take place only in the district in which the deponent resides

C is correct because the federal discovery rules require that a requesting party give reasonable written notice to every other party to the action to examine a witness. See FRCP 30(b)(1). This rule applies whether or not the deponent is a party. Notice to all parties is required in order to give them an opportunity to cross-examine the deponent should they choose to do so. The notice must state the time and place for the taking of the deposition, but is not required to state the subject matter of the examination. See Madison v. Cobb, 29 F. Supp. 881 (M.D. Pa. 1939). Depositions are admissible only if a party was given notice or was represented at the deposition. See Howard v. Everex Sys., Inc., 228 F.3d 1057 (9th Cir. 2000). Plaintiff may depose the party witness even if Defendant resides outside the district. A different rule applies to the location of a deposition of a nonparty witness (which is not the case here). FRCP 45(c)(1) sets limits to how far a nonparty deponent may be required to travel. Subparagraph (A) limits it to within 100 miles of where the person resides, is employed, or regularly transacts business in person. Subparagraph (B) limits it to the state of the person's residence, employment, or regular in-person business transaction if the person is a party, a party's officer, or is commanded to attend a trial and wouldn't incur substantial expense. (A) is not correct because leave of court is generally not needed to take the deposition of a party. Deposing a party requires only that the requesting party serve the deponent with reasonable written notice of the time and place of the deposition. Leave of court is needed only if the requesting party will exceed the limit on depositions set by FRCP 30(a)(2)(A)(i); if the deponent has already been deposed in the case, see FRCP 30(a)(2)(A)(ii); if the deposition is noticed earlier than permitted under FRCP 26(d) (unless the deponent is "expected to leave the United States and be unavailable for examination"); or if the deponent is "confined in prison." See FRCP 30(a)(2)(B). (B) is not correct because the deposition of a party or officer of a party does not require a subpoena. The Advisory Committee Note states, "[d]epositions of parties, and officers, directors, and managing agents of parties need not involve use of a subpoena." Keep in mind that the rule is different for a nonparty deponent; a nonparty deponent is not compelled to appear for a deposition in the absence of a subpoena. See FRCP 45. (D) is not correct because it does not correctly state the rule governing the place of examination of a party deponent. The requesting party may choose to locate the deposition anywhere it wishes, although the party deponent is free to move for a protective order to change the location. Moreover, both the requesting party and the party deponent may move the court for an order to take a deposition remotely (for example, by telephone or videoconference). See FRCP 30(b)(4). When there is a dispute about where the deposition will take place, the decision "is ultimately an exercise in the vast discretion a district court has in supervising discovery."New Medium Techs. LLC v. Barco N.V., 242 F.R.D. 460, 462 (N.D. Ill. 2007). Keep in mind that a different rule applies to the location of a deposition of a nonparty witness. FRCP 45(c)(1) sets limits to how far a nonparty deponent may be required to travel. Subparagraph (A) limits it to within 100 miles of where the person resides, is employed, or regularly transacts business in person. Subparagraph (B) limits it to the state of the person's residence, employment, or regular in-person business transaction if the person is a party, a party's officer, or is commanded to attend a trial and wouldn't incur substantial expense.

Plaintiff buys a sweater from Defendant, who runs a mail-order clothing business. After wearing the sweater, Plaintiff develops rashes and later suffers neurological damage. She sues Defendant in federal court alleging that the sweater was treated with chemicals that caused her injury. Defendant serves Plaintiff with a Request for a Physical Examination, and Plaintiff refuses to comply. Defendant moves to compel discovery. How should the district court resolve the dispute and why? (A) The district court should grant the discovery request, because a physical examination is relevant to the claims and defenses in the action (B) The district court should deny the discovery request, because conditioning Plaintiff's right to bring suit upon her submission to a court-ordered physical examination violates her right to bodily integrity (C) The district court should deny the discovery request, because Defendant did not move for such discovery and show good cause (D) The district court should grant the request and treat Plaintiff's refusal to appear for an examination as a contempt of court

C is correct because the physical or mental examination of a party will take place only upon court order if the requestor is able to show good cause for the request; when the matter is contested, granting the request falls within the trial court's discretion. As with any discovery request, the information sought must be relevant to the claims and defenses in the action. See FRCP 26(b). In addition, the federal rules impose other requirements before this kind of discovery will be permitted. The physical or mental examination of a party can take place "only on motion for good cause and on notice to all parties and the person to be examined," and must also specify, among other things, the "scope of the examination, as well as the person ... who will perform it." FRCP 35(a)(2)(A) & (B). Here you are told that Defendant simply served Plaintiff with the Request for a Physical Examination, treating this discovery device as if it were a deposition. Had Defendant made a proper motion, then the court would have considered such questions as whether Plaintiff's condition is in controversy, whether information about the condition can be obtained from other sources, and whether Plaintiff's right to privacy is out-weighed by the requestor's need for the information. See Schlagenhauf v. Holder, 379 U.S. 104 (1964). (A) is not correct because it misstates the rule that applies to requests for physical examinations. It is true that that the general scope of discovery includes "any non-privileged information that is relevant to any party's claim or defense." However, relevance alone is not sufficient to secure an order for a physical or mental examination; the condition to be examined must also be "in controversy." As the Court has explained, "The specific requirement of good cause would be meaningless if good cause could be sufficiently established by merely showing that the desired materials are relevant, for the relevancy standard has already been imposed by Rule 26(b)." Schlagenhauf v. Holder, 379 U.S. 104, 118 (1964) (quoting Guilford Nat'l Bank of Greensboro v. S. Ry. Co., 297 F.2d 921, 924 (4th Cir. 1962)). On the facts presented, Plaintiff certainly has made her medical condition an issue in the lawsuit, and the pleadings may be sufficient to show that the issue is "in controversy." But questions remain about the kind of examination to authorize, the nature of the procedures, and the identity of the examining physician. (B) is not correct because it misstates the governing legal principle. Any doubts as to the constitutionality of FRCP 35 and its authorization of physical and mental examinations upon court order were put to rest by the Supreme Court in Sibbach v. Wilson & Co., 312 U.S. 1 (1941). In this case, plaintiff sued for damages, and defendant moved for an order requiring a medical examination; plaintiff refused to appear for the examination and was held in contempt. On appeal, she argued that Congress was without power to enact a discovery rule that interfered with state privacy protections, a position that the Supreme Court emphatically rejected: "The suggestion that the rule offends the important right to freedom from invasion of the person ignores the fact that as we hold, no invasion of freedom from personal restraint attaches to refusal so to comply with its provisions." Id. at 14. Nevertheless, as explained in connection with Choice (C), the court is expected to limit the use of this discovery device and for this reason requires a showing of good cause. Moreover, upon demand, the examined party may obtain a copy of the examiner's report. See FRCP 35(b). (D) is not correct because it misstates both the procedures governing use of the rule providing for physical or mental examinations and the sanctions available for a party who resists examination. The examination of a person is permitted under FRCP 35 only if the requestor of the information makes a "motion for good cause and on notice to all parties and the person to be examined"; the motion must further "specify the time, place, manner, conditions, and scope of the examination, as well as the person or persons who will perform it." SeeFRCP 35(a)(2)(A)-(B). In its discretion, the court "may order" the examination. At that point, if the examined party refuses to comply she must seek a protective order; simply refusing to comply potentially triggers the sanctions that are available for failure to comply with any court order for discovery, with the exception of imprisonment for contempt. See FRCP 37(b)(2)(A)(vii).

Plaintiff, a State A resident, worked for a Delaware corporation for over 25 years. After working tirelessly for six months in anticipation of a promotion, Plaintiff was informed that the company had hired a new employee for the coveted position. Believing that she had been passed over based on her age and gender, Plaintiff filed suit against the corporation under federal and state anti-discrimination laws. Plaintiff also filed a claim against her district manager, a State A resident, alleging breach of employment contract, seeking $50,000 in damages. Plaintiff filed her action in federal court in State A. The district manager moved to dismiss for lack of subject matter jurisdiction. How should the court rule on this motion? (A) The court should deny the motion, because the federal anti-discrimination laws confer subject matter jurisdiction over this case. (B) The court should deny the motion, because diversity jurisdiction is satisfied between Plaintiff and the district manager. (C) The court should deny the motion, and in its discretion exercise supplemental jurisdiction over the contract claim against the district manager. (D) The court should grant the motion, because it cannot exercise supplemental jurisdiction over the district manager.

C is correct because this scenario demonstrates when a federal court has power to exercise supplemental jurisdiction. Supplemental jurisdiction provides federal court jurisdiction over claims which lack an independent basis for federal subject matter jurisdiction but that arise from the same set of operative facts as the principal claim validly asserted in federal court. Under § 1367A, "in any civil action of which the district courts have original jurisdiction, the district courts shall have supplemental jurisdiction over all other claims that are so related to claims in the action within such original jurisdiction that they form part of the same case or controversy under Article III of the United States Constitution." The exercise of such power is subject to exceptions set out in § 1367B when the original claim is based on diversity jurisdiction. Here, the court's original jurisdiction is based on a claim that arises under federal law; the claim against the district manager is related to that claim because it involves the conditions of Plaintiff's employment and loss of promotion at work, and thus falls under the federal court's supplemental jurisdiction. A is incorrect because it incorrectly states that Plaintiff's claim falls under the federal anti-discrimination statute. Plaintiff's original claim against her employer falls under the federal anti-discrimination statute but that does not cover her breach of contract claim against her district manager. Plaintiff's claim against her district manager does not arise under any federal statute, but is instead based on state law contract claims. In order for the court to entertain the state law contract claim against the district manager (since no diversity exists), the court would need to exercise supplemental jurisdiction over the state law claims. B is incorrect because it incorrectly applies the test for federal diversity jurisdiction. Federal courts can invoke diversity jurisdiction to adjudicate claims that are reasonably likely to exceed $75,000 between citizens of different states. Both Plaintiff and her district manager are citizens of Massachusetts, so diversity of citizenship is not present. D is incorrect because the federal statute authorizing the exercise of supplemental jurisdiction (28 U.S.C. § 1367) provides that the scope of such power "shall include claims that involve the joinder or intervention of additional parties," subject to exclusions that apply when the anchor claim is based on diversity jurisdiction. Section 1367 permits the exercise of supplemental jurisdiction over a party later joined in a lawsuit, subject to exclusions set out in 28 U.S.C. § 1367B.

Damien was a student attending State A University. While at school, Damien lived in a dormitory on campus. During holidays and breaks, Damien returned to his hometown in State B. During his senior year in college, Damien decided to move to State A permanently so he began looking for an off-campus apartment. After graduating, Damien was recruited by a multi-national advertising firm based in State C. Damien was recruited to head up their State D office. While travelling to State D to begin searching for an apartment, Damien was in a car accident which required a six month stay at a local State A hospital. Plaintiff wants to file a diversity action against Damien. Where is Damien domiciled? (A) State A, where Damien has been a student for 4 years and an additional six months for hospitalization (B) State D, where Damien is planning on living and working after his release from the hospital (C) State B, where he lived prior to attending college (D) State C, where his current employer is incorporated and has its principal place of business

C is correct. An individual is domiciled where his true, fixed, and permanent place of residence is located. Domicile is the place the person intends to return whenever they are away for a period of time. However, intent alone is not enough. The individual would have to take affirmative steps to establish a physical residence. Before establishing a new domicile, the person maintains their previous one. Here, Damien was domiciled in State B and although he lived in State A for four years, living in a dormitory on campus is not a permanent place of residence. Even after deciding to move to State A permanently, Damien had not yet established a permanent residence. Also, Damien did not establish a physical residence in State D. Damien's employer's domicile is irrelevant. Thus, since Damien has not established a new domicile, he keeps his old one, which is still State B. A is incorrect. An individual is domiciled where his true, fixed, and permanent place of residence is located, and is the place the person intends to return whenever they are away for a period of time. However, intent alone is not enough. The individual would have to take affirmative steps to establish a physical residence. Before establishing a new domicile, the person maintains their previous one. Here, Damien was a student in State A for four years, living in a dormitory on campus, which is not a permanent place of residence. Even after deciding to move to State A permanently, Damien had not yet established a permanent residence. Thus, State A is not his domicile. B is incorrect. An individual is domiciled where his true, fixed, and permanent place of residence is located. Domicile is the place the person intends to return whenever they are away for a period of time. However, intent alone is not enough. The individual would have to take affirmative steps to establish a physical residence. Before establishing a new domicile, the person maintains their previous one. Here, Damien was domiciled in State B. Even though Damien was recruited to work in State D, Damien did not yet establish a physical residence there. D is incorrect. An individual is domiciled where his true, fixed, and permanent place of residence is located. Domicile is the place the person intends to return whenever they are away for a period of time. However, intent alone is not enough. The individual would have to take affirmative steps to establish a physical residence. Before establishing a new domicile, the person maintains their previous one. Here, Damien was domiciled in State B. Damien's employer's domicile is irrelevant.

Plaintiff believes that she has been discriminated against on the basis of her sex in violation of both federal and state law. However, she wants to avoid being in federal court and so she files a complaint in state court asserting only a claim under the state anti-discrimination statute. Assuming the parties are not of diverse citizenship, will she be successful in staying out of federal court? (A) No, because she could state a claim arising under federal law. (B) No, because of the "artful pleading" doctrine. (C) Yes, because of the "master of the complaint" doctrine. (D) Yes, because the parties are not diverse.

C is correct. As the master of her complaint, the plaintiff has the right to choose which claims to assert and not assert. Assuming that a well-pleaded state law claim does not contain a substantial issue of federal law, as this would not, then the plaintiff can refuse to assert the federal claim to avoid having the defendant remove the case. A is incorrect because the fact that the plaintiff could state a federal claim is irrelevant. The plaintiff has the right to choose which claims to assert and which claims to forego. B is incorrect. Although the plaintiff has the right to choose which claims to assert and which not to assert, there is an exception to the master of the complaint doctrine, the so-called artful pleading doctrine. This states that when the plaintiff's purported state law claim is essentially a federal law claim disguised as a state law claim, it must be treated as a federal claim for subject matter jurisdictional purposes. But this occurs only when the purported state law claim asserts a right to relief in a subject that has been preempted by state law. The field of anti-discrimination law is not one in which federal law preempts any effort at state regulation. D is incorrect. The fact that the parties are not diverse would not preclude federal jurisdiction if the plaintiff's claim was "essentially" a federal claim disguised as a state law claim.

Plaintiff, from State A, sued Defendant, also from State A, in federal district court. Plaintiff alleged that Defendant caused damages in excess of $10,000,000 by violating Plaintiff's patent rights on a new electronic tablet design and software. Defendant filed a motion to dismiss challenging the court's subject matter jurisdiction over the case. How should the court rule? (A) The court should grant the motion, because both parties are citizens of State A. (B) The court should grant the motion, because the court lacks personal jurisdiction over Defendant. (C) The court should deny the motion, because the federal court has jurisdiction over the case. (D) The court should deny the motion, because the damages are well over $75,000.

C is correct. Federal Courts have exclusive jurisdiction over certain types of claims, including patent rights, securities, antitrust, and claims arising under the Federal Torts Claim Act. Exclusive jurisdiction is the federal court's exclusive power to adjudicate the claims. Here, Plaintiff is alleging a violation of patent rights, which falls under the federal court's exclusive jurisdiction. A and D are incorrect because jurisdiction is not based on diversity jurisdiction. Diversity jurisdiction provides litigants an avenue to federal court where the adversaries are completely diverse in citizenship and the amount in controversy is reasonably likely to exceed $75,000. Here, the cause of action arises under federal law, one which is exclusive to the federal court's jurisdiction, regardless of state of citizenship or amount in controversy. B is incorrect because Defendant's motion is not challenging the court's personal jurisdiction over the person but rather the subject matter over the controversy. Subject matter jurisdiction is the court's power over the controversy (here, patent rights violation), not the person (Defendant). Since the challenge is to the court's power over the controversy and not over the defendant, this answer choice is non-responsive to the question and is, therefore, incorrect.

Plaintiff, a citizen of State A, filed suit in federal district court against Defendant, citizen of State B alleging one count of negligence resulting from a car accident and one count for breach of contract resulting from an agreement the parties entered into a year prior to their accident. Plaintiff alleged damages to his car and person in the amount of $50,000. The contract called for liquidated damages in the amount of $35,000. Defendant filed a motion to dismiss challenging the court's subject matter jurisdiction over the controversy. How should the court rule? (A) The court should grant Defendant's motion, because Defendant lacks sufficient minimum contacts to be bound by the court's judgment. (B) The court should grant the motion, because Plaintiff cannot aggregate their damages from two unrelated claims in order to satisfy the amount-in-controversy requirement for diversity jurisdiction. (C) The court should deny the motion, because Plaintiff can aggregate their damages from two unrelated claims in order to satisfy the amount-in-controversy requirement for diversity jurisdiction. (D) The court should deny the motion, because Plaintiff and Defendant are from different states, thereby satisfying the complete diversity of citizenship required for subject matter jurisdiction.

C is correct. Federal courts have subject matter jurisdiction over state law claims where the adverse parties are diverse in citizenship (meaning no plaintiff is from the same state as any defendant) and the controversy is reasonably likely to exceed $75,000. In order to satisfy the threshold amount, aggregation of damages is permitted in unrelated casesbetween the same parties. A is incorrect because Defendant is not challenging the court's jurisdiction over the person but rather the controversy. Subject matter jurisdiction is the court's power over the controversy (here, through diversity jurisdiction), not the person (Defendant). Since the challenge is to the court's power over the controversy and not over Defendant, this answer choice is non-responsive to the question and is, therefore, incorrect. B is incorrect because the answer misstates the law. Federal courts have subject matter jurisdiction over state law claims where the adverse parties are diverse in citizenship (meaning no plaintiff is from the same state as any defendant) and the controversy is reasonably likely to exceed $75,000. In order to satisfy the threshold amount, aggregation of damages is permitted in unrelated casesbetween the same parties. Therefore, this answer choice is incorrect. D is incorrect because it is an incomplete response. Federal courts have subject matter jurisdiction over state law claims where the adverse parties are diverse in citizenship (meaning no plaintiff is from the same state as any defendant) and the controversy is reasonably likely to exceed $75,000. In order to exercise proper diversity jurisdiction, both requirements must be satisfied. Answer choice D suggests that complete diversity of citizenship alone is enough for federal subject matter jurisdiction, which is incorrect.

Plaintiff, a citizen of State A, filed suit in federal district court against Defendant, citizen of State B alleging one count of negligence resulting from a car accident and one count for breach of contract resulting from an agreement the parties entered into a year prior to their accident. Plaintiff alleged damages to his car and person in the amount of $50,000. The contract called for liquidated damages in the amount of $35,000. Defendant filed a motion to dismiss challenging the court's subject matter jurisdiction over the controversy. How should the court rule? (A) The court should grant Defendant's motion, because Defendant lacks sufficient minimum contacts to be bound by the court's judgment. (B) The court should grant the motion, because Plaintiff cannot aggregate their damages from two unrelated claims in order to satisfy the amount-in-controversy requirement for diversity jurisdiction. (C) The court should deny the motion because Plaintiff and Defendant are citizens of different states, and Plaintiff can aggregate his damages from his two unrelated claims in order to satisfy the amount-in-controversy requirement for diversity jurisdiction. D) The court should deny the motion because the court has diversity jurisdiction over Plaintiff's negligence claim and supplemental jurisdiction over Plaintiff's breach of contract claim.

C is correct. Federal courts have subject matter jurisdiction over state law claims where the adverse parties are diverse in citizenship (meaning no plaintiff is from the same state as any defendant) and the controversy is reasonably likely to exceed $75,000. In order to satisfy the threshold amount, aggregation of damages is permitted in unrelated claims between the same parties. A is incorrect because Defendant is not challenging the court's jurisdiction over the person but rather the controversy. Subject matter jurisdiction is the court's power over the controversy (here, through diversity jurisdiction), not the person (Defendant). Since the challenge is to the court's power over the controversy and not over Defendant, this answer choice is non-responsive to the question and is, therefore, incorrect. B is incorrect because the answer misstates the law. Federal courts have subject matter jurisdiction over state law claims where the adverse parties are diverse in citizenship (meaning no plaintiff is from the same state as any defendant) and the controversy is reasonably likely to exceed $75,000. In order to satisfy the threshold amount, aggregation of damages is permitted in unrelated claimsbetween the same parties. Therefore, this answer choice is incorrect. D is incorrect because the amount in controversy is too low with respect to the negligence claim for there to be diversity jurisdiction over that claim alone. Also, supplemental jurisdiction could not apply to the breach of contract claim because there is no valid anchor claim to which the contract claim could attach, and the contract claim is unrelated to the negligence claim. In any event, the use of supplemental jurisdiction is unnecessary here because the two claims can be aggregated to meeting the requirements for diversity jurisdiction.

The plaintiff, a citizen of State A, brings a state breach of contract action in federal district court alleging that the defendant, also a citizen of State A, agreed to purchase his home and subsequently refused to go forward with the deal. In his complaint, the plaintiff alleges that even if he misrepresented important features of the home in violation of the requirements of federal disclosure law, this statute is unconstitutional. Does the court have subject matter jurisdiction? (A) Yes, because this case arises under federal law. (B) Yes, because the defendant filed an answer, waiving his jurisdictional objection. (C) No, because this case does not arise under federal law and the parties are citizens of the same state. (D) No, because the parties are citizens of the same state.

C is correct. Federal subject matter jurisdiction is based on federal question issue or diversity jurisdiction. When seeking jurisdiction under federal question, the federal question must be part of the plaintiff's cause of action, not any alleged or anticipated defense. Although a federal constitutional issue is raised in the plaintiff's complaint, it is only there as a response to an anticipated federal defense. Here, this action is a basic breach of contract claim by the plaintiff, which arises under state law. A is incorrect. Although a federal constitutional issue is raised in the plaintiff's complaint, it is only there as a response to an anticipated federal defense. Thus, it is not deemed to have arisen within a well-pleaded complaint. Consequently, it does not arise under federal law under §1331. D is incorrect. While it is true that the parties are not of diverse citizenship, the court would have federal question jurisdiction under §1331 if the federal question met the well-pleaded complaint rule, i.e., the federal question was part of the plaintiff's prima facie case. B is incorrect. Objections to subject matter jurisdiction can never be waived pursuant to FRCP 12(h)(3).

The plaintiff, a citizen of State A, brought a state breach of contract action in federal district court alleging that the defendant, also a citizen of State A, agreed to purchase his home and subsequently refused to go forward with the deal. The defendant's answer admits that he refused to buy the home but alleges that because the plaintiff had lied in certain representations about the home contained in a federally required disclosure form, the deal was unenforceable under the governing federal disclosure statute. Does the court have subject matter jurisdiction? (A) Yes, because this case arises under federal law. (B) Yes, because the defendant filed an answer, waiving his jurisdictional objection. (C) No, because this case does not arise under federal law. (D) No, because the parties are citizens of the same state.

C is correct. Federal subject matter jurisdiction is based on federal question issue or diversity jurisdiction. When seeking jurisdiction under federal question, the federal question must be part of the plaintiff's cause of action, not any alleged or anticipated defense. The federal issue concerning the federal disclosure statute comes into the case only as a defense issue and therefore the federal question is not part of a well-pleaded complaint. Here, this action is a basic breach of contract claim by the plaintiff, which arises under state law. A is incorrect. The federal issue concerning the federal disclosure statute comes into the case only as a defense issue and therefore the federal question is not part of a well-pleaded complaint. Thus, it does not arise under federal law under §1331. D is incorrect. While it is true that the parties are not of diverse citizenship, the court would have federal question jurisdiction under §1331 if the federal question met the well-pleaded complaint rule, i.e., the federal question was part of the plaintiff's prima facie case. B is incorrect. Objections to subject matter jurisdiction can never be waived pursuant to FRCP 12(h)(3).

David was arrested at an anti-war rally. While the police officer was arresting David, an altercation between the two broke out. David claims that the police officer used excessive force in making the arrest. As a result of the arrest and altercation, David incurred damages totaling $7,500 for medical expenses. David filed suit against the police officer for violation of his federal constitutional rights in state court. The police officer filed a motion to dismiss for lack of subject matter jurisdiction. How should the court rule? (A) The court should grant the police officer's motion to dismiss, as federal courts have exclusive jurisdiction over federal civil rights violations. (B) The court should grant the police officer's motion to dismiss, as federal courts have exclusive jurisdiction over federal questions based on constitutional rights. (C) The court should deny the police officer's motion to dismiss, as state and federal courts have concurrent jurisdiction over most federal questions, including this one. (D) The court should deny the police officer's motion to dismiss, but only if David includes the state law claims for assault and battery.

C is correct. Generally, the state and federal courts have concurrent jurisdiction over federal question cases. In a few situations, federal statutes make federal jurisdiction exclusive over certain types of claims. In most situations, including a civil rights claim such as this one, state and federal courts have concurrent jurisdiction so the plaintiff can choose to file the case either in state court or federal court. Therefore, the state court has jurisdiction and the motion should be denied. A and B are incorrect. Federal jurisdiction is not exclusive over federal questions or federal constitutional rights. There are certain types of cases within the federal court's exclusive jurisdiction, such as patent rights or antitrust, but not constitutional right violations. D is incorrect. David need not have a state law based claim to authorize state court jurisdiction since state courts and federal courts have concurrent jurisdiction over these civil rights claims.

Dennis was arrested at an anti-war rally. While the police officer was arresting Dennis, an altercation between the two broke out. Dennis claims that the police officer used excessive force in making the arrest. As a result of the arrest and altercation, Dennis incurred damages totaling $7,500 for medical expenses. Dennis filed suit against the police officer for violation of his federal constitutional rights in state court. The police officer filed a motion to dismiss for lack of subject matter jurisdiction. How should the court rule? (A) The court should grant the police officer's motion to dismiss, as federal courts have exclusive jurisdiction over federal civil rights violations. (B) The court should grant the police officer's motion to dismiss, as federal courts have exclusive jurisdiction over federal questions based on constitutional rights. (C) The court should deny the police officer's motion to dismiss, as state and federal courts have concurrent jurisdiction over most federal questions, including this one. (D) The court should deny the police officer's motion to dismiss, but only if Dennis includes the state law claims for assault and battery.

C is correct. Generally, the state and federal courts have concurrent jurisdiction over federal question cases. In a few situations, federal statutes make federal jurisdiction exclusive over certain types of claims. In most situations, including a civil rights claim such as this one, state and federal courts have concurrent jurisdiction so the plaintiff can choose to file the case either in state court or federal court. Therefore, the state court has jurisdiction and the motion should be denied. A and B are incorrect. Federal jurisdiction is not exclusive over federal questions or federal constitutional rights. There are certain types of cases within the federal court's exclusive jurisdiction, such as patent rights or antitrust, but not constitutional right violations. D is incorrect. Dennis need not have a state law based claim to authorize state court jurisdiction since state courts and federal courts have concurrent jurisdiction over these civil rights claims.

Plaintiff, from State M, sued his employer, Del Corp., a Delaware corporation with its principal place of business in State O, for wrongful termination. Plaintiff's attorney prepared the complaint on May 1st, but did not email a copy of the prepared complaint to Del Corp. until August 2nd. After Del Corp did not respond to the email, Plaintiff had his attorney file the complaint in State M state court, seeking $100,000 in damages, and properly served Del Corp on August 15th. On August 16th, Del Corp. removed the case to federal court. Plaintiff filed a motion for remand arguing the time for removal had lapsed. How should the court rule? (A) The court should grant the motion, as the defendant had until May 31st to remove the case. (B) The court should deny the motion, as the defendant has until September 1st to remove the case. (C) The court should deny the motion, as the defendant has until September 14th to remove the case. (D) The court should deny the motion, as the defendant can remove a case from state court to federal court at any time prior to trial.

C is correct. Pursuant to 28 U.S.C. §1446, the defendant has 30 days after receiving the complaint through proper service of process to remove the case to federal court. Del Corp. was served on August 15th, which began the 30-day clock. Thirty days from that date is September 14. A is incorrect. This answer seems to suggest that the 30 days begin to run from the drafting of the complaint, which is incorrect. The removal statute allows for 30 days only after the defendant has been served with process. B is incorrect. While Del Corp. received a copy of the prepared complaint via email on August 2nd, the 30-day removal clock does not begin until the defendant has been served (through formal or informal) process. D is incorrect. The defendant cannot remove the case at any time prior to trial. The removal statute requires removal within 30 days after the defendant is served with process.

An automotive engineer employed by Ford Motor Company designed the accelerator system for the Ford Focus car. Defects in the Ford Focus accelerator system are now at issue in a federal court class action lawsuit filed by consumer harmed in accidents where the accelerator malfunctioned. May the consumers take the engineer's deposition? (A) No, because the engineer's testimony qualifies as trial preparation material (B) No, because the engineer is a consulting expert hired in anticipation of litigation (C) Yes, because the engineer is a witness with both expertise and personal knowledge of the facts relevant to the lawsuit (D) Yes, because the engineer is a testifying expert who must prepare an export report and be available to be deposed about the findings in that report

C is correct. The engineer may be deposed because the engineer's testimony will consist of personal knowledge and expert opinion acquired as the accelerator designer during the design phase—not after the fact as a testifying or consulting expert. See Rule 26(b)(4)(A) and Rule 26(b)(4)(D). A is incorrect because oral witness testimony can never be trial preparation material. Rule 26(b)(3)(A)'s trial preparation material provision applies only to "documents and tangible things." B is incorrect because the engineer was not "retained or specially employed in anticipation of litigation"; rather, the engineer was a Ford employee all along. See Rule 26(b)(4)(D). D is incorrect because the engineer was not "retained or specially employed to provide expert testimony in the case," nor is the engineer "one whose duties as the party's employee regularly involve giving expert testimony." As a result, under Rule 26(a)(2)(B) and (C), the engineer does not need to provide an expert report, only a written summary of what the engineer knows and opines from having designed the accelerator.

Plaintiff filed a diversity suit in federal district court in State A against Defendant. The Plaintiff alleged that the Defendant failed to stop at a stop sign and ran over Plaintiff while she was crossing the crosswalk. Eyewitness was standing a few feet away from the scene of the accident and claims to have seen the Defendant speed through the stop sign without slowing down at the intersection. Plaintiff plans to call Eyewitness to testify at trial about everything he saw regarding the accident. Defendant plans to rigorously cross-examine Eyewitness at trial and sends Eyewitness a Notice of Deposition under FRCP 30 to take Eyewitness's deposition. Defendant also included with the Notice of Deposition a Request for Physical Examination under FRCP 35, in order to examine Eyewitness's eyesight. Eyewitness failed to appear for the scheduled eye exam and then refused to attend the deposition. The Defendant filed a motion to compel the Eyewitness to comply with both requests. How should the court rule? (A) The court should grant the Defendant's motion to compel attendance both as to the deposition and the physical examination (B) The court should grant the Defendant's motion to compel attendance as to the deposition but not as to the physical examination (C) The court should deny the Defendant's motion to compel attendance both as to the deposition and the physical examination (D) The court should deny the Defendant's motion to compel attendance as to the deposition but not as to the physical examination

C is the correct answer. Discovery is the method by which the parties exchange information. Under FRCP 30, a party may take the deposition of any person without leave of court. If the party is seeking to depose a non-party, then the requesting party may compel the deponent's attendance by subpoena under FRCP 45. Here, the Eyewitness is not a party to the suit and to compel their attendance at a deposition, the requesting party would need a subpoena under Rule 45. Defendant only sent Eyewitness a Notice of Deposition, which cannot compel Eyewitness's attendance. FRCP 35 authorizes a party to seek a physical examination but only from another party to the suit, for a condition that is "in controversy", and only after seeking leave of court to do so. Here, Defendant sent the Rule 35 request to Eyewitness, who is not a party to the suit and cannot be compelled to submit to a physical examination. Therefore, Answers A and B are incorrect. Answer D is incorrect because FRCP 35 authorizes a party to seek a physical examination but only from another party to the suit, for a condition that is "in controversy," and only after seeking leave of court to do so. Here, Defendant sent the Rule 35 request to Eyewitness, who is not a party to the suit and cannot be compelled to submit to a physical examination.

Plaintiff from State Z was a student at University located in State A. After Plaintiff attended University for two years, University suspended Plaintiff for allegedly cheating on a final examination. After a full hearing on the matter, University expelled Plaintiff. Plaintiff filed a 2-count complaint against University in State A state court - Count I was based on gender discrimination in violation of federal law and Count II was based on breach of contract, stemming from University's statement found in the student handbook regarding nondiscrimination. Plaintiff claimed $55,000 in damages. After service of process was properly effectuated, University removed the case to federal court in State A. Following the completion of all discovery, University filed a motion to dismiss for failure to state a claim upon which relief can be granted on both counts of the complaint. The federal district court granted University's motion to dismiss the federal gender discrimination claim, but denied it for the breach of contract claim. Plaintiff then filed a motion to remand the matter to state court. How should the court rule on the motion? (A) The court should deny the motion to remand because it was filed more than 30 days after the University removed the case to federal court. (B) The court must deny the motion to remand because the court has supplemental jurisdiction over Count II. (C) The court may decline to exercise supplemental jurisdiction over the breach of contract claim and remand the case to state court because the claim over which it exercised original jurisdiction has been dismissed. (D) The court should grant the motion to remand if Plaintiff can point to a procedural defect in the removal process under §1441.

C is the correct answer. Federal courts have subject matter jurisdiction over claims arising under federal law and state law claims falling under diversity jurisdiction. The Plaintiff's breach of contract claim is a state law claim that does not meet the requirements for diversity jurisdiction. Therefore, in order for the federal court to exercise jurisdiction over the state law claim, it can do so under the supplemental jurisdiction statute 28 U.S.C. §1367 so long as the state law claim arises from the same transaction or occurrence as a federal claim properly in federal court. Here, the plaintiff filed a claim against University alleging gender discrimination in violation of federal law. This gender discrimination is also the basis for the breach of contract claim. Therefore, the court could exercise supplemental jurisdiction under §1367. However, supplemental jurisdiction is always discretionary and the statute allows the federal court to decline to exercise jurisdiction under §1367(c) if, inter alia, the federal court dismisses all claims over which it has original jurisdiction—here, the federal discrimination claim. Answer A is incorrect because the 30-day period for remand applies to procedural defects other than the lack of subject matter jurisdiction. See 28 U.S.C. §1447(c). Answer B is incorrect because a federal court may decline to exercise supplemental jurisdiction over related state law claims for reasons listed in §1367(c) and thus is not required to keep the case. Answer D is incorrect because even if a procedural defect could be identified in the removal process, the Plaintiff would have had to have moved to remand on that basis within 30 days after notice of removal. This motion to remand was filed well outside the 30-day period allotted in §1447.

An accident injured the Plaintiff at work and severed several of his fingers while using a printing press machine. The printing press machine was manufactured by Print Co., Ltd. a foreign company. Print Co., Ltd. distributed the printing press machine in the United States through its exclusive U.S. distributor, Print U.S.A. Print U.S.A. then sells the printing presses to companies, including Plaintiff's employer, at tradeshows across the U.S. The tradeshows do not occur in State A. Plaintiff filed a products liability suit against Print Co., Ltd. and Print U.S.A. in federal district court in State A. Plaintiff is a citizen of State A; Print Co., Ltd. is incorporated and has its principal place of business in Foreign Country. Defendant Print Co., Ltd. removed the case from state court to the federal district court in State A. Defendant Print Co., Ltd. then filed a motion to dismiss for lack of personal jurisdiction. How should the court rule on the motion? (A) The court should deny the motion because by removing the case to federal court in State A, the Defendant waived all objections as to personal jurisdiction. (B) The court should deny the motion because the press machine caused an injury in State A. (C) The court should grant the motion because Print Co., Ltd. did not engaged in any activities in State A that revealed an intent to invoke or benefit from the protection of State A's laws. (D) The court should grant the motion because Print Co. is incorporated in and has its principal place of business in Foreign Country.

C is the correct answer. Personal jurisdiction is the power of the court to bind the defendant to its judgment. In order to exercise this jurisdictional power, due process requires that the defendant have sufficient contacts with the forum state such that the maintenance of the suit within that forum will not offend traditional notions of fair play and substantial justice. See International Shoe Co. v. Washington, 326 U.S. 310 (1945). To determine whether a defendant has sufficient contacts with a forum, courts are directed to consider whether defendants purposefully availed themselves of the privilege of engaging in activities in the forum thereby invoking the benefits and protections of the forum's laws. See Hanson v. Denckla, 357 U.S. 235 (1958). A majority of the Supreme Court has never held that placing a product into the "stream of commerce" alone is sufficient to subject a defendant to jurisdiction in the forum state without additional activity directed at the forum itself (advertising, direct sales, etc.). See Asahi Metal Industry Co. v. Sup. Ct. of Ca., 480 U.S. 102 (1987), J. McIntyre Machinery, Ltd. v. Nicastro, 564 U.S. 873 (2011). Here, Print Co., Ltd. a foreign company, has not advertised or directed any efforts to State A in order to invoke or benefit from the protections of State A. See Nicastro where a plurality of the Court held that "[d]ue process protects petitioner's right to be subject only to lawful authority. At no time did petitioner engage in any activities in New Jersey that reveal an intent to invoke or benefit from the protection of its laws." Id. at 887. Answer A is incorrect because a defendant's right to remove a case from state court to federal court is not a "defensive" move and does not waive jurisdictional objections. See Danziger & De Llano, LLP v. Morgan Verkamp, LLC, 948 F.3d 124 (3d Cir. 2020). Answer B is incorrect because a majority of the Court has never held that an injury occurring in the forum state is alone sufficient to establish personal jurisdiction. Instead, there needs to be some act by which the defendant purposefully avails itself of the privilege of conducting activities in the forum. Answer D is incorrect because foreign companies can be subject to personal jurisdiction in the U.S., so long as the requirements of due process are satisfied, which they were not in this case.

A large group of in-state and out-of-state Plaintiffs filed suit against Pharma Co. in federal district court in State A, alleging one of its products caused severe internal organ damage. Pharma Co. is incorporated in State Z and has its principal place of business in State Y. The group of plaintiffs consisted of 100 residents of State A who used the product in State A and 350 residents of other states who were prescribed and used the product in their respective home states. Shortly after being served with the complaint, Pharma Co. filed a motion to dismiss the claims from the out-of-state residents for lack of personal jurisdiction. How should the court rule on Pharma Co.'s motion to dismiss? (A) The court should deny the motion as State A has general jurisdiction over Pharma Co. (B) The court should deny the motion as State A has specific jurisdiction over Pharma Co. (C) The court should grant the motion as State A does not have personal jurisdiction over Pharma Co. with respect to the out-of-state residents' claims. (D) The court should grant the motion as Pharma Co. is incorporated in State Z and has its principal place of business in State Y.

C is the correct answer. State A (the forum state) does not have personal jurisdiction over the defendant with respect to the out-of-state residents. In Bristol-Myers Squibb Co. v. Superior Court, 582 U.S. ___ (2017), the Court held that in order for a court within a forum to exercise specific jurisdiction, the suit must arise out of or relate to the defendant's contacts with the forum. Here, State A would have specific jurisdiction over the State A residents' claims against Pharma Co. but it would not have jurisdiction over the out-of-state residents' claims because those injuries did not arise from or relate to Pharma Co.'s activities in State A. Therefore, for this reason Answer C is correct and Answer B is incorrect. Answer A is incorrect because a corporation's state of incorporation and principal place of business determine which state has general jurisdiction over the defendant. Pharma Co. is incorporated in State Z with a principal place of business in State Y, meaning State Z and State Y would have general jurisdiction over Pharma Co., not State A. Answer D is incorrect because, while Pharma Co. is considered "at home" in 2 states (Z and Y) for purposes of general jurisdiction, this answer does not take into account that Pharma Co. could be subject to specific jurisdiction in other states if the contacts in the forum state gave rise to the cause of action.

The plaintiff and defendant assert ownership to the same home located in California. The plaintiff is a California citizen and the defendant, the plaintiff's former best friend, is a citizen of West Virginia. Plaintiff knows defendant's home address in West Virginia. The plaintiff files suit in federal court in California and the defendant refuses to waive service. The plaintiff obtains a court order seizing the property and serves the defendant by publication in a Los Angeles publication. The defendant, who stopped corresponding with the plaintiff one week before suit was brought, files a motion to dismiss for insufficient service. Should the court grant the defense motion? (A) No, because the property was seized. (B) No, because service was effected by publication. (C) Yes, because the plaintiff knew the defendant's location and could have mailed service to the defendant's home address. (D) Yes, because this is an in personam proceeding.

C is the correct answer. The due process requirements of the Fourteenth Amendment must be applied to determine the constitutional sufficiency of notice regardless of whether the proceeding is deemed to be in personam, in rem, or quasi in rem. Thus, although this is an in rem case, mere seizure plus publication is not constitutionally sufficient under these circumstances. The constitutional requirement for notice is that notice which is reasonably calculated under all the circumstances to apprise interested parties of the pendency of the action. Since the defendant's whereabouts were known to the plaintiff, the ruling in Mullane would require more than publication—the plaintiff would have to mail service to the defendant's known home address. Answers A and B are incorrect because the defendant's whereabouts were known to the plaintiff; due process thus would require more than seizure of the property or publication. Answer D is incorrect because the action is not in personambut rather it is an in rem action, where the subject matter of the suit is the property itself.

Jill went on a vacation to Hawaii with her best friend, Patty. Patty is a resident of Chicago, Illinois. Jill had been a lifelong resident of Chicago but moved to Philadelphia, Pennsylvania, just before leaving for vacation. While they were in Hawaii, Jill and Patty rented motorcycles and during their ride, Patty's motorcycle ran into Jill's motorcycle while Jill was on it. Jill returned home and filed a civil action in federal court in Philadelphia asserting $200,000 tort claims against Patty and Bikes, Inc., the Hawaii rental company that is incorporated under the laws of Delaware. Patty has never been to Pennsylvania and never had any contact with the state. Although Bikes, Inc. operates a highly interactive web site, no one in Pennsylvania has ever visited that site or purchased any products from the company. Does venue lie in the chosen forum? (A) Yes, because Jill is a resident of Philadelphia. (B) Yes, because Bikes, Inc. is subject to personal jurisdiction in Pennsylvania. (C) No, neither of the defendants resides there and the accident occurred in Hawaii. (D) No, because Patty is not subject to personal jurisdiction in Pennsylvania.

C is the correct answer. This case involves two defendants—one natural person and one corporation. Since residence is the relevant concept for venue purposes, we need to know the definition of residence for a corporate defendant. Under the terms of §1391(c)(2), a corporate defendant resides in any federal judicial district in which the corporation is subject to personal jurisdiction. And in multi-district states, the corporation is deemed to reside in any district in that state within which its contacts would be sufficient to subject it to personal jurisdiction if that district were considered to be a separate state. Here, defendant Patty is a resident of the Northern District of Illinois (Chicago). The defendant rental company clearly resides in Hawaii and is also subject to personal jurisdiction in the state under whose laws it is incorporated—Delaware. But it does not have any contact with Pennsylvania that would justify determining that it would be subject to personal jurisdiction in any part of Pennsylvania. Consequently, Bikes, Inc. does not reside in the Eastern District of Pennsylvania (Philadelphia). Therefore, we now have two defendants, each of whom resides in different states. Thus, residence of defendants is not useful for venue purposes. The events occurred in Hawaii so that will not support jurisdiction in the Eastern District of Pennsylvania (Philadelphia). And since the events occurred in Hawaii, venue would not lie in Philadelphia under §1391(b)(2). We cannot even consider the fall-back provision of (b)(3) since venue is available elsewhere under subsections (1) or (2), here in Hawaii. Answer A is incorrect because the residence of the plaintiff is irrelevant under the federal venue statute. Answer B is incorrect because the company is not subject to personal jurisdiction in Pennsylvania. Bikes, Inc. does not have any contact with Pennsylvania that would justify determining that it would be subject to personal jurisdiction in any part of Pennsylvania. Consequently, Bikes, Inc. does not reside in the Eastern District of Pennsylvania (Philadelphia). Answer D is incorrect because it is irrelevant. Since we have two defendants residing in different states, we cannot consider §1391(b)(1), Here, the residence of the defendants is not useful for venue purposes.

Family, from State A, was in the process of moving into their new home in State A when an issue with the plumbing caused the home to become temporarily uninhabitable. While the family hired a certified plumbing company to rectify the issue, the family moved into a hotel suite in State B but only miles from their new home. During their hotel stay, a diversity action was filed against the parents relating to a car accident they had been involved in 2 years prior. While the family was visiting the local zoo, a process server went to the hotel where the family was staying and left the paperwork with the hotel housekeeping staff with the explicit instruction to hand deliver the envelope to the parents upon their return to their room. Knowing that the parents made coffee every morning, the housekeeping staff left the envelope on top of the room's coffee machine. As soon as the family returned from their day trip that same evening, the parents saw and opened the envelope. The following week, the parents filed a motion to dismiss. How should the court rule on the motion? (A) The court should deny the motion because the parents are subject to personal jurisdiction in State A. (B) The court should deny the motion because service of process was conducted in accordance with the rules. (C) The court should grant the motion because service of process was not conducted in accordance with the rules. (D) The court should grant the motion because the parents received service of process while present in State B.

C is the correct answer. Under FRCP 4(e), Serving an Individual Within a Judicial District of the United States. Unless federal law provides otherwise, an individual—other than a minor, an incompetent person, or a person whose waiver has been filed—may be served in a judicial district of the United States by: (1) following state law for serving a summons in an action brought in courts of general jurisdiction in the state where the district court is located or where service is made; or (2) doing any of the following: (A) delivering a copy of the summons and of the complaint to the individual personally; (B) leaving a copy of each at the individual's dwelling or usual place of abode with someone of suitable age and discretion who resides there; or (C) delivering a copy of each to an agent authorized by appointment or by law to receive service of process. Here, the process server left the envelope with the hotel housekeeping staff, which is not authorized by appointment or by law to receive service on behalf of the parents. Even if the court were to find that the hotel room was the family's "dwelling or usual place of abode" (not very likely), the rule requires that service be left with someone of suitable age and discretion who resides there. The housekeeping staff is, presumably, of suitable age but does not reside in the family's hotel room. Therefore, C is correct and Answer B is incorrect. Answer A is incorrect because being subject to personal jurisdiction does not rectify the service of process deficiencies present under these facts. A party may be subject to personal jurisdiction but service of process must still satisfy the Constitution and the FRCP. Answer D is incorrect because receiving service while present in the state does not rectify the deficiencies present under these facts. A party may receive service while in the state but service of process must still satisfy the Constitution and the FRCP.

In a tort action filed in a federal district court in Alabama by an Alabama plaintiff against a citizen of California, the defendant refuses to waive service of process. The accident that gave rise to the lawsuit occurred in Mississippi. The plaintiff wants to serve the defendant while the defendant is on vacation in New York City. Which statute governs the sufficiency of service in this case? (A) California law, because the defendant is a citizen of that state. (B) Federal statute, because suit was brought in a federal court. (C) New York law, because the defendant is being served there. (D) Mississippi law, because the accident occurred in Mississippi.

C is the correct answer. Under Fed. R. Civ. P. 4(e)(1), in civil actions filed in federal court, service must be made either pursuant to governing federal law, the state statute of the forum state (Alabama) or the state in which service is effected, or the specific provisions of Rule 4(e)(2) of the Federal Rules of Civil Procedure. Since service is sought to be effected in New York, C is the correct answer. Answers A and D are incorrect because we must look to the state law of either the forum (Alabama) or the state in which service is made (New York). Answer B is incorrect since this claim was filed under state law, not a federal statute containing its own service provision.

Plaintiff and Defendant were married in State A, where both their children were also born. Thereafter, Plaintiff moved to State B where he was employed. After 15 years of marriage and accumulating significant marital assets totaling $10 million, the couple decided to divorce. Plaintiff filed for divorce in federal district court in State B, seeking joint custody of the children and a court order splitting the marital estate equally to each spouse. Defendant moved to dismiss the divorce action. How should the court rule on her motion? (A) The district court should deny the motion, because diversity jurisdiction is satisfied where Plaintiff and Defendant are now citizens of different states and the marital estate is worth $10,000,000. (B) The district court should remand the divorce action to a State B state court, because the federal court lacks federal subject matter jurisdiction. (C) The district court should transfer the divorce action to a State B probate and family court. (D) The district court will decline exercising jurisdiction over the action.

D is correct because even if diversity jurisdiction is present because federal courts will decline exercising such power when the lawsuit pertains to divorce and child custody. Generally, a federal court is required to exercise subject matter jurisdiction over a state law claim if the individual parties are citizens of different states and the amount in controversy is satisfied. However, the Supreme Court of the United States has recognized an exception in diversity actions for suits involving domestic relations. The exception covers divorce actions, alimony, and child custody cases, but does not include suits for torts. Here, Plaintiff seeks a divorce and child custody, and the district court should exercise its discretion and decline to exercise diversity jurisdiction even though diversity jurisdiction is present. A is incorrect because the Court has recognized the doctrine of abstention which allows a federal district court to decline to exercise federal jurisdiction in a diversity action that involves divorce, alimony, or child custody. Therefore, even if diversity jurisdiction is present, the court will grant the motion. B is incorrect because it incorrectly states that a federal court can remand an action when a party moves to dismiss the action for lack of subject matter jurisdiction. Remand is the process whereby the federal court transfers an action back to state court when defendant has improperly removed an action from state court to federal court. There was no such removal here as the case was originally filed in federal court. C is incorrect because transfer is permitted only between federal judicial districts, not federal to state as this answer choice suggests. While the district court will exercise its discretion and decline to exercise diversity jurisdiction in a suit that pertains to divorce, alimony, or child custody, the proper course would be to dismiss the action and allow the parties to file it in the appropriate state court. A federal court does not have power to transfer a federal lawsuit to a court in a state court system, even if it is the same state in which the federal court sits.

Plaintiff, an individual who lives and works in California, was visiting New York. He decided to take a taxi to go across town. While driving on Sixth Avenue, Taxi Driver collided with Defendant's truck. Defendant is an individual who lives and works in New Jersey and is employed by a trucking company that is incorporated and headquartered in New Jersey. Taxi Driver lives and works in New York. Plaintiff was seriously injured and eventually sued Taxi Driver, Defendant, and Defendant's employer in federal district court on the basis of diversity jurisdiction. Will Plaintiff be able to discover the existence of the three defendants' insurance coverage? (A) No, because the existence and extent of the defendants' insurance coverage is not relevant to the trial on the merits (B) No, because discovery of the insurance coverage could lead to an inflated verdict and would therefore be unfair to the defendants (C) Yes, but only the existence of the insurance coverage and not the monetary limits of the policy (D) Yes, because the facts of insurance coverage must be disclosed even without a discovery request

D is correct because the federal discovery rules make the disclosure of insurance agreements a subject of mandatory initial disclosure without a discovery request. Under the rule, every party must provide "for inspection and copying ... any insurance agreement under which an insurance business may be liable to satisfy all or part of a possible judgment in the action or to indemnify or reimburse for payments made to satisfy the judgment." FRCP 26(a)(1)(A)(iv). As with other mandatory initial disclosures, insurance information must be provided without a discovery request at or within 14 days after the required discovery meeting, see FRCP 26(f), unless otherwise ordered by the court or the parties stipulated to a different time. Keep in mind that the federal rules did not always explicitly authorize the disclosure of insurance information from an opposing party. In 1970 the federal rules were amended to allow discovery of the existence and substance of insurance agreements (in what was then numbered FRCP 26(b)(2)). In 1993 the federal rules again were amended, and now insurance agreements were made a topic of required disclosure; a party's failure to disclose insurance as mandated is, like other discovery violations, subject to sanction. See Wickens v. Shell Oil Co., 620 F.3d 747, 759 (7th Cir. 2010) (finding that a party "evaded" its responsibilities to disclose an "insurance company funding the litigation," and stating that "[t]he choice of a proper sanction for violations of the discovery rules ... lies in the discretion of the district court"). (A) is not correct because it misstates the rule of relevance and the current position taken by the federal rules on whether information about a party's insurance coverage is relevant. Whether information sought to be discovered is relevant within the meaning of the federal rules is determined with respect to the claims and defenses in the action. See FRCP 26(b)(1) ("Parties may obtain discovery regarding any non-privileged matter that is relevant to any party's claim or defense ... ."). Therefore, the statement in Choice (A) that insurance coverage is not "relevant to the trial on the merits" is not an accurate description of the governing rule. Moreover, as explained with respect to Choice (D), the federal rules now make the disclosure of insurance agreements a matter of required disclosure. See FRCP 26(a). (B) is not correct because the view that it expresses, while shared by some courts and commentators, is not the view reflected in the current federal rules. See FRCP 26(a). The 1970 amendment to the federal rules, permitting the discovery of insurance agreements, was intended to "enable counsel for both sides to make the same realistic appraisal of the case, so that settlement and litigation strategy are based on knowledge and not speculation"; at least in some cases, it was assumed that knowledge about insurance coverage would promote settlement and avoid "protracted litigation." Proposed Amendments to the Federal Rules of Civil Procedure Relating to Discovery, 48 F.R.D. 487, 499 (1969). However, keep in mind that apart from the disclosure of insurance agreements, the federal courts generally do not permit discovery of a defendant's financial condition. Although plaintiff may wish to know if defendant will be able to satisfy a judgment, the trend of decision is that such information is not relevant and therefore is not subject to discovery. See Sierrapine v. Refiner Prods. Mfg., Inc., 275 F.R.D. 604 (E.D. Cal. 2011). Rather, the appropriate way to obtain such information is post-judgment. See FRCP 69(a)(2). (C) is not correct because the rule that it states is not correct. The federal discovery rules mandate the production of information about the existence of insurance coverage as well as the monetary limits of the policy. See FRCP 26(a)(1)(A)(iv). However, it is recognized that the insurance application "may contain personal and financial information concerning the insured," and discovery of that information is beyond the scope of discovery of insurance agreements. See Proposed Amendments to the Federal Rules of Civil Procedure Relating to Discovery, 48 F.R.D. 487, 499 (1969). What is required, is the production "of any insurance policies that give rise to an insurer's obligation to indemnify or hold its insured harmless for a judgment," and generally does "not require the production of all agreements relating to insurance." Excelsior College v. Frye, 233 F.R.D. 583, 585 (S.D. Cal. 2006).

Plaintiff, from State M, sued his employer, Del Corp., a Delaware corporation with its principal place of business in State O, for wrongful termination. Plaintiff's attorney prepared the complaint on May 1st, but did not email a copy of the prepared complaint to Del Corp. until August 2nd. Plaintiff had his attorney file the complaint in State O state court seeking $100,000 in damages. Plaintiff then properly served Del Corp. on August 15th. On August 16th, Del Corp. removed the case to federal court. Plaintiff filed a motion to remand to state court. How should the court rule? (A) The court should deny the motion, because the defendant Del Corp. can remove the case at any time prior to trial. (B) The court should deny the motion, because the defendant Del Corp. can remove this case from state court to federal court because the diversity of citizenship and the amount in controversy make it a case that could have been brought in federal court. (C) The court should grant the motion, as Del Corp. must first file a motion, which the court must grant prior to removal. (D) The court should grant the motion, as Del Corp cannot remove this case.

D is correct. Although this case satisfies the requirements of diversity jurisdiction, removal will not be permitted due to the removal exception for diversity cases. The removal statute permits a defendant to remove a case from state court to federal court if the case could have originally been filed in federal court, unless the basis for removal is diversity jurisdiction, then the defendant cannot remove if the defendant is from the jurisdiction where the action is pending. Plaintiff is from State M and Defendant is from Delaware and State O. Damages are reasonably likely to exceed $75,000. Thus, federal jurisdiction would be satisfied through diversity. However, Plaintiff filed his state law claims in State O state court. Defendant Del Corp. has its principal place of business in State O, thus precluding Del Corp. from removing the case to State O federal court. Since there is no federal question or constitutional law violation (Plaintiff has not pled any), there is no other basis for federal subject matter jurisdiction. Therefore, D is correct. A is incorrect. The defendant cannot remove the case at any time prior to trial. The removal statute requires removal within 30 days after the defendant is served with process. B is incorrect. Although Plaintiff could have filed the case in federal court, because he chose State O state court, Del Corp. cannot remove it. C is incorrect. Where removal is proper, the case is automatically removed upon notice and there is no requirement that the court approve or deny removal. If removal is improper, it is upon the plaintiff to file a motion to remand the case to state court.

Plaintiff, a citizen of State X, filed suit against her neighbor, also a citizen of State X, for damages to her new car resulting from an accident on their shared driveway. The neighbor had also caused significant damage to Plaintiff's fence, garage, and prized rose bushes. After failed attempts to settle their dispute, Plaintiff filed suit in federal district court in State X alleging damages against the neighbor exceeding $75,000. The neighbor filed a motion to dismiss. How should the court rule? (A) The court should deny the neighbor's motion, because the federal district court has exclusive jurisdiction over claims that exceed $75,000. (B) The court should grant the motion, because Plaintiff cannot aggregate the damages to the car, fence, garage, and rose bushes in order to meet the jurisdictional threshold for diversity jurisdiction. (C) The court should deny the motion, because the plaintiff is the master of her complaint and can choose which court she wishes to hear her case. (D) The court should grant the motion, because Plaintiff has not met the requirements for diversity jurisdiction.

D is correct. Federal courts have subject matter jurisdiction over state law claims where the adverse parties are diverse in citizenship (meaning no plaintiff is from the same state as any defendant) and the amount in controversy is reasonably likely to exceed $75,000. Here, Plaintiff has a state law claim of negligence against her neighbor. In order to assert proper federal jurisdiction, Plaintiff would have to satisfy both prongs. While the damages are reasonably likely to exceed $75,000, the parties are neighbors, meaning that they are citizen of the same state. Therefore, complete diversity of citizenship is lacking. A is incorrect because there is no requirement that claims exceeding $75,000 be adjudicated in federal court. Federal courts have exclusive jurisdiction over claims arising under certain federal statutes such as antitrust claims, patent violations, bankruptcy claims, etc. B is incorrect because aggregation of damages resulting from the same cause of action is permitted. Federal courts have subject matter jurisdiction over state law claims where the adverse parties are diverse in citizenship (meaning no plaintiff is from the same state as any defendant) and the controversy is reasonably likely to exceed $75,000. Here, Plaintiff has a state law claim of negligence against her neighbor where Defendant's conduct caused damages to Plaintiff's property. Plaintiff is permitted, (and in fact required to or else waives her rights), to assert all damages and claims against Defendant in a single action. C is incorrect because, although the plaintiff is the master of the complaint and can in fact choose which court to file the claim, the chosen court must have proper jurisdiction over the claim in order to render a valid judgment. Here, Plaintiff chose to file her claim in federal court. In order for the court to have power to adjudicate the claim, it must fall under a claim of federal/constitutional law violation or a state law claim under diversity jurisdiction. As this is a state law claim that does not meet diversity jurisdiction requirements, the court must dismiss for lack of subject matter jurisdiction.

Plaintiffs, a husband and wife, were travelling in their home state of A when Defendant, from State Z, crashed into Plaintiffs' pick-up truck, causing the truck to spin out of control and into oncoming traffic. Both plaintiffs suffered significant personal injuries, requiring long-term hospital stays and rehabilitation. A year after the accident, Plaintiffs filed suit against Defendant in federal district court in State Z based on diversity jurisdiction, each plaintiff alleging damages exceeding $250,000. A week later, Plaintiffs decided to permanently move to State Z — they sold their home in State A and purchased a new home in State Z. Plaintiffs also both sought employment in State Z. During discovery, Defendant learned of Plaintiffs' move and filed a motion to dismiss for lack of subject matter jurisdiction. How should the court rule? (A) The court should grant the motion, because by moving to State Z, Plaintiffs destroyed diversity of citizenship. (B) The court should grant the motion, because Plaintiffs cannot prove with certainty that their damages are $250,000 each. (C) The court should deny the motion, because the parties have already conducted discovery and dismissing the case at this stage would cause undue hardship to Plaintiffs. (D) The court should deny the motion, because the court has proper subject matter jurisdiction over this claim.

D is correct. Federal courts have subject matter jurisdiction over state law claims where the adverse parties are diverse in citizenship (meaning no plaintiff is from the same state as any defendant) and the controversy is reasonably likely to exceed $75,000. Individuals are citizens in the state where they are domiciled, meaning where their true, fixed, and permanent place of residence is located. For purposes of diversity jurisdiction, citizenship is determined at the time of filing and diversity of citizenship need not continue after filing. Here, at the time of filing, Plaintiffs were citizens of State A and Defendant was a citizen of State Z, thus complete diversity existed. The fact that Plaintiffs moved a week after filing the suit does not affect the court's jurisdiction, as diversity need not continue after the suit is filed. A is incorrect. As noted above, for purposes of diversity jurisdiction, citizenship is determined at the time of filing and diversity of citizenship need not continue after filing. Here, at the time of filing, Plaintiffs were citizens of State A and Defendant was a citizen of State Z, thus complete diversity existed. The fact that Plaintiffs moved a week after filing the suit does not affect the court's jurisdiction, as diversity need not continue after the suit is filed. B is incorrect. For diversity jurisdiction, plaintiffs only need a good faith basis for alleging damages that are reasonably likely to exceed $75,000. Considering the extent of their injuries and medical expenses, Plaintiffs have met their burden of alleging in good faith that their damages will meet the amount-in-controversy requirement. C is incorrect. Lack of subject matter jurisdiction cannot be waived. One of the fundamental principles in civil procedure is the court's power over the controversy. Subject matter jurisdiction, the court's power over the controversy, can never be waived. The lack of the court's subject matter jurisdiction can be asserted at any stage of the litigation, even for the first time on appeal. See FRCP 12(h).

Apex Corp. sued Bemis Corp. for breach of contract under federal diversity jurisdiction in the Southern District of Illinois. In response to the complaint, Bemis filed a timely Rule 12(b)(3) motion to dismiss for improper venue. The court denied the motion. Bemis then filed its answer, raising two defenses: (1) lack of personal jurisdiction under Rule 12(b)(2); and (2) failure to state a claim upon which relief can be granted under Rule 12(b)(6). Apex filed a Rule 12(f) motion to strike both defenses. How should the court rule on Apex's motion? (A) The court should deny Apex's motion to strike both defenses. (B) The court should deny Apex's motion to strike the personal jurisdiction defense but grant Apex's motion to strike the failure to state a claim defense. (C) The court should grant Apex's motion to strike both defenses. (D) The court should grant Apex's motion to strike the personal jurisdiction defense but deny Apex's motion to strike the failure to state a claim defense.

D is correct. Personal jurisdiction is a waivable defense that must be raised in the defendant's first response. Rules 12(g)(2), 12(h)(1). In contrast, failure to state a claim can be raised later: in an answer, a 12(c) motion for judgment on the pleadings, or at trial. Rule 12(h)(2). Here, Bemis' first response was a Rule 12(b)(3) motion challenging venue. The personal jurisdiction defense should have been raised in the same motion because it was available at the time. By omitting that defense, Bemis waived it and cannot belatedly raise it in the answer. A is incorrect because it is proper to strike an "insufficient defense" under Rule 12(f)—here the personal jurisdiction defense is insufficient because it has been waived. B is incorrect because it has matters backwards. The failure to state a claim defense can be raised later in the litigation, while the personal jurisdiction defense must be in the defendant's first response. C is incorrect because the failure to state a claim defense can be raised for the first time in an answer even if the defendant has filed a Rule 12(b) motion on other grounds.

Plaintiffs, both citizens of State N, filed an action in federal court in Delaware against Corp. Inc., a Delaware corporation with its principal place of business in State N, alleging that the prospectus issued by Corp. Inc. in connection with the sale of its stock contained misrepresentations in violation of both federal securities law and Delaware corporate law. In connection with that claim, Plaintiffs each sought $250,000 in damages. Corp. Inc. filed a $3 million counterclaim against Plaintiffs under State N law for malicious prosecution. Plaintiffs filed a motion to dismiss the counterclaim for lack of jurisdiction. How should the court rule? (A) The court should grant the motion, because the parties are not diverse. (B) The court should deny the motion, because Corp. Inc. is seeking damages in excess of $75,000. (C) The court should grant the motion, because Corp. Inc.'s claim is unrelated to the plaintiffs' claims against Corp. Inc. (D) The court should deny the motion, because Corp. Inc.'s claim falls within the court's supplemental jurisdiction.

D is correct. The claim against Plaintiffs is a compulsory counterclaim. It arises under state law and the adverse parties are both citizens of State N. Thus, there is no original jurisdiction under either federal question (§1331) or diversity jurisdiction (§1332). But under §1367A, supplemental jurisdiction is available over "all other claims" that form part of the "same case" as a claim that falls within that court's original jurisdiction. Since the counterclaim forms part of the same case as the plaintiffs' claim as a compulsory counterclaim (federal securities claims against Corp. Inc.), it falls within the court's original jurisdiction. A is incorrect. Since the counterclaim falls within the court's supplemental jurisdiction and none of the discretionary factors listed in §1367C that would justify declining to exercise supplemental jurisdiction exist, supplemental jurisdiction is available. Thus, the lack of diversity is not fatal to the problem. B is incorrect because this fact is irrelevant since jurisdiction over this non-diverse state law claim is not and cannot be predicated upon diversity under §1332. C is incorrect because the two claims do form part of the same case and thus fall under the court's supplemental jurisdiction.

Plaintiffs, both citizens of State N, filed an action in federal court in Delaware against Corp. Inc., a Delaware corporation with its principal place of business in State N, alleging that the prospectus issued by Corp. Inc. in connection with the sale of its stock contained misrepresentations in violation of both federal securities law and Delaware corporate law. In connection with that claim, Plaintiffs each sought $250,000 in damages. In the same lawsuit, Plaintiffs also asserted a $300,000 claim against Stock Co., a stock brokerage firm, incorporated in State N, alleging that Stock Co. had distributed these prospectuses in full knowledge of the existence of these material misrepresentations in violation of Delaware law. Stock Co. in turn filed a $500,000 breach of contract claim against Corp. Inc. alleging that Corp. Inc. failed to pay its commission earned by selling shares of Corp. Inc. Corp. Inc. filed a motion to dismiss the breach of contract claim based on lack of subject matter jurisdiction. Stock Co. opposed the motion arguing that the federal court has jurisdiction over the breach of contract claim. How should the court rule on the motion? (A) The court should deny the motion, because the claim is reasonably likely to exceed $75,000. (B) The court should grant the motion, because the parties are not diverse. (C) The court should deny the motion, because it falls within the court's supplemental jurisdiction. (D) The court should grant the motion, because the claim is unrelated to the plaintiffs' claims against Corp. Inc.

D is correct. The claim is a cross-claim brought by one defendant against a co-defendant. It is a state law claim brought by one N citizen against another N citizen. Therefore there is no original jurisdiction over this claim under either federal question (§1331) or diversity (§1332). A cross-claim, such as the one at issue here, may fall under the court's supplemental jurisdiction (§1367), provided it forms part of the same case as the claim that falls within the court's original jurisdiction, (here, Plaintiffs' federal securities claims against Corp. Inc.). This breach of contract claim from Stock Co. against Corp. Inc. is not part of the same case. Thus, it does not fall under the court's supplemental jurisdiction. C is incorrect. This breach of contract claim arises out of Corp. Inc.'s failure to pay Stock Co. a royalty from the amount of shares that the firm sold to the public. These two claims do not arise out of a common nucleus of operative fact and, therefore, supplemental jurisdiction over this claim is unavailable. A is incorrect because the claim is a non-diverse state law claim and so the fact that the claim meets the amount in controversy requirement is irrelevant. B is incorrect because the absence of diversity is not jurisdictionally fatal since the claim could still fall within the court's supplemental jurisdiction under §1367A.

Company is the manufacturer of off-road recreational vehicles. Driver owns a vehicle manufactured by Company. Driver was injured while driving the vehicle on the highway. Driver claims that the vehicle accelerated when it was supposed to brake, and as a result it rolled into a ditch. A State Trooper was called and arrived at the scene of the accident. Plaintiff has informed Company that he intends to file a federal action, based on diversity jurisdiction, for injuries caused by the car's alleged product-design defects. Company wants to find out what the State Trooper knows about the accident and whether Driver behaved in any erratic or unusual way, and has filed a verified petition with the federal district court in the district in which the accident occurred to deport the State Trooper. Does the Company have a right to depose the State Trooper? (A) No, a party may not take discovery from a nonparty because the commencement of a federal lawsuit (B) No, a party may take discovery before a suit is commenced but only if the request is accompanied by a draft complaint against the party from whom discovery is sought (C) Yes, a party may take discovery of material that is relevant to claims and defenses (D) No, a party may not take discovery before the commencement of a federal lawsuit absent a special need to preserve testimony that is not present on these facts

D is correct. The federal discovery rules allow a party to petition a district court for an order to perpetuate testimony before a lawsuit has commenced. See FRCP 27. A party seeking pre-complaint discovery must file a verified petition with the district court which shows: "(A) that the petitioner expects to be a party to an action cognizable in a United States court, but cannot presently bring it or cause it to be brought; (B) the subject matter of the expected action and the petitioner's interest; (C) the facts that the petitioner wants to establish by the proposed testimony and the reasons to perpetuate it; (D) the names or a description of the persons whom the petitioner expects to be adverse parties and their addresses, so far as known; and (E) the name, address, and expected substance of the testimony of each deponent." FRCP 27(a)(1). Pre-complaint testimony "cannot be used to discover evidence for the purpose of filing the complaint." In re Allegretti, 229 F.R.D. 93, 96 (S.D.N.Y. 2005). The consensus reading of FRCP 27 is that the rule "should be used 'in special circumstances to preserve testimony which could otherwise be lost.'" In Re Yamaha Corp., U.S.A., 251 F.R.D. 97, 99 (2008) (quoting Ash v. Cort, 512 F.2d 909 (3d Cir. 1975)). The facts here indicate that Company would be unable to show that State Trooper's testimony would be lost, concealed, or destroyed, and thus there is no justification to grant the motion for the pre-complaint deposition. Moreover, Company cannot specify "facts that [it] wants to establish by the proposed testimony"; it simply wishes to learn more about the accident. Further, Company has no "reason to perpetuate [the testimony]." Company does not allege that State Trooper is ill or planning soon to leave the country, or that he plans to destroy or conceal evidence. See id. at 96. There is no reason to believe that State Trooper will not be able to give any testimony if Driver does file a federal action against Company. Be aware that in a few states, pre-litigation discovery is more generous than under the federal discovery rules. See, e.g., Ohio Civ. R. 34(D)(1) (pre-filing discovery permitted by "a person who claims to have a potential cause of action"). (A) is not correct because it mistakenly applies an incorrect rule. As discussed in the explanation for Choice (D) above, the Federal Rules of Civil Procedure explicitly allow for limited discovery before commencement of a suit, as long as the party seeking discovery can show that it expects a suit to be brought, but temporarily cannot and the testimony it seeks is at risk of being lost or destroyed. See FRCP 27. Pre-litigation discovery is so limited in order to avoid exploitation of the judicial system by, for example, firms seeking information from their competitors, with no intention of pursuing litigation. See Intel Corp. v. Advanced Micro Devices, Inc., 542 U.S. 241, 270 (2004). Thus, though Company's request should be denied, it is not for this reason, but rather the reason given in Answer (D). (B) is not correct because it misstates the requirements of the relevant federal rule. A party seeking pre-complaint discovery must file a verified petition with the district court as explained in Answer (A). Though pre-complaint testimony "cannot be used to discover evidence for the purpose of filing the complaint," In re Allegretti, 229 F.R.D. 93, 96 (S.D.N.Y. 2005), the federal rule does not require that a complaint be drafted, only that the petitioner expects to be a party in a lawsuit. Thus, though Company's request should be denied, it is not for this reason, but rather the reason given in Answer (D). (C) is not correct because the rule that it recites, although true as far as it goes, does not control the result on these facts. The federal discovery rules define what is discoverable, who may discover information, and when information may (or must) be discovered. Even if the deposition of the State Trooper will lead to information that is relevant to the claims and defenses in an anticipated law suit, see FRCP 26(b), the rules seriously curtail any discovery prior to the commencement of a litigation. Importantly, pre-litigation discovery "cannot be used to discover evidence" for use in the pleadings. See In re Allegretti, 229 F.R.D. 93, 96 (S.D.N.Y. 2005). While the pre-complaint deposition of State Trooper might generate information relevant to Company's defenses, this is not an acceptable reason to grant a petition for pre-litigation discovery under FRCP 27

The plaintiff, a citizen of State A, files a claim in federal district court alleging that the defendant, a Delaware corporation, violated her rights under the federal Civil Rights Act of 1964 and seeks damages in the amount of $50,000. Does the court have subject matter jurisdiction? (A) No, because the plaintiff is seeking only $50,000. (B) No, because the claim does not arise under federal law. (C) Yes, because the parties are diverse. (D) Yes, because the claim arises under federal law.

D is correct. The plaintiff's case arises under the federal statute, giving the federal court subject matter jurisdiction regardless of the amount in controversy. Unlike diversity cases, there is no jurisdictional amount in controversy requirement for federal question cases under §1331. A is incorrect. Unlike diversity cases, there is no jurisdictional amount in controversy requirement for federal question cases under §1331. The plaintiff only needs to allege some damages. C is incorrect. The fact that the parties are diverse does not, itself, provide a basis for jurisdiction. Section 1332 also requires an amount in controversy in excess of $75,000, which is not present here. B is incorrect. The case clearly arises under federal law because the plaintiff's cause of action arises under the Civil Rights Act of 1964, which is a federal law.

Plaintiff, a citizen from State N, brought a patent infringement claim against Avant Co., a Delaware corporation, in federal court in State N. Plaintiff alleged that Avant Co. had infringed her patent in violation of the federal patent statute and sought damages in the amount of $50,000. Plaintiff also asserted a $250,000 intentional infliction of emotional distress claim against the president of Avant, James Avant, a citizen of State N. In this claim, Plaintiff alleges that James Avant had subjected her to repeated series of acts of sexual harassment and that this caused Plaintiff to suffer severe emotional distress. Defendant James Avant filed a motion to dismiss challenging the court's exercise of jurisdiction over the supplemental claim. How should the court rule? (A) The court should deny the motion, because the federal court can exercise jurisdiction over the supplemental state law claim. (B) The court should grant the motion, because courts do not permit pendent party jurisdiction. (C) The court should deny the motion, because the value of the two claims can be aggregated. (D) The court should grant the motion, because the two claims do not form part of the same case.

D is correct. Under §1367A if the plaintiff asserts a federal question claim against one defendant, the court can exercise supplemental jurisdiction over a non-diverse state law claim against a separate defendant if the two claims form part of the same case, i.e., arise out of a common nucleus of operative fact, and if none of the factors listed in §1367C justify declining to exercise such supplemental jurisdiction. Here, the patent claim against the corporation is a federal question claim. The tort claim against the president, however, does not form part of the same case as the patent claim since it is based on acts of sexual harassment and the former claim arose out of the company's acts of infringement. Thus, supplemental jurisdiction under §1367A is not available, which makes A incorrect. B is incorrect because §1367A does permit pendent party jurisdiction in cases involving a federal question claim. Pendent party jurisdiction refers to the court's power to adjudicate a claim against a party who would not otherwise be subject to the court's jurisdiction but the claim against the party arises from the same set of operative facts as the original claim. Here, the federal court would not have diversity jurisdiction over the emotional distress claim against James Avant, as both Plaintiff and Defendant are from State N. Also, since the emotional distress claim does not arise from the same set of operative facts as the original claim under federal jurisdiction, the federal court will not exercise supplemental jurisdiction over the emotional distress claim against Avant. C is incorrect because it is irrelevant and wrong as a matter of law. The plaintiff and defendant James are not diverse and so §1332, and therefore its amount in controversy requirement, is irrelevant to this question. Moreover, a plaintiff is not permitted to aggregate the value of claims against multiple defendants in a diversity-based case.

Jake, a lifetime resident of Boston, made his first ever venture out of Massachusetts when he flew to San Francisco, California. He was unable to get a nonstop flight and so had to change planes at John F. Kennedy International Airport in New York City, New York. While sitting in a coffee shop at Kennedy airport, he was served with process in connection with a lawsuit filed against him by his former next-door neighbor. His neighbor, now a citizen of New York, brought suit in New York claiming that Jake had sold him a lemon—Jake's 2005 automobile. Jake files a motion to dismiss for lack of personal jurisdiction. How should the court rule? (A) The court should grant the motion because the car sale was consummated entirely in Massachusetts. (B) The court should grant the motion because Jake is a citizen of Massachusetts. (C) The court should deny the motion because the plaintiff is a citizen of New York. (D) The court should deny the motion because Jake was served in New York.

D is the correct answer. A court has jurisdiction over people and property within their jurisdictional borders. If a defendant is present in the jurisdiction when served with process, however briefly, that defendant will be subject to the court's personal jurisdiction over him. This is called "tag jurisdiction" and has been ruled constitutionally sufficient by the United States Supreme Court. Answer A is incorrect because personal jurisdiction concerns the court's power over the defendant. More than one state may exert personal jurisdiction over a defendant. Here, Massachusetts would likely have personal jurisdiction over the parties because Jake is a lifelong resident of Massachusetts (hence, general jurisdiction) and the plaintiff had minimum contacts with Massachusetts because the transaction was in Massachusetts. However, another state may have jurisdiction over Jake because of his contacts with that state. Physical presence serves as minimum contacts under "tag jurisdiction." B is incorrect because a defendant can be subject to personal jurisdiction in a state other than the state of citizenship, as is this case with "tag jurisdiction." C is incorrect because personal jurisdiction deals with the court's power over the defendant; the plaintiff's citizenship is irrelevant to determining the constitutionality of exercising personal jurisdiction over the defendant.

Plaintiffs were injured in a bus accident outside of Capital City, Foreign Country while on vacation. The injury occurred when two of the bus's tires exploded en route to their destination causing the bus to veer off the marked road and land in a ditch. Plaintiffs, from State A, filed suit in federal district court in State A under diversity jurisdiction. Defendants were Tire Inc. USA (incorporated and principal place of business in State Z) and Tire Inc.'s foreign subsidiary, Spoke Co. (organized and operating in Foreign Country). The tires used on the bus were designed and manufactured in Faraway Land and were slightly different in design and size from tires sold in the U.S. Spoke Co. is not registered to do business in State A and has not sold any tires to State A customers. However, Tire Inc. affiliates have distributed a type of Spoke Co. tires different than those involved in the accident to State A customers. Defendant Spoke Co. filed a motion to dismiss for lack of personal jurisdiction. How should the court rule? (A) The court should deny the motion because Spoke Co. tires have been distributed to State A customers, which establishes sufficient contacts. (B) The court should deny the motion because Spoke Co. is a subsidiary of a U.S. company, Tire Inc., which is subject to personal jurisdiction in State A. (C) The court should grant the motion because Spoke Co. is a foreign company organized and operating outside the U.S. (D) The court should grant the motion because Spoke Co. does not have sufficient contacts in State A.

D is the correct answer. A foreign subsidiary of a U.S. corporation is not subject to general jurisdiction within a forum unless it is considered "at home" in that forum (generally, where an entity is incorporated and where it has its principal place of business). See Goodyear Dunlop Tires Operations, S.A., et al. v. Brown, 564 U.S. 915 (2011), DaimlerAG v. Bauman, 571 U.S. 117 (2014). Answer A is incorrect because the Spoke Co. tires that ended up in State A were not those involved in the accident or even of the same design. Therefore, Spoke Co.'s limited activities in State A (such as they are) do not relate to or give rise to the underlying cause of action. This means that specific personal jurisdiction is not available and only general personal jurisdiction could be established. As discussed above, corporations are only subject to general personal jurisdiction where they are considered "at home." In no way could Spoke Co. be considered at home in State A. Further, it is unlikely that even specific personal jurisdiction would be satisfied, because Spoke Co. does not appear to direct activities into State A based on the facts presented. See J. McIntyre Machinery, Ltd. v. Nicastro, 564 U.S. 873 (2011) Answer B is incorrect as a subsidiary of a corporation is a separate legal entity and the activities of its parent within a forum generally cannot be attributed to it. See Daimler. Answer C is incorrect because it does not take into consideration that a foreign company can be subject to specific personal jurisdiction in the U.S. if the company's activities in the forum state satisfy the requirements for continuous and systematic activity, such that exercising jurisdiction over the foreign defendant is consistent with due process.

Plaintiff from State A and Defendant from State B were involved in a multi-vehicle crash on a busy State B highway. After all necessary medical treatments had been completed, Plaintiff filed a lawsuit in federal district court in State B based on diversity jurisdiction. Unbeknownst to the Plaintiff, after the accident but prior to the filing of the lawsuit, Defendant moved to State A. After the suit was filed, Defendant filed a motion to dismiss for lack of personal jurisdiction. Two days after the trial began, the Defendant filed another motion to dismiss for lack of subject matter jurisdiction and improper venue. How should the court rule on each motion to dismiss? (A) The court should deny all motions. (B) The court should deny the motion to dismiss for lack of subject matter jurisdiction, but grant the others. (C) The court should grant all the motions. (D) The court should grant the motion to dismiss for lack of subject matter jurisdiction and deny the rest.

D is the correct answer. Defendant's motion to dismiss for lack of personal jurisdiction should be denied because, while domiciled in State A, the cause of action arose from Defendant's contacts in State B, granting State B specific jurisdiction over Defendant. The subsequent motion on improper venue was waived as Defendant did not raise it in his first defensive move (the first motion to dismiss) and thus should be denied. See FRCP 12(g)-(h). As to the motion to dismiss for lack of subject matter jurisdiction, under FRCP 12(h)(3), the lack of subject matter jurisdiction cannot be waived and can be asserted at any time. When assessing whether a court has subject matter jurisdiction based on diversity jurisdiction, the relevant time for citizenship is at the time of filing. Here, the Plaintiff was, at all relevant times, domiciled in State A. Defendant was from State B at the time the accident took place but moved to State A before suit was filed. Therefore, the court should deny the lack of personal jurisdiction and improper venue, and grant the motion to dismiss for lack of subject matter jurisdiction.

Dan, a lifelong resident of New York, went to New Orleans for a convention. One night, he tasted a local Cajun whiskey. Liking what he tasted, he bought a case of the whiskey to take back to New York. Upon his return to New York, Dan gave a bottle to his boss, Ben, a citizen of New York. After work that evening, Ben decided to try the whiskey and prepared himself a cocktail consisting of the Cajun whiskey and water. After three or four sips of his cocktail, Ben experienced a severe burning sensation in his throat and stomach. He called his doctor, who advised him to come to the hospital and bring the bottle of the whiskey with him. At the hospital it was determined that the bottle contained a high percentage of acid. Ben was treated accordingly. He survived, but had to have part of his stomach removed and will talk in a low raspy voice for the rest of his life. Ben comes to you, an attorney in New York, and wants you to represent him in his personal injury action. He wants to sue for $1 million to pay for his medical expenses and be compensated for his pain and suffering and permanent physical impairments. You agree to represent him and immediately begin making certain investigations. You learn that the Cajun whiskey is a product distilled by the De-Lis Whiskey Company, a Louisiana corporation with its principal place of business in Louisiana. It distributes its products in Louisiana, Alabama, and Mississippi. You learn that about 45% of all sales of the Cajun whiskey are made to New York tourists who take the product back to their home state and 50% of its sales are made to New Yorkers who purchase the liquor through De-Lis's highly interactive web site. Sales to New Yorkers account for in excess of $3 million annually. In addition, you learn that De-Lis Whiskey Company has $500,000 on deposit in a New York bank. Assume that the legislature in every state in the country has passed the following statute: The courts of this state shall have personal jurisdiction over an individual, corporation or other entity who, in person or through an agent: transacts business within the state; or commits a tortious act without the state causing injury within the state; or is personally served within the state; or owns property within the state. If the plaintiff chose to file this action in a federal district court in New York, which long-arm statute would apply? (A) Louisiana, because the defendant is a citizen of that state. (B) New York, because the plaintiff is a citizen of that state. (C) Louisiana, because that is where the tortious act occurred. (D) New York, because it is the forum state.

D is the correct answer. Federal Rule of Civil Procedure 4(k)(1) provides that in the absence of a governing federal statute, the federal court applies the state long-arm statute of the forum state. Since this is a tort claim governed by state law, there is no applicable federal long-arm provision and so the forum's long-arm statute applies. No other factor is relevant, which is why the other answers are incorrect.

Kane Corp., a U.S. company headquartered in Nebraska and incorporated in Delaware, manufactures and markets a variety of products made from high-tech fibers. Among these products is a surgical gown for doctors that Kane advertises as providing the "highest level of protection from infectious diseases." For many years, Kane has sold large quantities of these gowns directly to four different states' bio-containment centers, which treat patients with infectious diseases. Texas is one of those states. Kane sources all of its fabric for these gowns from a supplier in Singapore called Singtel. Singtel supplies many other companies with its fabric in other products and does not keep track of where all of its materials end up. A group of Texas bio-containment center doctors who wear Kane's gowns as a result of Kane's direct sales to the state became infected with a dangerous virus. They brought a product liability suit against Kane and Singtel in Texas state court. Which choice below states most accurately how personal jurisdiction would be analyzed for both defendants? (A) Kane would be subject to personal jurisdiction in Texas because of its direct sales there, and Singtel would be subject to personal jurisdiction in Texas because it profits from those sales. (B) Kane would not be subject to personal jurisdiction in Texas because it sells gowns to three other states besides Texas, but Singtel would be subject to personal jurisdiction in Texas because it blankets the world with its fabric products. (C) Neither Kane nor Singtel would be subject to personal jurisdiction in Texas, because their gowns and fabric are not related to the doctors' injuries in Texas. (D) Kane would be subject to personal jurisdiction in Texas, because of its direct sales there, but Texas' personal jurisdiction over Singtel would be uncertain.

D is the correct answer. Kane's direct sales of gowns to the Texas bio-containment center is a classic example of purposeful availment of the benefits of the forum. See International Shoe Co. v. Washington; Worldwide Volkswagen Corp. v. Woodson. Because it intentionally sells a large volume of gowns consistently to the forum state and makes a product that meets the unique needs of a customer in that state, Kane cannot claim to be surprised by a Texas suit based on a defect in the gowns it sold there. Jurisdiction over Singtel must be analyzed under one of the stream of commerce approaches in J. McIntyre Machinery v. Nicastro. That's because Singtel does not sell directly to Texas but gets its products to the state through a middleman, Kane. The Justices' approaches in Nicastro, none of which garnered a majority, might reach different results, making personal jurisdiction over Singtel uncertain. The Kennedy plurality in Nicastro would not find jurisdiction because Singtel does nothing affirmatively to try to target customers in Texas. The Ginsburg opinion may or may not find personal jurisdiction; it would depend on Singtel's knowledge and expectations after delivering its fabric into the stream of commerce. A is incorrect because the Supreme Court in Worldwide Volkswagen rejected the idea that profiting constitutes purposeful availment for personal jurisdiction purposes. B is incorrect because no group of Justices on the Supreme Court has ever said that indiscriminate blanketing across the world can constitute purposeful availment of the benefits and privileges of a particular state. C is incorrect because under the Supreme Court's decision in Bristol-Myers Squibb v. Superior Court of California and the question facts, the injuries to the doctors in Texas are clearly related to the defendants' Texas sales. The jurisdictional problem raised by these facts is purposeful availment, not relatedness.

A large group of in-state and out-of-state residents opted in a class action lawsuit filed against Pharma Co. in state court in State A, alleging one of its products caused severe internal organ damage. Pharma Co. is incorporated in State A and has its principal place of business in State B. The named representative of the class was a citizen of State B; the class consisted of 5,000 members from State A, State B and State C. The class was seeking damages in excess of $5,000,000. Shortly after being served with the complaint, Pharma Co. removed the case from state court in State A to the federal district court in State A. Fifteen days later, the class representative filed a motion to remand. How should the court rule on the plaintiff's motion? (A) The court should grant the motion because Pharma Co. is a citizen of the state where the action is pending. (B) The court should grant the motion because the State A state court has an interest in adjudicating these claims. (C) The court should deny the motion because the federal district court in State A is the more appropriate forum to adjudicate these claims. (D) The court should deny the motion because Pharma Co. had a right to remove this case to federal court.

D is the correct answer. Removal jurisdiction under 28 U.S.C. 1441 allows a defendant to remove a case from state court to federal court in the judicial district where the action is pending if the case could have originally been brought in federal court unless the basis for removal is diversity jurisdiction and the defendant is from the state where the action is pending. However, pursuant to 28 U.S.C. 1453(b), a class action may be removed to a district court in the district where the action is pending without regard to whether any defendant is a citizen of the state where the action is brought. Here, the class action was filed in state court in State A and Pharma Co. is a citizen of State A (incorporation). Since this is a class action, 1453(b) is triggered and thus Pharma Co. may remove this class action even though it is from State A. Therefore, D is correct and Answer A is incorrect. Answer B is incorrect because, while this statement may be true, a defendant has a statutory right to remove the case from state court to federal court, provided the federal court has subject matter jurisdiction, which it does here under 28 U.S.C. 1332(d)(2). Answer C is incorrect because, while this statement may be true, a defendant has a statutory right to remove the case from state court to federal court, provided the federal court has subject matter jurisdiction, which it does here under 28 U.S.C. 1332(d)(2).

Sam is injured in a crash between his car and another car and a truck on a narrow country road in Concord, New Hampshire. Sam is a citizen of New Hampshire. The driver of the other car, Karen, is a citizen of Maine, and the owner of the truck, Mac Truck, is incorporated under the laws of Delaware and has its principal place of business in Connecticut. Sam files an action in the federal court for the District of New Hampshire seeking damages of $250,000 against each of the two named defendants, Karen and Mac Truck. In the same suit, Karen files a claim against Mac Truck seeking over $75,000 in damages for the damage to her car. Mac Truck files a tort claim against Sam seeking over $70,000 in damages for the damages to its truck. Mac Truck also files a claim against Ben, a citizen of Connecticut who was driving its truck at the time of the accident, seeking indemnity from any loss it might suffer in connection with Sam's claim against it. Does venue lie over Sam's claim against Karen? (A) Yes, because the parties are diverse. (B) Yes, because Sam resides in New Hampshire. (C) No, because none of the defendants reside in New Hampshire. (D) Yes, because the accident occurred in New Hampshire.

D is the correct answer. Since original jurisdiction was founded solely on diversity in this civil action, the relevant venue provision is §1391(b). Under §1391(b)(2), venue will lie in the district where a substantial part of the events that gave rise to the claim occurred. Since the accident occurred in the chosen venue, venue lies there. Answer A is incorrect because that fact goes to subject matter jurisdiction and not to venue. Answer B is incorrect because the residence of the plaintiff is irrelevant to the venue calculation required by §1391(b). Answer C is incorrect because although it is true, venue will still lie where the underlying events occurred pursuant to §1391(b)(2).

Dan, a lifelong resident of New York, went to New Orleans, Louisiana, for a convention. One night, he tasted a local Cajun whiskey. Liking what he tasted, he bought a case of the whiskey to take back to New York. Upon his return to New York, Dan gave a bottle to his boss, Ben, a citizen of New York. After work that evening, Ben decided to try the whiskey and prepared himself a cocktail consisting of the Cajun whiskey and water. After three or four sips of his cocktail, Ben experienced a severe burning sensation in his throat and stomach. He called his doctor, who advised him to come to the hospital and bring the bottle of the whiskey with him. At the hospital it was determined that the bottle contained a high percentage of acid. Ben was treated accordingly. He survived, but had to have part of his stomach removed and will talk in a low raspy voice for the rest of his life. Ben comes to you, an attorney in New York, and wants you to represent him in his personal injury action. He wants to sue for $1 million to pay for his medical expenses and be compensated for his pain and suffering and permanent physical impairments. You agree to represent him and immediately begin making certain investigations. You learn that the Cajun whiskey is a product distilled by the De-Lis Whiskey Company, a Louisiana corporation with its principal place of business in Louisiana. It distributes its products in Louisiana, Alabama, and Mississippi. Assume that the legislature in every state in the country has passed the following statute: The courts of this state shall have personal jurisdiction over an individual, corporation or other entity who, in person or through an agent: transacts business within the state; or commits a tortious act without the state causing injury within the state; or is personally served within the state; or owns property within the state. Can a state court in Louisiana exercise general jurisdiction over De-Lis Whiskey Company? (A) No, because it does not have a bank account in Louisiana. (B) Yes, because it transacts business in Louisiana. (C) No, because it did not commit a tortious act in Louisiana. (D) Yes, because it is a citizen of Louisiana.

D is the correct answer. State courts have personal jurisdiction over their citizens, even if they are not residing in the forum state at the time suit is brought. There is no need to examine the application of the long-arm statute since these statutes deal with obtaining personal jurisdiction over noncitizens and nonresidents. There is also no constitutional objection to exercising personal jurisdiction over a forum citizen. Consequently, all the other answers are incorrect. A is incorrect because the mere fact that a defendant does not have a bank account in the forum state does not rule out constitutionally exercising personal jurisdiction over that defendant. For general jurisdiction, the court would look to the defendant's contacts in the forum state in order to assess whether the defendant is "at home" in the forum state. This is a heightened standard that is not satisfied by the mere existence of (or lack thereof) of one single contact in the state. B is incorrect because the existence of a bank account is only one form of contact with the forum state that is not, by itself, sufficient to satisfy the requirements for personal jurisdiction. For general jurisdiction, the court would look to the defendant's contacts in the forum state in order to assess whether the defendant is "at home" in the forum state. This is a heightened standard that is not satisfied by the mere existence of (or lack thereof) of one single contact in the state. C is incorrect because a tortuous act was committed in Louisiana; namely, the whiskey was manufactured in Louisiana. For general jurisdiction, the court would look to the defendant's contacts in the forum state in order to assess whether the defendant is "at home" in the forum state. This is a heightened standard that is not satisfied by the mere existence of (or lack thereof) of one single contact in the state.

An industrial accident injured Worker while using a machine for work. Worker was using the machine as intended while working for Shipping Co., incorporated in State A and principal place of business in State B. Worker worked in State B but was a citizen of State C. Worker was severally injured requiring months of hospitalization and rehabilitation. After Worker recovered from his injuries, he filed suit in State C state court against his employer Shipping Co. alleging that the shift supervisor at Shipping Co. had intentionally placed the machine on a decline, causing the machine to tip over and pin Worker to the ground, because the supervisor was angry with Worker. Before filing its answer, Shipping Co. removed the case from state court to federal court in State C. Worker filed a motion to remand. How should the court rule on the Worker's motion? (A) The court should grant the motion because the court lacks subject matter jurisdiction over Worker's claim. (B) The court should grant the motion because cases arising under State B's workman's compensation statutes are barred from removal under 28 U.S.C. §1445. (C) The court should deny the motion because federal courts have exclusive jurisdiction over diversity suits under 28 U.S.C. §1332. (D) The court should deny the motion because the federal district court has subject matter jurisdiction over this case under §1332.

D is the correct answer. The elements of diversity jurisdiction under 28 U.S.C. §1332 are satisfied where the Worker is from State C, Shipping Co. is from States A and B, and with extended hospital and rehabilitation stays the amount in controversy is reasonably likely to exceed $75,000. Therefore, removal under §1441 is proper based on §1332. The exclusion found in 28 U.S.C. §1445(c) does not apply here because the Worker's cause of action does not arise under the State's workman's compensation laws but rather a common law intentional tort claim, which would not be barred by the State's workman's compensation laws. Therefore, D is correct and Answer A is incorrect. Answer B is incorrect because 28 U.S.C. §1445 excludes certain cases, which would otherwise be removable, from being removed. Under §1445(c), cases arising under state workman's compensation statutes are barred from removal, even if they otherwise satisfy the requirements for diversity jurisdiction under §1332. However, Worker's claim is not based on a workman's compensation statute but rather is an intentional tort, which is a common law claim not barred by workman's compensation laws. Answer C is incorrect because federal courts do not have exclusive jurisdiction over diversity cases. 28 U.S.C. §1332 grants federal courts authority over cases arising under state law when there is complete diversity of citizenship between the adverse parties and the damages are reasonably likely to exceed $75,000. However, this authority is not exclusive to the federal courts, but rather concurrent. The plaintiff may choose to file their case in state court, which is a court of general jurisdiction, or plaintiff may choose to file in federal court, which is a court of limited jurisdiction. If plaintiff files in state court, then the defendant may have the ability to remove the case to federal court under §1441 or remain in state court.

The plaintiff, a Texas citizen, brings an action against the defendant, an Ohio corporation, in federal district court in Texas asserting two claims. The first claim alleges a violation of the federal antitrust law, which provides for nationwide service of process. The second claim is for breach of contract with respect to an unrelated matter. The trial court has determined that it can constitutionally exercise personal jurisdiction over the defendant with respect to the federal claim. It has also determined that the defendant is not subject to personal jurisdiction under the terms of the Texas long-arm statute. How will the federal district court likely handle the personal jurisdiction issue and why? (A) Decline to exercise personal jurisdiction over the defendant with respect to the entire case, because there is no personal jurisdiction under the Texas long-arm statute. (B) Exercise personal jurisdiction over the defendant with respect to the entire case, because of the doctrine of pendent personal jurisdiction. (C) Exercise personal jurisdiction over the defendant with respect to the entire case without need to evaluate the Texas long-arm statute, because federal law trumps state law under the Supremacy Clause of the U.S. Constitution. (D) Decline to exercise personal jurisdiction over the defendant with respect to the state claim, because the federal and state law claims do not arise out of a common nucleus of operative fact.

D is the correct answer. The issue here is whether the fact that the defendant would not be subject to personal jurisdiction with respect to one of the claims, here the state law claim, precludes the court from exercising jurisdiction over it with respect to the entire case. Also, under the Texas state long-arm statute, there is no basis for exercising personal jurisdiction over the defendant (absent supplemental jurisdiction). Since the state law claim arises out of "an unrelated matter" from the event giving rise to the federal claim, the two claims do not constitute one "case," i.e., they do not arise out of a common nucleus of operative fact. Hence, the court would not apply the doctrine of pendent personal jurisdiction and the state law claim would have to be dismissed. Answer A is incorrect because it does not take into consideration that the defendant is subject to personal jurisdiction on the federal claim but rather precludes the exercise of personal jurisdiction altogether under the Texas long-arm statute. Answer B is incorrect because under pendent personal jurisdiction, the claims would have to arise from the same nucleus of operative facts. Under this set of facts, the two claims are "unrelated." Thus, pendent personal jurisdiction would not apply. Answer C is incorrect because a court cannot exercise jurisdiction over a party unless it is constitutionally sufficient to do so. Here, the defendant is not subject to the court's power under the Texas long-arm statute.

Plaintiff from State A filed suit against the Defendant, from State B in state court in State B. In her complaint, the Plaintiff alleged negligence, negligent workmanship, breach of contract, and misrepresentation stemming from a construction contract she entered into in order to remodel her home. The damages to her home, car, and manicured lawn were well in excess of $150,000. Following the court's swift denial of Defendant's motions to dismiss for improper service and failure to state a claim upon which relief can be granted, the Defendant timely removed the case to federal district court in State B. In the Notice of Removal, Defendant indicated that subject matter jurisdiction was predicated on diversity jurisdiction. Fifteen days later, Plaintiff filed a motion to remand the action to state court. How should the court rule on Plaintiff's motion? (A) The court should deny the Plaintiff's motion to remand because diversity jurisdiction is present under these facts. (B) The court should deny the Plaintiff's motion to remand because it was untimely. (C) The court should grant the Plaintiff's motion to remand because Defendant waived its right to remove when it failed to raise the lack of subject matter jurisdiction in its first motion to dismiss. (D) The court should grant the Plaintiff's motion to remand because the Defendant does not have the right to remove this case to federal court.

D is the correct answer. Under 28 U.S.C. §1441, the Defendant has the right to remove a case from state court to federal court in the judicial district where the action is pending, if the case could have otherwise been brought in federal court originally. However, under 28 U.S.C. §1441(b)(2), if the basis for removal is diversity jurisdiction, the Defendant cannot remove the case if the Defendant is from the state where the action is pending. Here, Defendant is from State B and the action is pending in State B. Since the basis for removal is diversity jurisdiction, Defendant does not have the right to remove this case. Answer A is incorrect because, while it is true that diversity jurisdiction is present under these facts, this answer does not take into consideration the exception to removal jurisdiction when removal is predicated on diversity jurisdiction. Answer B is incorrect because under 28 U.S.C. §1447, the Plaintiff has 30 days from notice of removal to file a motion for remand, which makes this motion filed in 15 days timely. Answer C is incorrect because removal jurisdiction allows the Defendant to remove a case from state court to federal court in the judicial district where the action is pending if the case could have originally been filed in federal court. It is not a means to test the court's subject matter jurisdiction. Also, this answer incorrectly suggests that the lack of subject matter jurisdiction can be waived if not filed in the first response. See FRCP 12(h).

In an action filed in federal district court, the defendant refuses to waive service. The plaintiff serves the defendant by placing a copy of the summons and complaint in the hands of the defendant's housekeeper while she is working in the defendant's home. She forgets to turn it over to the defendant. Is this service proper under the Federal Rules of Civil Procedure? (A) Yes, because the housekeeper is in the defendant's home. (B) No, because the defendant was not personally served. (C) Yes, because the housekeeper is a person of suitable age and discretion. (D) No, because the housekeeper does not reside in the defendant's home.

D is the correct answer. Under Fed. R. Civ. P. 4(e)(2)(B), service is sufficient if it is delivered to the defendant's dwelling house or usual place of abode with some person of suitable age and discretion who resides therein. Since the housekeeper does not live in the defendant's home, this latter requirement has not been met. While some courts have said that this defect can be cured by the fact that the defendant actually receives the notice, this defendant did not receive the process. Answers A and C are incorrect because they do not consider the requirement that the person accepting service must reside therein, which the housekeeper does not. Answer B is incorrect because Fed. R. Civ. P. 4(e)(2)(B) allows for substituted service, meaning that the defendant need not be personally served so long as service is made at the individual's dwelling or usual place of abode with someone of suitable age and discretion who resides there.

Plaintiff files a claim against the defendant, a Delaware corporation with its headquarters located in New York, in federal district court in Los Angeles, California, alleging a violation of the federal antitrust statute. The federal statute contains a provision for nationwide service of process and personal jurisdiction over corporations. The defendant is a major retailer with retail shops in all 50 states and a highly interactive web page that generates hundreds of thousands of dollars of business annually from citizens of all 50 states. Which of the following governs whether or not the defendant is subject to personal jurisdiction? (A) California law, because California is the forum state. (B) Delaware law, because the defendant is a Delaware citizen. (C) New York law, because the defendant's headquarters are there. (D) The federal antitrust statute

D is the correct answer. Under Fed. R. Civ. P. 4(k)(1), a federal court looks to the forum state's jurisdictional provision unless otherwise provided by federal law. Here, the substantive federal law contains its own jurisdictional provision. Consequently, the federal antitrust statute, rather than state law, applies. Answers A, B, and C are incorrect because the federal statute provides for nationwide service of process, as authorized by Fed. R. Civ. P. 4(k)(1).

Plaintiff is injured in a crash between his car and another car and a truck on a narrow country road in Concord, New Hampshire. Plaintiff is a citizen of New Hampshire. The driver of the other car is a citizen of Maine, and the owner of the truck, Trucks, Inc., is incorporated under the laws of Delaware and has its principal place of business in Connecticut. Plaintiff filed an action in the federal court for the District of New Hampshire seeking damages of $250,000 against each of the two named defendants, Driver and Trucks, Inc. In the same suit, Driver filed a claim against Trucks, Inc. seeking $75,000 in damages for the damage to her car. Trucks, Inc. filed a tort claim against Plaintiff seeking $70,000 in damages for the damages to its truck. Trucks, Inc. also filed a claim against Trucker, a citizen of Connecticut who was driving its truck at the time of the accident, seeking indemnity from any loss it might suffer in connection with Plaintiff's claim against it. Can the court exercise subject matter jurisdiction over Trucks, Inc.'s claim against Trucker? (A) No, because Trucks, Inc. and Trucker are citizens of the same state. (B) Yes, because Trucker is not a citizen of Georgia. (C) Yes, because this claim arose out of the car crash. (D) No, because original jurisdiction was based solely on diversity.

The correct answer here is C. This is a state law claim between non-diverse parties so there is no original jurisdiction over this claim. It is a third-party claim in a case where original jurisdiction was founded solely on diversity. Thus, we need to look to §1367(b), but the ban on supplemental jurisdiction in §1367(b) only applies to claims by plaintiffs and this is a third-party claim by a defendant and so the limitation of §1367(b) is inapplicable. We then turn to §1367(a), which would allow supplemental jurisdiction if this claim forms part of the same case as the claims within the court's original jurisdiction. Since this third-party claim is for indemnity for any loss suffered by Trucks, Inc. in connection with the plaintiff's claim against it, the common nucleus of operative fact standard of §1367(a) has been met and therefore the court can exercise supplemental jurisdiction. Answer A is incorrect because the lack of diversity is not fatal to the exercise of subject matter jurisdiction. This answer fails to consider the exercise of supplemental jurisdiction. Answer B is incorrect because it is irrelevant to this question of joinder and subject matter jurisdiction. Answer D is incorrect because while this fact is true, the prohibition on supplemental jurisdiction contained in §1367(b) does not apply to this claim because it is not a claim by a plaintiff.

A citizen of Oregon was injured in an accident by a citizen of Wyoming. Before filing suit in state court, the Oregonian assigned his claim to a citizen of Wyoming on the understanding that the Wyoming citizen would transfer all proceeds recovered at trial back to the injured party in exchange for a fee for expenses and services provided. This individual then filed the suit seeking $100,000 in damages in Oregon state court. The defendant sought to remove the case on the basis of diversity jurisdiction. Does the federal court have subject matter jurisdiction? (A) Yes, because the parties are diverse and the claim is for $100,000. (B) Yes, because §1359 is inapplicable. (C) No, because the claim arises under state law. (D) No, because the parties are not diverse.

The correct answer is A. The trick here is that the assignment, if collusive in intention, was made for the purpose of defeating diversity jurisdiction. Section 1359 expressly applies only to collusive joinder for the purpose of invoking federal jurisdiction. Nevertheless, the federal courts have construed §1359 to apply to collusive joinder to both invoke and defeat jurisdiction. Thus, §1359 would apply here if the assignment was collusive. Since the original claimholder retained all real interest in the claim minus a service fee, the assignment was collusive. Thus, the original claimholder's citizenship governs and there is diversity jurisdiction since the state law claim is for $100,000. Answer B is incorrect. Section 1359 expressly applies to collusive joinder for the purpose of invoking federal jurisdiction. Nevertheless, the federal courts have construed §1359 to apply to collusive joinder to both invoke and defeat jurisdiction (as is the case here). Thus, §1359 would apply here because the assignment was collusive. Answer C is incorrect because federal courts have subject matter jurisdiction over state law matters when the parties can satisfy the requirements for diversity jurisdiction under 28 U.S.C. §1332. Here, the plaintiff is from Wyoming, the defendant is from Oregon, and the claim is for $100,000. Both requirements have been met. Answer D is incorrect. Under §1359, if there is collusive joinder, the original claimholder's citizenship governs. The original claimholder was from Oregon and the defendant was from Wyoming, making the parties diverse in citizenship.

A citizen of Pennsylvania is injured in a car accident with another citizen of that same state who was not injured in the crash. The injured party was in desperate need of cash and so he assigned his cause of action to a citizen of New York for $50,000. The New Yorker then filed the claim in federal court in New York, seeking damages in the amount of $100,000 against the other driver. Does the court have subject matter jurisdiction? (A) No, because the parties are not diverse. (B) Yes, because the parties are diverse and the claim is for $100,000. (C) No, because this is a collusive assignment. (D) Yes, because Plaintiff is a citizen of New York.

The correct answer is B. 28 U.S.C. §1359 states that a district court will not have subject matter jurisdiction when a party has been collusively joined for the purpose of invoking that court's subject matter jurisdiction. The question, therefore, is whether the assignment to the New Yorker was made by the original claimholder for the purpose of creating diversity-based jurisdiction. This turns on whether the original claimholder/assignor retained an interest in the claim or where other aspects of the assignment indicate that it was made solely for the purpose of invoking federal jurisdiction. Since, in this case, the original claimholder gave up all interest in the claim for a reasonable amount of money, it is clear that this was not a collusive assignment within the meaning of §1359. Thus, since the defendant and (New York) plaintiff are diverse and the claim is in excess of $75,000, there is jurisdiction under §1332(a)(1). Answer A is incorrect as the parties are diverse where Plaintiff is from New York and Defendant is from Pennsylvania. The original injured party (a Pennsylvania citizen) is no longer a part of the action and cannot destroy diversity. Answer C is incorrect as this was not a collusive assignment. The question is whether the original claimholder/assignor retained an interest in the claim or where other aspects of the assignment indicate that it was made solely for the purpose of invoking federal jurisdiction. Since the original claimholder gave up all interest in the claim for a reasonable amount of money, it is clear that this was not a collusive assignment within the meaning of §1359. Answer D is incorrect. The fact that the plaintiff is a citizen of the forum state is irrelevant to this question as the issue turns on whether there was a collusive assignment. Also, there is no rule which requires the plaintiff to file in their home state nor is there a rule that provides the home state plaintiff with subject matter jurisdiction in that forum.

A citizen of Indiana brings an action against her employer, an Indiana corporation, in federal district court. Claim I alleges that she was discriminated against on the basis of her sex in violation of the federal Civil Rights Act of 1964. The second claim consists of a tort claim for intentional infliction of emotional distress arising out of the same series of acts of sexual harassment that formed the basis of her federal sex discrimination claim. The plaintiff seeks $250,000 in damages in connection with each claim. Does the court have subject matter jurisdiction over the entire lawsuit? (A) No, not over the second claim because it is a non-diverse state law claim. (B) Yes, because the two claims arise out of a common nucleus of operative fact and there is no reason to decline to exercise supplemental jurisdiction over the state claim. (C) No, because the defendant is a citizen of the forum state. (D) Yes, because federal courts have concurrent jurisdiction over most federal law claims.

The correct answer is B. A federal court must have subject matter jurisdiction over every claim in the lawsuit. The federal claim clearly falls within the court's §1331 jurisdiction. The second claim, however, is a non-diverse state law claim since both parties are citizens of Indiana. Thus, it does not fall within the court's original jurisdiction under either §1331 or §1332. This, then, raises the question of whether the claim falls within the court's supplemental jurisdiction as provided by 28 U.S.C. §1367. This is a "pendent claim jurisdiction" first articulated by the Supreme Court in UMW v. Gibbs. It consists of a federal question claim and a non-diverse state claim brought by one plaintiff against one defendant. Congress codified this doctrine in §1367(a), which gives federal courts the discretion to exercise supplemental jurisdiction under these circumstances as long as the two claims form part of the same "case" under Article III of the Constitution, i.e., they arise out of a common nucleus of operative facts. If they do not, then the state claim must be dismissed and the federal court retains jurisdiction over the federal claim only. If they do form part of the same "case," then the court has the discretion to exercise jurisdiction over the (non-diverse state law) claim that did not fall within its original jurisdiction. And §1367(c)(1)-(4) lists the factors that the court should consider in deciding whether or not to decline to exercise this supplemental jurisdiction over the non-diverse state law claim. In this question, both claims arise out of the same series of alleged acts of harassment and so the common nucleus test is satisfied. And none of the factors listed in §1367(c) such as predominance of state issues or novelty of state law issues or dismissal of the federal claim is present to justify dismissal of the state law claim. Therefore, the fact that there is no independent subject matter jurisdiction over the non-diverse state law claim is not fatal and so answer A is incorrect. Answer C is incorrect because the fact that the defendant is a forum citizen is irrelevant to this question dealing with pendent non-diverse claims. Answer D is incorrect because the fact that federal courts have concurrent, rather than exclusive subject matter jurisdiction over most federal claims is also irrelevant to this question. The federal law violation claim is not at issue, but rather at issue is the non-diverse state law intentional infliction of emotional distress arising from the sexual harassment. In order to adjudicate the entire controversy, the federal court must have subject matter jurisdiction over each claim.

A citizen of Pennsylvania is injured in a car accident with another citizen of that same state who was not injured in the crash. The injured party assigned his claim to a New York citizen on the understanding that the New Yorker would transfer all proceeds recovered at trial back to the injured party in exchange for a fee for expenses and services provided. The New Yorker then files the claim in federal court in Pennsylvania against the other driver, seeking damages in the amount of $100,000. Does the court have subject matter jurisdiction? (A) Yes, because all §1332(a)(1) requirements are met. (B) Yes, because this is not a collusive assignment. (C) No, because the parties are not diverse. (D) No, because the defendant is a citizen of Pennsylvania.

The correct answer is C. 28 U.S.C. §1359 states that a district court will not have subject matter jurisdiction when a party has been collusively joined for the purpose of invoking that court's subject matter jurisdiction. The question, therefore, is whether the assignment to the New Yorker was made by the original claimholder for the purpose of creating diversity-based jurisdiction. This turns on whether the original claimholder/assignor retained an interest in the claim or where other aspects of the assignment indicate that it was made solely for the purpose of invoking federal jurisdiction. Since, in this case, the original claimholder retained all of his interest in the claim and only paid the putative plaintiff a service fee, the courts would find this to be a collusive joinder and disregard the citizenship of the assignee. Answer A is incorrect because the diversity requirements have not been met. Since the citizenship of the original claimholder (Pennsylvania) would govern, the parties are not diverse. Also, the court would find a collusive assignment took place. Answer B is incorrect because, under these facts, there is collusive joinder, where the original claimholder/assignor retained an interest in the claim or where other aspects of the assignment indicate that it was made solely for the purpose of invoking federal jurisdiction. Under 28 U.S.C. §1359, the federal court does not have subject matter jurisdiction. Answer D is incorrect because the fact that the defendant is a citizen of the forum state is irrelevant to this question. This incorrect answer choice is addressing the forum defendant rule for removal jurisdiction, which does not apply under these facts because the original cause of action was filed in federal court. Under that rule, a defendant from the forum state cannot remove a case from state court to federal court, if the basis for removal is diversity jurisdiction. Again, this is inapplicable, as the case has already been filed in federal court.

Plaintiff is injured in a crash between his car and another car and a truck on a narrow country road in Concord, New Hampshire. Plaintiff is a citizen of New Hampshire. Driver, the driver of the other car, is a citizen of Maine, and the owner of the truck, Trucks, Inc., is incorporated under the laws of Delaware and has its principal place of business in Connecticut. Plaintiff filed an action in the federal court for the District of New Hampshire seeking damages of $250,000 against each of the two named defendants, Driver and Trucks, Inc. In the same suit, Driver filed a claim against Trucks, Inc. seeking $75,000 in damages for the damage to her car. Trucks, Inc. filed a tort claim against Plaintiff seeking $70,000 in damages for the damages to its truck. Trucks, Inc. also filed a claim against Trucker, a citizen of Connecticut who was driving its truck at the time of the accident, seeking indemnity from any loss it might suffer in connection with Plaintiff's claim against it. Can the court exercise subject matter jurisdiction over Driver's claim against Trucks, Inc.? (A) Yes, because Driver and Trucks, Inc. are citizens of different states. (B) No, because Driver is seeking $70,000. (C) Yes, because this claim arose out of the car crash. (D) No, because original jurisdiction was based solely on diversity.

The correct answer is C. Driver and Trucks, Inc. are diverse, but since the amount in controversy is only $75,000, it does not exceed $75,000 and therefore does not meet the amount in controversy requirement of §1332. Since this is a state law claim, the court does not have original jurisdiction over this cross-claim. Since original jurisdiction over the civil action is founded solely on §1332, whether or not the court can exercise supplemental jurisdiction over Driver's claim involves the application of §1367(b). This section prohibits the exercise of supplemental jurisdiction in diversity-based cases, but only over claims by "plaintiffs." Although Driver is a cross-plaintiff, this does not meet the §1367(b) requirement of being a plaintiff. She is a defendant and cross-plaintiff and so the limitation of §1367(b) is inapplicable here. So we must go to §1367(a) and ask if this claim forms part of the same "case" as claims over which the court has original jurisdiction. And it does, since this claim arises out of the accident that gave rise to the plaintiff's claims. So the common nucleus of operative fact standard of §1367(a) is met. Answer A is incorrect because the amount in controversy requirement has not been met so the fact of diversity is not sufficient to create original jurisdiction under §1332. Answer B is incorrect because while not meeting the amount in controversy requirement is a problem for original jurisdiction, it does not preclude the exercise of supplemental jurisdiction under §1367(a). Answer D is incorrect because, while true, it does not preclude the exercise of supplemental jurisdiction since the limitation in §1367(b) does not apply here because this was not a claim by a plaintiff.

A purchaser of a television brought an action against the manufacturer in federal district court in Massachusetts containing a claim that the manufacturer participated in a conspiracy to engage in unlawful price-fixing in violation of the federal antitrust laws. The plaintiff is a citizen of Massachusetts and the defendant is a citizen of Delaware and Massachusetts. In his complaint, the plaintiff also asserted a breach of contract claim, alleging that the television was defective and that the manufacturer refused to honor the warranty contained in the purchase agreement. In connection with this second claim, the plaintiff seeks $100,000 in damages. Can the court exercise subject matter jurisdiction over the entire lawsuit? (A) No, because both parties are citizens of Massachusetts. (B) Yes, because the court can exercise supplemental jurisdiction over the state claim and §1331 jurisdiction over the federal claim. (C) No, because the claims do not arise out of a common nucleus of operative fact. (D) Yes, because the value of the state claim exceeds $75,000.

The correct answer is C. The federal claim falls within the court's §1331 jurisdiction. The non-diverse state claim (both parties are citizens of Massachusetts) is subject to supplemental jurisdiction under §1367 only if the two claims form part of the same case, meaning they arise out of a common nucleus of operative facts and if none of the factors listed in §1367(c)(1)-(4) justify declining to exercise such supplemental jurisdiction. Here, the facts giving rise to the price-fixing conspiracy are different from and unrelated to the facts giving rise to the breach of contract claim; the two claims do not form part of the same case. Therefore, there can be no supplemental jurisdiction over the non-diverse state law claim. Answer A is incorrect because the fact that the claim is a non-diverse state law claim would not preclude the exercise of supplemental jurisdiction if the requirements of §1367 were satisfied; namely, that the federal claim and the non-diverse state law claims arose from the common nucleus of operative facts, which they did not. Section 1367(b) only applies when jurisdiction is based solely on diversity jurisdiction under §1332. Answer B is incorrect because it does not take into account the fact that two claims do not form part of the same case, as required to exercise supplemental jurisdiction over non-diverse state law claims under §1367. Answer D is incorrect because the fact that the state law claim meets the jurisdictional amount in controversy requirement is not sufficient for §1332-based jurisdiction since the parties are not diverse.

Plaintiff is injured in a crash between his car and another car and a truck on a narrow country road in Concord, New Hampshire. Plaintiff is a citizen of New Hampshire. Driver, the driver of the other car, is a citizen of Maine, and the owner of the truck, Trucks, Inc., is incorporated under the laws of Delaware and has its principal place of business in Connecticut. Plaintiff filed an action in the federal court for the District of New Hampshire seeking damages of $250,000 against each of the two named defendants, Driver and Trucks, Inc. In the same suit, Driver filed a claim against Trucks, Inc. seeking $75,000 in damages for the damage to her car. Trucks, Inc. filed a tort claim against Plaintiff seeking $70,000 in damages for the damages to its truck. Trucks, Inc. also filed a claim against Trucker, a citizen of Connecticut who was driving its truck at the time of the accident, seeking indemnity from any loss it might suffer in connection with Plaintiff's claim against it. Can the court exercise subject matter jurisdiction over Trucks, Inc.'s claim against Plaintiff? (A) Yes, because Trucks, Inc. and Plaintiff are citizens of different states. (B) No, because Trucks, Inc. is seeking $70,000. (C) Yes, because this claim arose out of the car crash. (D) No, because original jurisdiction was based solely on diversity.

The correct answer is C. This is a state law counterclaim between diverse parties but as to which the amount in controversy requirement has not been met. So there is no original jurisdiction over this claim. But since original jurisdiction over the civil action is founded solely on §1332, whether or not the court can exercise supplemental jurisdiction over Trucks, Inc.'s counterclaim involves the application of §1367(b). This section prohibits the exercise of supplemental jurisdiction in diversity-based cases, but only over claims by "plaintiffs." Although Trucks, Inc. is a counter-plaintiff, this does not meet the §1367(b) requirement of being a plaintiff. Trucks, Inc. is a defendant and counter-plaintiff and so the limitation of §1367(b) is inapplicable here. So we must go to §1367(a) and ask if this claim forms part of the same "case" as claims over which the court has original jurisdiction. And it does, since this claim arises out of the accident that gave rise to the plaintiff's claims. So the common nucleus of operative fact standard of §1367(a) is met. Answer A is incorrect because the amount in controversy requirement has not been met, so the fact of diversity is not sufficient to create original jurisdiction under §1332. Answer B is incorrect because while not meeting the amount in controversy requirement is a problem for original jurisdiction, it does not preclude the exercise of supplemental jurisdiction under §1367(a). Answer D is incorrect because, while true, it does not preclude the exercise of supplemental jurisdiction since the limitation in §1367(b) does not apply to this claim by a non-plaintiff.

A citizen of Indiana brings an action against her employer, an Indiana corporation, in federal district court. Claim I alleges that she was discriminated against on the basis of her sex in violation of the federal Civil Rights Act of 1964. The second claim consists of a tort claim for negligent infliction of emotional distress arising out of the same series of acts of sexual harassment that formed the basis of her federal sex discrimination claim. The plaintiff seeks $100,000 in compensatory damages and $1 million in punitive damages under each of these two claims. The availability of punitive damages in negligent infliction of emotional distress cases has not previously been addressed under the governing Indiana state law. Should the court exercise subject matter jurisdiction over the entire lawsuit? (A) Yes, because the two claims arise out of a common nucleus of operative fact. (B) No, not over the state law claim because the parties are not diverse. (C) Yes, because the issues raised in the two claims are nearly identical. (D) No, not over the state law claim because it raises a novel issue of state law.

The correct answer is D. The federal claim falls within the court's §1331 jurisdiction. The non-diverse state claim is subject to supplemental jurisdiction under §1367 if the two claims form part of the same case, i.e., arise out of a common nucleus of operative fact AND if none of the factors listed in §1367(c) justify declining to exercise such supplemental jurisdiction. Here, the two claims do form part of the same case, but since the damages issue under state law is a question of first impression, this falls within the §1367(c)(1) factor justifying a decision to decline to exercise supplemental jurisdiction over the state claim. Answer A is incorrect because, while it is true as stated, the answer choice fails to consider the discretionary factors of §1367(c), which permit the federal court to decline to exercise supplemental jurisdiction over state law claims that arise from the common nucleus of operative facts. Answer B is incorrect because the fact that the parties are non-diverse is only fatal to jurisdiction under §1332, not under §1367. Answer C is incorrect because whether or not claims are nearly identical is not a standard for the exercise of supplemental jurisdiction. The standard under §1367 allows that in any civil action of which the district courts have original jurisdiction, the district courts shall have supplemental jurisdiction over all other claims that are so related to claims in the action within such original jurisdiction that they form part of the same case or controversy under Article III of the United States Constitution.

Plaintiff, a lifelong resident of Atlanta, Georgia, brought suit in federal district court in Atlanta against a publisher, Books, Inc., alleging copyright infringement under the federal copyright statute, for which she sought $100,000 in damages. Books, Inc. is incorporated under the laws of Delaware and has its principal place of business in New York City. Plaintiff also filed a claim against Books, Inc. alleging that it had breached its agreement with her to publish her book upon receipt of a manuscript. She sought $60,000 in damages in connection with that claim. In that same suit, Plaintiff also filed a $105,000 tort claim against the president of Books, Inc., a citizen of New York, alleging that he had intentionally inflicted emotional distress upon her by sending emails to dozens of other publishers denouncing her as a horrible writer and a fraud and explaining that those were the reasons for his company's refusal to publish her book. In this same action, Books, Inc. filed a claim against President seeking indemnity for any liability it might accrue in connection with Plaintiff's copyright infringement claim against it. In response, President filed a breach of contract claim against Books, Inc. alleging that Books, Inc. had improperly withheld two weeks of salary from him totaling $65,000. Finally, President asserted a tort claim against his neighbor, claiming that her negligent maintenance of her home significantly depreciated the value of his property, for which he sought an injunction and $45,000 in damages. Can the court exercise subject matter jurisdiction over President's claim against neighbor? (A) No, because the claim is for $45,000. (B) No, because President is seeking an injunction. (C) No, because neither President nor neighbor are citizens of Georgia. (D) No, because this claim seeks relief from neighbor's maintenance of her property.

The correct answer is D. The parties are both citizens of New York and this is a state law claim so there is no original jurisdiction over this claim. There is a federal question claim (plaintiff's copyright infringement claim) forming original jurisdiction in the case so the question is whether this claim falls within the court's supplemental jurisdiction under §1367(a), which turns on whether this claim forms part of the same "case" as claims within the court's original jurisdiction. Because this claim alleges that the neighbor negligently maintained her property, it does not arise out of the nucleus of operative fact giving rise to the copyright claim. So the court cannot exercise supplemental jurisdiction over it. Answer A is incorrect because the parties are not diverse and so the amount in controversy is irrelevant. Moreover, if the parties had been diverse, the amount in controversy requirement might have been met since President is seeking both damages and an injunction and the court would have to place a value on the rights sought to be protected by the injunction, which could bump the amount in controversy above the minimum requirement. Answer B is incorrect because it is irrelevant. Federal courts can have subject matter jurisdiction over claims where the claimant is seeking injunctive relief, provided that the claim meets federal subject matter jurisdiction requirements. Answer C is incorrect because the fact that neither of the parties is a forum citizen might be relevant to personal jurisdiction, but is not relevant to subject matter jurisdiction.

Plaintiff owns a deli called Health Foods. Defendant runs a company that sells "organic" baked goods. Plaintiff entered into a contract with Defendant for the weekly delivery of organic oatmeal cookies. It turns out that the "organic" cookies are really repackaged highly-processed cookies that Defendant buys in bulk from a foreign wholesale company. Plaintiff sued Defendant in federal court, alleging diversity jurisdiction. The complaint included a proper jurisdictional statement and adequately made a demand for judgment. It further alleged: (1) Defendant entered into a contract with Plaintiff to deliver thirty pounds of organic oatmeal cookies weekly, (2) Defendant instead fraudulently delivered non-organic cookies, and (3) Defendant engaged in fraud. Defendant moved to dismiss the complaint for failing to state a claim. How should the court rule on the motion? (A) The court should grant the motion, because the complaint fails to allege special matters. (B) The court should deny the motion, because Defendant has not shown that Plaintiff can prove no set of facts in support of the fraud claim. (C) The court should deny the motion, because Defendant was required to move for judgment on the pleadings. (D) The court should grant the motion, because the complaint improperly joins a contract claim with a tort claim.

(A) is correct because the complaint alleges fraud and so is required to plead "special matters." FRCP 9(b) requires that, "[i]n alleging fraud or mistake, a party must state with particularity the circumstances constituting fraud or mistake." Here, the complaint includes only the conclusory allegation that Defendant "fraudulently" delivered goods that did not comply with the contract and that Defendant engaged in fraud. Courts interpret the special pleading rules for fraud to require that the complaint allege the "precise misconduct" that constitutes the fraud. Generally, this heightened pleading requirement is met when the complaint alleges the who, what, when, where, and how of the fraudulent representation. Lerner v. Fleet Bank, N.A., 459 F.3d 273 (2d Cir. 2006) (to meet the requirements of FRCP 9(b) when fraud is alleged, "the complaint must: (1) specify the statements that the plaintiff contends were fraudulent, (2) identify the speaker, (3) state where and when the statements were made, and (4) explain why the statements were fraudulent") (citations omitted). Here, the allegations of the complaint do not meet this standard because they do not include the "when, where, and how" elements. Moreover, to allege fraud with particularity, the complaint must include specific facts to give rise to a "strong inference" of fraud, and the complaint here contains only a bare assertion that fraud occurred. Not only is this allegation insufficient to meet the heightened pleading requirement of FRCP 9(b), but also it likely would fail the general pleading rule of plausibility under FRCP 8 as interpreted by the Supreme Court in Bell Atl. Inc. v. Twombly, 550 U.S. 544 (2007). (B) is not correct because it relies on a pleading standard that the Supreme Court has abrogated. Every complaint—even if it does not allege special matters—must meet the plausibility requirement of FRCP 8. Earlier, in Conley v. Gibson, 355 U.S. 41, 45-46 (1957), the Court held that a complaint should not be dismissed for failure to state a claim "unless it appears beyond doubt that the plaintiff can prove no set of facts in support of his claim which could entitle him to relief." However, the Court has since held that the quoted language "earned its retirement" and that a complaint is insufficient if it alleges merely conclusory statements that do not rise above the level of speculation. See Bell Atl. Corp. v. Twombly, 550 U.S. 544, 554-563 (2007). The pleading of allegations in formulaic terms is considered conclusory under Twombly. (C) is not correct because a party is never required to move for a judgment on the pleadings and, in any event, such a motion is available only after the pleadings are "closed." FRCP 12(c). In contrast, a motion to dismiss for failure to state a claim is filed prior to any responsive pleading, and thus prior to any motion for judgment on the pleadings. SeeFRCP 12(b)(6) ("A motion asserting any of these defenses must be made before pleading if a responsive pleading is allowed."). The pleadings are closed only after an answer has been filed. See FRCP 7(a). A motion for judgment on the pleadings seeks a disposition on the merits when the material facts are not in dispute and the movant is entitled to judgment as a matter of law. (D) is not correct because under federal pleading rules, a plaintiff may join a tort and a contract claim in one lawsuit and, indeed, may join "as many claims as it has against an opposing party." FRCP 18(a). The federal joinder rules are broad and do not restrict the joinder of claims even if they relate to different transactions or depend on different legal theories. Because joinder under the Federal Rules is generally permissive and allows for the pleading of all potential claims against a single party, the joinder of a contract claim and a tort claim does not serve as grounds to dismiss the complaint. Keep in mind that although the joinder rule is permissive, it does not confer jurisdiction on the court (FRCP 82).

Plaintiff is the owner of a company that rents ski equipment. Defendant is a college student who writes a weekend column in the student newspaper about sports events. Defendant wrote an uncomplimentary article about Plaintiff's company, stating that the rental equipment "was old, shoddy, and dangerous." In response, Plaintiff filed a diversity action in federal court against Defendant alleging libel and seeking $300,000 in damages and reasonable attorney's fees. Defendant failed to appear. Plaintiff submitted an affidavit showing Defendant's failure and the clerk entered a default. Can the clerk also enter a default judgment? (A) No, only the judge can enter the judgment and must make an independent determination of damages. (B) Yes, the clerk can enter the judgment if Plaintiff submits an affidavit stating a sum certain for damages. (C) The entry of default automatically converts into a judgment of default if Defendant fails to object. (D) Yes, the clerk can enter the judgment after Defendant has been served with written notice of the application at least seven days before a hearing.

(A) is correct because under the federal rules, only the court can enter a default judgment if plaintiff's claim is not "for a sum certain or a sum that can be made certain by computation." FRCP 55(b)(1). When there is any doubt about the amount of damages, then only the court can enter the judgment. See FRCP 55(b)(2). Although the complaint contains an ad damnum clause, the amount of damages here are not a "sum certain" in the sense contemplated by FRCP 55. See KPS & Assocs., Inc. v. Designs By FMC, Inc., 318 F.3d 1, 19 (1st Cir. 2003) ("in the Rule 55 context, a claim is not a sum certain unless there is no doubt as to the amount to which a claimant is entitled as a result of the defendant's default", stating that the court cannot award the amount of the ad damnum clause in the complaint when the proper amount is uncertain). The usual example of a dispute involving a sum certain, in which resort to "extrinsic proof" is not needed to determine damages, is an action to enforce a money judgment or a negotiable instrument. Id. When the complaint does not involve a sum certain, the federal rule provides that the party seeking judgment must apply to the court for the entry of a default judgment, and the court may hold a hearing "to determine the amount of damages" or to "investigate any other matter." FRCP 55(b)(2). It is not clear without further evidence whether the $300,000 that Plaintiff has alleged is the appropriate amount in damages for the libel; moreover, the amount of a reasonable attorney's fee remains disputed. (B) is not correct because although the federal rule permits the clerk to enter a default judgment for liability and damages, the clerk may do so only if it is clear from the complaint, without resort to extrinsic evidence, that the damages sought are for a sum certain. See FRCP 55(b)(1). Even if for a sum certain, a default judgment may be entered only if two other conditions are met: it must be shown that "defendant has been defaulted for not appearing"; and defendant "is neither a minor nor an incompetent person." Id. Thus, in addition to the sum-certain requirement that damages be "capable of simple mathematical computation," the clerk may enter judgment only against "parties who have never appeared in the action." Franchise Holding II, LLC. v. Huntington Rests. Group, Inc., 375 F.3d 922, 928-929 (9th Cir. 2004), and are not entitled to the protection afforded to minors and incompetents. (C) is not correct because it misstates the rule pertaining to how the entry of default becomes a default judgment. The entry of default does not automatically convert into a judgment; rather, the federal rule "provides a two-step process for obtaining a default judgment," and either the clerk or the court must enter the judgment. See New York v. Green, 420 F.3d 99, 104 (2d Cir. 2005); FRCP 55 (b). An entry of default is a notation made by the clerk after the party seeking the default submits an affidavit showing that a party has "failed to plead or otherwise defend." See FRCP 55(a). The entry "formalizes a judicial recognition that a defendant has, through its failure to defend the action admitted liability to the plaintiff; the entry is not an admission of damages"; the entry of a default judgment "converts the defendant's admission of liability into a final judgment that terminates the litigation and awards the plaintiff any relief to which the court decides it is entitled..." City of New York v. Mickalis Pawn Shop, LLC, 645 F.3d 114, 128-129 (2d Cir. 2011). (D) is not correct because it misstates the timing and other requirements for the entry of a default judgment following the entry of default. In cases where only the judge can enter the judgment, see FRCP 55(b)(2), the federal rule provides that a party who "has appeared personally or by a representative ... must be served at least 7 days before the hearing" on the application. Id. However, the seven-day rule is not implicated when the clerk enters the judgment. SeeFRCP 55(b)(1).

Plaintiff was driving a car and stopped at a red light. Defendant, a taxi driver, crashed into the rear end of Plaintiff's car. Defendant was employed by Company, which owns Defendant's taxi. Plaintiff filed a diversity action in federal court against Defendant and Company alleging negligence. Paragraph 16 of the complaint alleged, "Defendant was acting in the course of his employment when the accident took place." Before answering the complaint, Company reviewed three internal documents: (a) a scheduling book that showed Defendant was not officially assigned to work on the day of the accident; (b) a telephone log that indicated two of the Company's other drivers called in sick on the day of the accident; and (c) a garage receipt showing that Defendant signed the taxi out of the garage on the day of the accident. Moreover, Company requested an interview with Defendant, but Defendant's counsel refused to make him available for questions. How should Company plead in response to the allegation of Paragraph 16? (A) Company should answer, "No response is required because the allegation is conclusory and without factual support." (B) Company should answer, "No response is required because the allegation is directed to a different party." (C) Company should answer, "Company is without knowledge or information sufficient to form a belief as to the truth of the allegation." (D) Company should answer, "No response is required because the allegation is a legal conclusion."

(C) is correct because it appropriately applies the federal rules regarding a responsive pleading when a defendant lacks sufficient knowledge to admit or deny an allegation of the complaint. The federal rules address the responsibility of a defendant, as here, who seeks to answer the complaint, but does not have sufficient knowledge to know whether an allegation is true: "A party that lacks knowledge or information sufficient to form a belief about the truth of an allegation must so state, and the statement has the effect of a denial." FRCP 8(b)(5). Company knows that Defendant is its employee, but does not know for sure whether Defendant was acting in the scope of his employment when it checked the taxi out of the garage on the day of the accident. From the documents that Company has found, it cannot conclude that Defendant was acting in the course of employment. Defendant was not scheduled to work that day, and so Company is not obliged to admit the allegation, and can keep its relationship with Defendant in contention. On the other hand, it is possible that Defendant was standing in for another driver who called in sick on the day of the accident, and so Company cannot conclude that Defendant was not driving in the scope of his employment, nor can Company deny this fact in good faith. Since an answer that says Company is without knowledge or information to form a belief as to the truth of the allegations "has the effect of a denial," FRCP 8(b)(5), Company does not lose anything by not denying the allegation. Moreover, this answer satisfies the obligation of Company and Company's attorney's to conduct a reasonable investigation before filing the responsive pleading. Company's position is "to the best of the person's knowledge, information, and belief, formed after an inquiry reasonable under the circumstances." FRCP 11. (A) is not correct because although it correctly recognizes that Paragraph 16 is alleged in a barebones and formulaic way—and so it is "conclusory"—it misstates Defendant's obligation to provide a responsive pleading under FRCP 8(b). In 2007, the U.S. Supreme Court held that the allegations of a complaint, to survive a motion to dismiss under FRCP 12(b)(6) for failure to state a claim, must contain "more than labels and conclusions," and that a "formulaic recitation of the elements of a cause of action will not do." Rather, the complaint's factual allegations must be "enough to raise a right to relief above the speculative level." Bell Atl. Corp. v. Twombly, 550 U.S. 544, 555 (2007). Thus, were Defendant moving to dismiss the complaint for failure to state a claim, the district court likely would not credit the conclusory allegation of Paragraph 16 because it is expressed in formulaic terms and is not accompanied by facts that show a "plausible entitlement" to relief. Id. at 546. However, Defendant has not chosen to move to dismiss; it has chosen to answer, and its obligation is to "admit or deny the allegations" or to state that it "lacks knowledge or information sufficient to form a belief about the truth of an allegation." FRCP 8(b)(1)(A) & (5). (B) is not correct because it does not correctly apply the federal pleading rule to the facts of the problem and leaves Defendant vulnerable to having admitted a significant allegation of the complaint pertinent to its tort liability. The federal rules make explicit that "in responding to a pleading, a party must ... admit or deny the allegations asserted against it by an opposing party." FRCP 8(b)(1)(B) (emphasis added). Thus, while it is true that a party is not obliged to respond "to a separate claim brought only against other parties to the litigation," the allegation about Defendant's scope of employment indeed targets Company and so a response is required. Milton v. Gen. Dynamics Ordnance & Tactical Sys., Inc., 2011 WL 4708637 (S.D. Ill. 2011) (quotation omitted). Moreover, if Defendant refuses to respond to the allegation of the complaint, the refusal will be deemed an admission. See FRCP 8(b)(6) ("An allegation—other than one relating to the amount of damages—is admitted if a responsive pleading is required and the allegation is not denied."). (D) is not correct because the federal pleading rules permit only three kinds of responses—admit, deny, or state a lack of knowledge or information necessary to admit or deny, seeFRCP 8(b)—and the rule applies even when the allegation of the complaint is expressed as a legal conclusion.

Defendant, a technology company, hired Plaintiff, a technology specialist, as a part-time employee on an at-will basis. The employment contract provided that Defendant would reimburse Plaintiff for all "reasonable travel expenses in connection with work-related assignments." After sending Plaintiff on an important assignment to Hong Kong, Defendant refused to reimburse her travel and hotel expenses, and then fired her without explanation. Two years later, Plaintiff filed a diversity action in federal court alleging breach of contract. Eight months after service of the summons and complaint and six months after service of the answer, Plaintiff was granted leave to amend the complaint to add a claim of tortious interference with contractual relations, alleging that Defendant's actions prevented her from acquiring employment with other companies. Defendant moves to dismiss the amended claim as time-barred under the applicable state statute of limitations. What is Defendant's best argument in support of the motion to dismiss? (A) The state relation-back rule does not permit relation back. (B) The tort claim rests on a legal theory different from that of the original claim. (C) The tort claim arises out of a different pattern of conduct than the original claim. (D) Plaintiff engaged in undue delay in amending the complaint.

(C) is the best argument because it recognizes that an untimely amended claim will "relate back" to the date of the original pleading only if it is factually related to the original claim—so that it does not, in effect, present a new and independent action. The federal rule thus treats an amended claim as if it were filed on the date of the original pleading if it "asserts a claim or defense that arose out of the conduct, transaction, or occurrence set out ... in the original pleading." FRCP 15(c). Determining whether the amended and original claim are transactionally related often involves an inquiry into whether the claims share and are united by a "common core of operative facts." Mayle v. Felix, 545 U.S. 644, 646 (2005). For example, relation back has been permitted when the original complaint alleged that a defendant union breached the duty of fair representation by inadequately representing plaintiff because of gender, and the amended claim alleged a gender discrimination claim under federal law, but was not permitted when the amended claim alleged retaliation. Maegdlin v. Int'l Ass'n of Machinists & Aerospace Works, 309 F.3d 1051, 1052 (8th Cir. 2002). Critical to this inquiry is whether the "opposing party is not unduly surprised or prejudiced." Hill v. Shelander, 924 F.2d 1370, 1377 (7th Cir.1991); accord Schiavone v. Fortune, 477 U.S. 21 ("The linchpin [of Rule 15(c)] is notice ..."). A pertinent question, therefore, is whether the breach-of-contract claim alleged in the original complaint—relating to unpaid reimbursable expenses—put Defendant on notice that it might be tortiously liable for Plaintiff's inability to obtain subsequent employment. Arguably, the amended tort claim depends on "new and distinct conduct, transactions, or occurrences not found in the original complaint," but the question is a close one. McGregor v. Louisiana State Univ. Bd. of Supervisors, 3 F.3d 850, 864 (5th Cir. 1993) (original claim alleging violation of the federal Rehabilitation Act did not put defendant on notice of amended claim alleging due process violation). (A) is not correct because it misstates the relation between the federal rule on amendment and the state relation-back rule. The federal rule explicitly states that relation back will be permitted when "the law that provides the applicable statute of limitations allows relation back." FRCP 15(c)(1)(A). However, the federal rule does not state the converse, and courts consistently hold that "less restrictive state relation-back rules will displace federal relation-back rules, but more restrictive state relation-back rules will not." Morel v. Daimler Chrysler AG, 565 F.3d 20, 26 (1st Cir. 2009). (B) is not correct because it misstates the standard for determining whether an amended claim is sufficiently related to the original claim as to permit relation back. Merely changing the legal theory upon which relief is sought is generally not a bar to relation back under FRCP 15(c). What is critical, by contrast, is the factual overlap of the amended claim and the original claim. As the Eighth Circuit Court of Appeals has explained, "a change in legal theory is not fatal to the rule's application." Maegdlin v. Int'l Ass'n of Machinists & Aerospace Works, 309 F.3d 1051, 1053 (8th Cir. 2002). (D) is not correct because the rule that it states is irrelevant to the determination of relation back under FRCP 15(c). In some circumstances, a court may exercise discretion and deny leave to amend a pleading because the party unduly delayed without good cause in seeking to amend. However, delay is not a factor that affects whether the amended claim relates back to the original claim. See Arthur v. Maersk, Inc., 434 F.3d 196, 203 (3d Cir. 2006) ("There is no allowance in Rule 15(c) for inquiry into a party's delay in moving for leave to amend.").

Plaintiff, a citizen of Missouri, is a widow whose husband was killed in an accident while working for Defendant, a construction company. Defendant is incorporated and has its principal place of business in Idaho. Plaintiff brought a diversity action against Defendant in federal district court in Missouri alleging negligence and seeking $100,000 in damages. Defendant filed a third-party claim against Distributor, seeking $100,000 in damages and alleging that if Defendant is liable to Plaintiff, Distributor is liable to Defendant for having leased the equipment that allegedly caused the accident. Distributor is incorporated in Missouri. Plaintiff seeks to amend her complaint to assert a tort claim against Distributor and to seek $100,000 in damages from that party. Defendant and Distributor each move to dismiss the claims against them based on subject-matter jurisdiction objections. Defendant moves to dismiss Plaintiff's negligence claim; Distributor moves to dismiss Defendant's third-party claim; and Distributor also moves to dismiss Plaintiff's tort claim. How should the court resolve the motions? (A) Grant all motions. (B) Grant Distributor's motion against Defendant and Plaintiff. (C) Grant Distributor's motion against Plaintiff. (D) Grant Defendant's motion.

(C) is the correct answer because Plaintiff's claim against Distributor is not within the court's diversity jurisdiction; supplemental jurisdiction cannot be exercised over it; and the facts do not suggest that any other jurisdictional basis is available. When faced with a multi-party and multi-claim action, it helps to diagram the action: Plaintiff (Missouri) v. Defendant (Idaho) v. Distributor (Missouri) Here, Plaintiff is seeking to amend her complaint to sue Distributor, who has been joined in the action as a third-party defendant, under FRCP 14. However, in order to bring a claim against a third-party defendant, the court must be able to exercise subject-matter jurisdiction. One form of subject-matter jurisdiction is diversity jurisdiction under 28 U.S.C. § 1332(a). The federal diversity statute permits the exercise of diversity jurisdiction if the plaintiff and the defendant are "citizens of different States" and "the matter in controversy exceeds the sum or value of $75,000 exclusive of interests and costs." 28 U.S.C. § 1332(a)(1). Plaintiff (Missouri) and Distributor (Missouri) are citizens of the same state, and so diversity jurisdiction is not present. The second form of subject-matter jurisdiction is supplemental jurisdiction, though this too is unavailable in this situation. Here, Plaintiff and Defendant are citizens of different states and the amount-in-controversy exceeds $75,000, and so diversity jurisdiction is present. When the anchor claim is grounded in diversity jurisdiction, federal law withholds supplemental jurisdiction over any claim "...by plaintiffs against persons made parties under Rule 14, 19, 20, or 24 of the Federal Rules of Civil Procedure..." 28 U.S.C. § 1367(b). Defendant's third-party claim against Distributor requires that Distributor be joined under FRCP Rule 14 which governs third-party practice in federal court. Thus, Distributor is a person made a party under FRCP 14, and Plaintiff is prohibited from joining a claim under § 1367(b); supplemental jurisdiction may not be exercised on Plaintiff's claim against Distributor. The statute codifies the practice that existed before enactment of 28 U.S.C. § 1367, see Owen Equip. & Erection Co. v. Kroger, 437 U.S. 365 (1978), which did not permit ancillary jurisdiction when its exercise would violate the rule of complete diversity as articulated in Strawbridge v. Curtiss, 7 U.S. 267 (1806). (A) is not correct because, of the three claims in the case, the only claim that lacks subject-matter jurisdiction is Plaintiff's claim against Distributor, for the reason described in the explanation to Choice (C). Diversity jurisdiction exists over Plaintiff's claim against Defendant because both parties are citizens of different states and the amount-in-controversy requirement is met. Supplemental jurisdiction exists over Defendant's third-party claim against Distributor because the anchor claim is grounded in diversity jurisdiction; federal law permits the exercise of supplemental jurisdiction over a third-party claim brought by Defendant against a party joined by FRCP 14. The fact that Distributor and Plaintiff are not diverse does not divest the district court of power to hear the anchor claim or the third-party claim. (B) is not correct because Distributor's motion to dismiss Defendant's third-party claim for lack of subject-matter jurisdiction should not be granted. When the anchor claim is grounded in diversity jurisdiction, federal law permits the exercise of supplemental jurisdiction over a third-party claim brought by Defendant against a party joined by FRCP 14. As seen in the explanation to Answer (C), 28 U.S.C. § 1367(b) bars the exercise of supplemental jurisdiction in the absence of complete diversity when the claim against the Rule 14 party is brought by the plaintiff, not by the defendant. Thus, subject-matter jurisdiction exists in the claim by Defendant against Distributor, and it should not be dismissed. Moreover, the lack of diversity between Plaintiff and Distributor does not divest the district court of jurisdiction in the original suit between Plaintiff and Defendant. (D) is not correct because the court may exercise diversity jurisdiction over Plaintiff's claim against Defendant and so the motion to dismiss should be denied. The complaint alleges that the parties are citizens of different states (Plaintiff is a citizen of Missouri and Defendant is a citizen of Idaho) and that the amount in controversy exceeds $75,000; therefore, both conditions of diversity jurisdiction are present. See 28 U.S.C. § 1332(a)(1).

Plaintiff owns a company that manufactures edible decorations used by commercial bakers for holiday cakes. Defendant owns a company that distributes food coloring. Plaintiff enters into a two-year contract with Defendant for the purchase of red and blue food coloring, both of which are essential for cakes baked for the Fourth of July. Defendant is late shipping the food coloring, and as a result Plaintiff loses out on a number of important contracts. Three years later, Plaintiff sues Defendant in federal court for breach of contract. After the requisite jurisdictional allegations, including that the court may exercise diversity jurisdiction, the complaint avers: "Defendant entered into a contract with Plaintiff and Defendant failed to deliver the goods on time." Defendant enters a general denial. Five months after filing its answer, Defendant moves to amend its answer by raising the statute of limitations as an affirmative defense. May the court grant the motion? (A) No, because more than 21 days have passed since Defendant's service of its original answer. (B) No, because the failure to plead an affirmative defense results in waiver of that defense and cannot be cured by amendment. (C) No, because the defense of the statute of limitations can be raised only by denial, and not by affirmative defense. (D) Yes, in the court's discretion and if justice so requires

(D) is the correct answer because it correctly states the standard that governs amendment under the federal rules when a responsive pleading omits an affirmative defense. Under the liberal amendment procedure of the federal rules, a party may amend once as a matter of right within 21 days of serving a pleading, or, if the pleading is one that requires a response, once within 21 days "after service of either a responsive pleading or a motion under Rule 12(b), (e), or (f), whichever is earlier." FRCP 15(a)(1)(A) & (B). Here, however, Defendant cannot file its proposed amendment as a matter of right, and so must seek leave to do so. FRCP 15(a)(2). The motion is directed to the court's discretion, and the rule recites that "[t]he court should freely give leave when justice so requires." Id. In exercising its discretion the district court will be guided by a number of factors. These include a showing of bad faith on the part of the movant, undue delay, the substantive futility of the proposed amendment, or the movant's continued failure to cure deficiencies in the pleading after previously permitted amendments have been filed. These factors do not appear to be present in the problem. This analysis also does not trigger the relation-back doctrine which governs an amendment to add a substantive claim, not the statute of limitations defense. (A) is not correct because it misstates the timing requirement of the federal rule with respect to amendment. A party has an automatic right to amend his or her pleadings if 21 days have not elapsed since service of the paper. SeeFRCP 15(a). In addition, if a responsive pleading is required, a party may amend without leave of court 21 days after service of the responsive pleading or 21 days after service of a motion under FRCP 12(b), 12(e), or 12(f), "whichever is earlier." If no responsive pleading is required—and a reply is not required to an answer unless ordered by the court, see FRCP 7(a)(7) —and if 21 days have passed, then the party seeking to amend must request leave of court to do so. But the federal rule does not impose a blanket ban on amendments simply because 21 days have passed since service of the answer. (B) is not correct because the rule that it states does not control the disposition of the motion to amend in this problem. Generally, a defendant's failure to plead an affirmative defense, such as the statute of limitations, will act as a waiver of that defense. "Ordinarily in civil litigation, a statutory time limitation is forfeited if not raised in a defendant's answer or in an amendment thereto... An affirmative defense, once forfeited, is "exclu[ded] from the case," ... and, as a rule, cannot be asserted on appeal. Wood v. Milyard, 132 S. Ct. 1826, 1832 (2012) (internal citations and quotation marks omitted). However, a party that fails to include an affirmative defense in its answer may amend the pleading as of right if 21 days have not passed since service of the paper, or upon consent of the opposing party, or with leave of the court. See FRCP 15(a). Indeed, courts have permitted amendment to include an omitted affirmative defense as late as at trial. See, e.g., Haskins v. Roseberry, 119 F.2d 803 (9th Cir.), cert. denied, 314 U.S. 655 (1941). Keep in mind that a different rule of waiver applies to defenses under FRCP 12(b)(2)-(5). See FRCP 12(h). (C) is incorrect because it incorrectly states that the expiration of the statute of limitations is not an affirmative defense. Under the federal pleading rules, arguing that the statute of limitations has expired is one of several affirmative defenses that a defendant can raise in an answer to a complaint. See FRCP 8(c). The pleading of an affirmative defense involves the assertion of new facts that, if true, defeat the opposing party's claim. The defense thus is not raised by denial.

Plaintiff owns and operates a retail store in California. It licenses software from Defendant, a Texas company that specializes in security technology. Despite the software, hackers broke into Plaintiff's financial system. Plaintiff sued Defendant in a state court in Delaware for breach of contract, choosing the forum because of its unusually long statute of limitations. Defendant did not object to personal jurisdiction. Plaintiff loses on the merits; the state court holds that Plaintiff was contributorily negligent for its injuries because it did not properly install the software. Plaintiff then filed a second lawsuit against Defendant, largely repeating the breach-of-contract claim that was dismissed in the first lawsuit, but this time filing the lawsuit in federal district court in Delaware. Defendant answers the complaint raising the affirmative defense of claim preclusion and asserts a counterclaim that the filing of multiple lawsuits has caused Defendant to suffer wrongful injury to reputation, and alleges special damages of lost business in excess of $75,000. Other than filing the pair of lawsuits, Plaintiff has no contacts with Delaware. Plaintiff moves to dismiss the counterclaim for lack of personal jurisdiction, arguing that no summons was served with the counterclaim. How should the court rule on the motion? (A) The court should deny the motion, because a compulsory counterclaim does not require an independent basis for jurisdiction. (B) The court should grant the motion, because a permissive counterclaim requires an independent basis of jurisdiction. (C) The court should grant the motion for lack of service of process. (D) The court should deny the motion, because Plaintiff has effectively consented to suit in the forum state.

(D) is the correct answer because the filing of a lawsuit is treated as consent by the plaintiff to personal jurisdiction in the forum "for all purposes for which justice to the defendant requires his presence." Adam v. Saenger, 303 U.S. 59, 67-68 (1938). As the Supreme Court has explained, deeming the plaintiff to have consented to personal jurisdiction in the forum in which he has elected to sue "is the price which the state may exact as the condition of opening its courts to the plaintiff" and is consistent with the requirements of the Due Process Clause. Id.; see alsoSchnabel v. Lui, 302 F.3d 1023, 1037-1038 (9th Cir. 2002) ("[A] party cannot avail itself of the court's jurisdiction, bringing claims which mandate the filing of counterclaims, and thereafter attack personal jurisdiction when judgment is entered against the party on counterclaims."). The Plaintiff in this case availed itself of personal jurisdiction in the federal district court in Delaware by bringing suit there. By so doing, Plaintiff effectively waived any objection to personal jurisdiction he might have asserted had Defendant elected to file his counterclaim as an original claim in an independent lawsuit. (A) is not correct because it relies on an irrelevant legal rule. Answer A deals with subject matter jurisdiction, while the motion challenges the court's personal jurisdiction over the defendant-in-counterclaim. A compulsory counterclaim is a claim made by any party against an opposing party that "arises out of the transaction or occurrence that is the subject matter of the opposing party's claim." FRCP 13(a)(1)(A). Generally, a court has power to exercise supplemental jurisdiction over a counterclaim that arises out of the same transaction or occurrence as the plaintiff's original claim. 28 U.S.C. § 1367. Thus, while it is true that a compulsory counterclaim does not require an independent basis of subject-matter jurisdiction, this rule is irrelevant to whether the court may exercise personal jurisdiction over Plaintiff with respect to Defendant's counterclaim. (B) is not correct because it relies on an irrelevant legal rule. Answer B deals with subject matter jurisdiction while the motion challenges the court's personal jurisdiction over the defendant-in-counterclaim. A permissive counterclaim is defined as a counterclaim that does not arise out of the same transaction or occurrence as the opposing party's claim. See FRCP 13(b). Unlike a compulsory counterclaim, a permissive counterclaim requires an independent basis of subject-matter jurisdiction. 28 U.S.C. § 1367. See, e.g., Oak Park Trust & Sav. Bank v. Therkildsen, 209 F.3d 648, 651 (7th Cir. 2000) (permissive counterclaims "require their own jurisdictional basis"). However, the circuits are now divided on this question, and some courts of appeal have held that supplemental jurisdiction may be exercised over permissive counterclaims that are logically related to the original claim over which the court has subject-matter jurisdiction. Whatever way this disagreement is resolved, it is irrelevant to whether the court can exercise personal jurisdiction over Plaintiff with respect to Defendant's counterclaim. (C) is not correct because it misapplies the governing procedure. A party asserts a counterclaim either by pleading the counterclaim in an answer or by raising it as a reply to a prior-asserted counterclaim. A summons is not required.

Golfco, a golf course management company, adopted a hiring plan designed to increase gender diversity in its workforce. Pursuant to this plan, the plaintiff, a male groundskeeper, was rejected for a promotion in favor of a female applicant. During a Golfco tournament two days later, the plaintiff and several co-workers walked the sidelines chanting loudly "Golfco discriminates!" All were fired the next day. The plaintiff sued in federal court for retaliation under the federal anti-discrimination laws, alleging all of the above facts and stating, "I was terminated solely on account of complaining about sex discrimination and not for any legitimate business reason." Assume that it is common knowledge that golf tournaments require spectators to keep quiet avoid distracting golfers. Golfco files a motion to dismiss the plaintiff's complaint for failure to state a claim under Rule 12(b)(6). How should the court rule on Golfco's motion? (A) The court should grant the motion, because the plaintiff's allegations, taken as true, along with common knowledge, suggest an obvious alternative legal explanation for his termination. (B) The court should grant the motion, because the complaint's allegations are entirely conclusory. (C) The court should deny the motion, because discrimination is the most likely explanation for the plaintiff's termination. (D) The court should deny the motion, because every litigant who files a complaint is entitled to get discovery.

A is correct because it accurately states the current pleading standard under Rule 8(a)(2) as interpreted by Bell Atlantic v. Twombly and Ashcroft v. Iqbal. This answer also accurately applies that standard to the complaint's allegations. Even taking the specific factual allegations as true, the complaint alleges that the plaintiff engaged in behavior that, according to common knowledge, is unacceptably disruptive. Because Iqbal permits "judicial experience and common sense" to inform the Rule 8(a)(2) analysis, in light of this common knowledge, the complaint does not sufficiently allege that the plaintiff was fired "solely" for reporting discrimination. The "obvious alternative legal explanation" is that he was terminated for being unacceptably disruptive—particularly since the complaint says that all of the participating co-workers were fired along with him. B is incorrect because the complaint's allegations are fact-specific about many of these events giving rise to the lawsuit; they are not "entirely" conclusory. C is incorrect because it states the wrong legal standard. The question is not whether the complaint makes discrimination the "most likely" explanation; rather, the question is whether the well-pled allegations, taken as true, support a reasonable inference of illegal conduct. D is incorrect. It contradicts Rule 8(a)(2) as well as the authoritative decisions interpreting that rule on 12(b)(6) motions.

Toyco, a toy manufacturing company, sued one of its former employees in federal court under federal and state laws for designing and marketing a competing toy while she was still employed at Toyco. In discovery, Toyco asked the former employee to produce hard drives from any personal laptop computer she used for design purposes while working at Toyco. The former employee responded that no such hard drives existed. Later, in her deposition, the former employee testified that she had a personal laptop while she worked at Toyco but used it only for gaming, never for designing. She testified that she had cleaned the hard drive and disposed of the laptop one year after she stopped working for Toyco because she was seeking treatment for a gaming addiction—even though by then Toyco's lawyer had threatened to sue her. Other discovery evidence corroborated that the former employee's competing designs were composed on a work-issued computer. What sanctions, if any, should the court impose for the former employee's failure to preserve her personal laptop's hard drive? (A) The court should not impose sanctions, because the loss of the laptop and its hard drive did not prejudice Toyco (B) The court should give the jury an adverse inference instruction against the former employee (C) The court should enter a default judgment against the former employee (D) The court should impose sanctions, because the former employee anticipated litigation at the time she disposed of the laptop

A is correct. Because a laptop hard drive contains electronically stored information, Rule 37(e) applies. Under Rule 37(e)(1), sanctions for non-intentional deprivation of evidence may be imposed only on a finding of prejudice to the opposing party. Nothing in the question suggests that the former employee disposed of the laptop to intentionally deprive Toyco of evidence. As to prejudice, the loss of a hard drive that would reveal gaming habits but not competing designs will not prejudice Toyco's unfair competition claim. First, a laptop used only for gaming and not designing is not even responsive to Toyco's discovery request for "any personal laptop computer used for design purposes." Second, in any event, Toyco has evidence of the employee's competing designs from another source. B is incorrect. The adverse inference sanction applies only to "acting with intent to deprive another party of the information's use in litigation." Rule 37(e)(2). As explained above, the facts don't support this intent. C is incorrect for the same reason. Under Rule 37(e)(2)(C), default is not an appropriate sanction for non-intentional destruction of electronically stored information. D is incorrect. Even though the threat of a lawsuit by Toyco's lawyer would suggest litigation is coming, it is much less clear that a laptop used for gaming "should have been preserved in the anticipation or conduct of" this litigation—a threatened dispute over the employee's competing designs. Moreover, anticipating litigation is not the only requirement that the rule imposes for sanctions; it also requires prejudice. See Rule 37(e)(1).

Plaintiff filed suit in federal district court after the Defendant refused to perform under a signed contract. The contract was for the sale of real estate worth $75,000. In fact, not only did the Defendant refuse but the day after signing the contract with Plaintiff, the Defendant sold the land to a bona fide purchaser. Plaintiff is a citizen of State M and Defendant is a citizen of State C. After suit was filed, Defendant filed a motion to dismiss based on lack of subject matter jurisdiction. How should the court rule? (A) The district court should grant the motion, because the court lacks subject matter over the controversy. (B) The district court should grant the motion, because the federal court does not have jurisdiction over state law contract claims. (C) The district court should deny the motion, because the court has jurisdiction over the claim based on diversity jurisdiction. (D) The district court should deny the motion, because the parties stipulated in the signed contract that the federal district court will resolve all disputes arising from performance or non-performance of the contract.

A is correct. Federal courts have subject matter jurisdiction over violations of federal law, constitutional rights violations, and state law claims under diversity jurisdiction. Diversity jurisdiction requires that there is complete diversity of citizenship between plaintiff and defendant and the claim is reasonably likely to exceed $75,000. Here, the real estate was worth exactly $75,000 and there is no indication that the Plaintiff has damages above the worth of the land. Since the claim is for exactly $75,000, it is not reasonably likely to exceed $75,000; therefore, diversity jurisdiction is not satisfied. B is incorrect because federal courts may exercise subject matter jurisdiction over state law claims if the litigants can satisfy the requirements for diversity jurisdiction, which they have not done so here. C is incorrect for the reasons stated in A. D is incorrect because subject matter jurisdiction cannot be conferred by consent or waiver. The court must have proper subject matter jurisdiction in order to render a valid judgment. Since the court here did not have subject matter jurisdiction, the only proper ruling is to grant the motion to dismiss.

Plaintiff, a citizen of State O, filed a lawsuit in O State court asserting a $250,000 negligence claim against a citizen of State I and a $20,000 breach of contract claim against a citizen of State O. Three days after filing the complaint, Plaintiff voluntarily dismissed the claim against the State O defendant. The next day, the defendant from State I removed the case. Plaintiff filed a motion to remand to state court. How should the court rule? (A) The court should deny the motion, because it now falls within the court's original jurisdiction. (B) The court should grant the motion, because there is not complete diversity of citizenship. (C) The court should deny the motion, because the plaintiff is a citizen of the forum state. (D) The court should grant the motion, because the breach of contract claim is for $10,000.

A is correct. Originally, this case was not removable because it was a state law claim lacking in complete diversity. However, the case became removable when Plaintiff voluntarily dismissed the claim against the non-diverse defendant, the State O defendant. By so doing, there was complete diversity between Plaintiff and the remaining defendant, the State I defendant. Where the plaintiff effectuates the change in the status quo, the case is removable. Section 1446B directly addresses this scenario by providing that where the case stated by the original complaint is not removable, but a change is made thereafter that renders the case removable, the defendant is not precluded by the generally applicable requirement in §1446A that the notice of removal be filed within 30 days after receipt of the initial complaint but may remove the case within 30 days after the case becomes removable. This is not without its limits: In diversity cases that were not removable on the basis of the initial pleading, §1446B provides for a one-year statute of limitations on removal dating from the commencement of the suit. B is incorrect. After the plaintiff voluntarily dismissed the claim against the non-diverse defendant, the remaining claim becomes removable and, between the plaintiff and the remaining defendant, there is complete diversity of citizenship, making this answer choice incorrect. C is incorrect because this fact is irrelevant. In removal jurisdiction, the defendant has the right to remove a case from state court to federal court if the case could have been filed in federal court. If the basis for removal is predicated on diversity, the defendant cannot be from the forum where the action is pending. D is irrelevant because the defendant as to that claim has been dismissed and is now irrelevant to the question of removal.

Following an auto accident, Plaintiff-driver and Passenger of a car bring a negligence action against Defendant driver in state court in State T. Plaintiff-driver is a citizen of State T and Passenger is a citizen of State L. Defendant-driver is a citizen of State T. Each of the plaintiffs seeks $100,000 in damages against Defendant. Passenger also files a $100,000 claim against Plaintiff-driver for damages resulting from the injury. Can Plaintiff-driver properly remove the case? (A) No, because plaintiffs cannot remove cases. (B) Yes, because the cross-claim meets all the requirements of diversity jurisdiction under §1332. (C) No, because Plaintiff-driver is a citizen of the forum state. (D) Yes, because Passenger is not a citizen of the forum state.

A is correct. Section 1441A provides for removal "by the defendant or the defendants." Here, the removing party is a plaintiff who is the defendant to a cross-claim by a co-plaintiff. A plaintiff can never remove a case, even if the plaintiff is a "defendant" in a cross-claim (as here) or counterclaim. Thus, the case by Passenger against Plaintiff-driver (defendant in cross-claim) cannot be removed on the basis of a state law cross-claim even if it meets all requirements set forth in §1332 for original jurisdiction based on diversity of citizenship. B is incorrect. A plaintiff is not deemed a "defendant" for removal purposes even when it is the counter-defendant to a claim within the federal court's original jurisdiction. The courts apply this same doctrine to bar removal on the basis of a diverse cross-claim asserted by one plaintiff against a co-plaintiff. C is incorrect because since the case cannot be removed on the basis of a diverse cross-claim against a plaintiff, the fact that the responding party to that diverse claim is a citizen of the forum state is irrelevant. D is incorrect because this fact is irrelevant to this problem.

BigTech, Inc. sued one of its former software developers in federal district court for copyright infringement and for violating a noncompete agreement. The court took federal question jurisdiction over BigTech's federal copyright claim and took supplemental jurisdiction over BigTech's noncompete claim, which is governed by state statutory law. Eight months after the lawsuit was filed, the legislature in the state where the case arose and was filed passed a law making noncompete agreements illegal. The law was retroactive, applying to noncompetes signed even before the law's passage. After the law was passed, BigTech argued the noncompete claim's merits in its written response to the former employee's summary judgment motion. Assume the law's retroactivity does not pose any constitutional due process problems. Which of the following is true about Rule 11's application to BigTech's summary judgment response? (A) BigTech violated Rule 11(b)(2)'s requirement that legal contentions be warranted by existing law or by a non-frivolous argument for changing the law. (B) BigTech violated Rule 11(b)(3)'s requirement that factual contentions have evidentiary support. (C) BigTech did not violate Rule 11(b)(2) or 11(b)(3), because its noncompete claim has sufficient factual and legal support. (D) BigTech did not violate Rule 11(b)(2) or 11(b)(3), because those provisions do not apply to a party's responses to motions.

A is correct. The state legislature's retroactive law invalidated BigTech's noncompete agreement, so the claim is no longer warranted by existing law. There is no wiggle room for a non-frivolous argument against the law, as there might be with precedent that is non-binding or that leaves room for interpretation. See Rule 11(b)(2). Because Rule 11 applies to "written motion[s] or other paper[s]" and to "later advocating" matters presented in the pleadings, it applies to BigTech's continued pursuit of the noncompete claim in its summary judgment response. See Rule 11(b). B is incorrect because nothing in the fact pattern suggests that BigTech lacks the evidence needed to sustain a noncompete claim; the problem is legal, not factual. C is incorrect because the noncompete claim lacks legal support as explained above. Dis incorrect because Rule 11(b) explicitly applies to "other papers," not just pleadings or motions.

Plaintiff, a citizen of State N, brought a patent infringement claim against Avant Co., a Delaware corporation, in federal court in State N. Plaintiff alleged that Avant had infringed her patent in violation of the federal patent statute and sought damages in the amount of $50,000. Plaintiff also asserted a $150,000 intentional infliction of emotional distress claim against the president of Avant, James Avant, a citizen of State N. In this claim, Plaintiff alleged that James Avant, acting on behalf of Avant Co., engaged in the infringing conduct and that this caused Plaintiff to suffer severe emotional distress. The defendant James Avant files a motion to dismiss for lack of subject matter jurisdiction over the emotional distress claim. How should the court rule? (A) The court should deny the motion, as the federal court has jurisdiction over the supplemental claim. (B) The court should grant the motion, because courts do not permit pendent party jurisdiction. (C) The court should deny the motion, because the value of the two claims can be aggregated. (D) The court should grant the motion, because Plaintiff sought less than $75,000 in the patent infringement claim.

A is correct. Under §1367A, if the plaintiff asserts a federal question claim against one defendant, the court can exercise supplemental jurisdiction over a non-diverse state law claim against a separate defendant if the two claims form part of the same case—meaning, the two claims arise out of a common nucleus of operative facts—and if none of the factors listed in §1367C justify declining to exercise such supplemental jurisdiction. Here, the patent claim against the corporation is a federal question claim. The tort claim against the president does form part of the same case as that federal question claim since the infringing conduct gave rise to both claims. And none of the discretionary factors listed in §1367C justifies declining to exercise supplemental jurisdiction. Thus, jurisdiction can be exercised under §1367A. B is incorrect because §1367A does permit pendent party jurisdiction in cases involving a federal question claim, even when the supplemental state claim is against a non-diverse defendant. Pendent party jurisdiction refers to the court's power to adjudicate a claim against a party who would not otherwise be subject to the court's jurisdiction but the claim against the party arises from the same set of operative facts as the original claim. Here, the federal court would not have diversity jurisdiction over the emotional distress claim against James Avant, as both he and Plaintiff are from State N. However, since the emotional distress claim arises from the same set of operative facts as the original claim under federal jurisdiction, the federal court has supplemental jurisdiction over the emotional distress claim against Avant. C is incorrect because it is irrelevant and wrong as a matter of law. First of all, there is no aggregation of claims by a single plaintiff against multiple defendants. More important, aggregation of damages against a single defendant is a concept found in diversity jurisdiction; here, the plaintiff and defendant James Avant are not diverse and so §1332—and therefore its amount in controversy requirement—is inapplicable to this question. D is incorrect because §1331, which confers federal court jurisdiction over claims arising under federal law has no amount in controversy requirement.

A Texas citizen was given a two-year contract by a Middle Eastern oil company to work in one of its plants in that country. After six months, the plaintiff was fired. She returned to the United States and filed a claim in federal court in Dallas, Texas, against the company alleging that she was terminated on the basis of her sex in violation of Title VII of the federal Civil Rights Act of 1964. The defendant moved to dismiss the action on the ground that although it admitted that it was subject to personal jurisdiction in Texas because of its past business dealings there (including recruiting the plaintiff from Texas), maintaining the suit there would result in extreme inconvenience to it since all the alleged acts of discrimination occurred in the Middle East, all the witnesses to the events in question were there, and defending in the United States would be extremely inconvenient. The plaintiff opposed the defense motion on the ground that the foreign court would apply foreign law to this dispute and that under that law, she would have to establish malicious conduct beyond a reasonable doubt, whereas under Title VII, she would only have to prove negligent conduct by a preponderance of the evidence. Should the court dismiss the case? (A) Yes, under the doctrine of forum non conveniens (B) No, because the foreign law is so hostile to the plaintiff's claim. (C) Yes, because the defendant is a foreign company. (D) No, because the plaintiff is an American citizen.

A is the correct answer. In Piper Aircraft v. Reyno, an airplane crashed off the coast of Scotland. The plane was manufactured in Pennsylvania while the propellers were manufactured in Ohio. A suit was filed in California on behalf of the deceased. The California case was removed to federal district court and the case was transferred to the federal district court in Pennsylvania. Upon motion, the Pennsylvania court dismissed the action under the doctrine of forum non conveniens. While the Third Circuit reversed, the Supreme Court dismissed under the doctrine of forum non conveniens. The defendant here is relying on the doctrine of forum non conveniens as the basis for dismissing an action over which the court clearly has subject matter jurisdiction, in which venue lies, and with respect to a defendant over which it clearly has personal jurisdiction. Courts will grant this motion upon a showing not only that the chosen forum is extremely inconvenient but that a distinctly more convenient forum exists in which the court would have subject matter jurisdiction, venue, and personal jurisdiction. Assuming the court finds the chosen forum to be manifestly and significantly inconvenient, it will grant the motion in light of the availability of this alternative, more convenient forum. Answer B is incorrect. In Piper Aircraft v. Reyno, the Supreme Court indicated that, in determining whether an alternative forum exists, if the remedy provided by the alternative forum is so clearly inadequate or unsatisfactory that it is no remedy at all, the unfavorable change in law may be given substantial weight and the district court may conclude that dismissal would not be in the interests of justice. But the Court also stated that this standard is not met merely when the remedy provided by the alternate forum is either more difficult to obtain or not as lavish as that provided by the law that would be applied by the initially chosen forum. The facts in this problem would not meet this "inadequate remedy" standard. Answer C is incorrect because this fact, standing alone, is not a sufficient basis for invoking the doctrine of forum non conveniens. Answer D is also incorrect because this is not a sufficient basis for refusing to invoke the doctrine of forum non conveniens.

A California plaintiff brought a $250,000 breach of contract action against an Arizona citizen in federal court in Phoenix, Arizona. The complaint alleges that the defendant agreed to purchase $250,000 of merchandise from the defendant's store, shipped the merchandise to his home in Phoenix, and then refused to pay the invoice. The defendant moved to have the case transferred to the federal district court in Santa Fe, New Mexico. In his motion to transfer, the defendant declared that although he had no contacts with the state of New Mexico, he would waive any objections to personal jurisdiction and venue in that state. Should the court grant the motion to transfer? (A) Yes, because the defendant has waived his objections to personal jurisdiction and venue. (B) No, because the plaintiff would be subject to personal jurisdiction in New Mexico. (C) No, because New Mexico is not a district where this action might have been brought by the plaintiff. (D) Yes, because the plaintiff has no objection to the transfer.

A is the correct answer. In this problem, the defendant seeks to transfer the case to a district that would not have personal jurisdiction over him. Pursuant to the recent amendment to the venue statute, a case can be transferred to a district where the action could not have been brought initially, since the defendant is willing to consent to the exercise of jurisdiction for the purposes of effecting a transfer. Since the defendant is willing to consent to personal jurisdiction for purposes of transfer, the presumption is that it would have been willing to consent to the exercise of jurisdiction by that court if the plaintiff had initially chosen that forum. See 28 U.S.C. §1404. Answer C is incorrect because the venue statute (28 U.S.C. §1404) was amended in order to address the controversial ruling in Hoffman v. Blaski where the Supreme Court construed the language to mean that the transferor court must possess personal jurisdiction over the defendant and any lack of jurisdiction cannot be cured by the defendant's consent to the exercise of jurisdiction over it by the transferor court. Answer B is incorrect because this fact, if true, is irrelevant to this problem. Answer D is incorrect because this fact is irrelevant to this problem.

Jill went on a vacation to Paris, France, with her best friend, Sandy. Sandy is a resident of Chicago, Illinois. Jill had been a lifelong resident of Chicago but moved to Philadelphia, Pennsylvania, just before leaving for vacation. While they were in Paris, Jill and Sandy rented motorcycles and during their ride, Sandy's motorcycle ran into Jill's motorcycle while Jill was on it. Jill returned home and filed a civil action in federal court in Chicago asserting $200,000 tort claims against Sandy and Moto, the motorcycle rental company in Paris. Does venue lie in the chosen forum? (A) Yes, because Sandy resides there and Moto is a French company. (B) No, because Moto is a French company. (C) Yes, because Jill used to live in Chicago. (D) No, because the accident occurred in Paris.

A is the correct answer. Subject matter jurisdiction is predicated on §1332(a)(1) (as to the claim against Sandy) and §1332(a)(2) (as to the claim against Moto). So the governing venue statute is §1391(b)(1), which provides for venue in the district where the defendant resides. Since Sandy resides in Chicago, venue is clearly appropriate as to the claim against her. Moto is a foreign company. But §1391(c)(3) expressly allows that a defendant not residing in the United States may be sued in any judicial district, and that joinder of such defendant shall be disregarded in determining where the action may be brought with respect to other defendants. Answer B is incorrect because aliens can be sued in any federal district. Answer C is incorrect because the residence of the plaintiff is irrelevant for venue purposes under §1391(c)(3). Answer D is incorrect because the fact that the accident did not occur in the chosen venue does not preclude venue if venue can be predicated on the residence of the defendant(s) under §1391(b)(1).

Dan, a lifelong resident of New York, went to New Orleans, Louisiana, for a convention. One night, he tasted a local Cajun whiskey. Liking what he tasted, he bought a case of the whiskey to take back to New York. Upon his return to New York, Dan gave a bottle to his boss, Ben, a citizen of New York. After work that evening, Ben decided to try the whiskey and prepared himself a cocktail consisting of the Cajun whiskey and water. After three or four sips of his cocktail, Ben experienced a severe burning sensation in his throat and stomach. He called his doctor, who advised him to come to the hospital and bring the bottle of the whiskey with him. At the hospital it was determined that the bottle contained a high percentage of acid. Ben was treated accordingly. He survived, but had to have part of his stomach removed and will talk in a low raspy voice for the rest of his life. Ben comes to you, an attorney in New York, and wants you to represent him in his personal injury action. He wants to sue for $1 million to pay for his medical expenses and be compensated for his pain and suffering and permanent physical impairments. You agree to represent him and immediately begin making certain investigations. You learn that the Cajun whiskey is a product distilled by the De-Lis Whiskey Company, a Louisiana corporation with its principal place of business in Louisiana. It distributes its products in Louisiana, Alabama, and Mississippi. You learn that De-Lis places ads in NOLA magazine at all local hotels, including the one where Dan stayed during his visit. You learn that about 45% of all sales of the Cajun whiskey are made to New York tourists who take the product back to their home state and 50% of its sales are made to New Yorkers who purchase the liquor through De-Lis's highly interactive web site. Sales to New Yorkers account for in excess of $3 million annually. In addition, you learn that De-Lis Whiskey Company has $500,000 on deposit in a New York bank. Assume that the legislature in every state in the country has passed the following statute: The courts of this state shall have personal jurisdiction over an individual, corporation or other entity who, in person or through an agent: transacts business within the state; or commits a tortious act without the state causing injury within the state; or is personally served within the state; or owns property within the state. Can a state court in New York exercise specific jurisdiction over De-Lis Whiskey Company? (A) Yes, because De-Lis transacts business in New York and the cause of action arose out of those contacts. (B) Yes, because of the presence of De-Lis's bank account in New York (C) No, because it did not commit a tortious act in New York. (D) No, because De-Lis has no contacts with New York.

A is the correct answer. The proper approach to determining whether or not a state court can exercise personal jurisdiction is a two-step analysis. First, one must determine whether the applicable state long-arm statute applies. Here, De-Lis is doing more than $3 million in annual business with New York customers, both those that come to New Orleans and bring the whiskey to New York and those who buy it over the Internet. So the statute applies. One must then determine the constitutionality of exercising jurisdiction. That is a multi-step process. First, is the court attempting to exercise specific or general jurisdiction? The question asks only for specific jurisdiction, which means that the cause of action arises out of the defendant's relationships with the forum state. Here, the cause of action is for damages caused by the defective product. While it could be argued that the defendant's connection with the forum is only the transport and not the manufacture of the product, if one views the connection as the entire business of producing and distributing liquor, then the cause of action does arise out of the defendant's relationships with the forum. In that case, one must then evaluate the extent of the defendant's relationship. Since this appears to be a continuous and systematic relationship, the International Shoe standard is met. One must then also evaluate the "fairness" factors set forth in Volkswagen. Here, the plaintiff is a forum citizen and so both he and the forum state have an interest in having the case heard in the chosen forum. And the interstate judicial system has an interest in having the case heard in New York since many of the witnesses including the plaintiff and his doctors are in the forum. Answer B is incorrect because the cause of action would not arise out of those contacts (the bank account) and so this could not be the exercise of specific jurisdiction. Answer C is incorrect because the long-arm statute does not require the tortious act to have been committed in New York. It permits jurisdiction where the defendant commits a tortious act outside the forum but causes injury inside the forum, which happened here. Answer D is incorrect because De-Lis clearly has extensive contacts with New York.

Plaintiff from State Z was a student at University located in State A. After Plaintiff attended University for two years, University suspended Plaintiff for allegedly cheating on a final examination. After a full hearing on the matter, University expelled Plaintiff. Plaintiff filed a 2-count complaint against University in State A state court—Count I was based on gender discrimination in violation of federal law, and Count II was based on breach of contract, stemming from University's statement found in the student handbook regarding nondiscrimination. Plaintiff claimed $55,000 in damages. After service of process was properly effectuated, University removed the case to federal court in State A. Following the completion of all discovery, University filed a motion to dismiss for failure to state a claim upon which relief can be granted on both counts of the complaint. The federal district court granted University's motion to dismiss on the federal gender discrimination claim, but denied it on the breach of contract claim. Plaintiff then filed a motion to remand the matter to state court, which the court granted. After trial in state court, University appealed the federal trial court's earlier decision to remand the case with the federal circuit court of appeals. How should the federal circuit court respond to the appeal? (A) The circuit court may hear the appeal because the district court did not lack subject matter jurisdiction over the state law breach of contract claim. (B) The circuit court should reverse the district court's decision because the district court erred as a matter of law in granting Plaintiff's motion to remand. (C) The circuit court should deny the appeal because the circuit court lacks jurisdiction to hear the appeal. (D) The circuit court should deny the appeal because the district court has the sole and final authority to accept or decline to exercise subject matter jurisdiction over all cases on its docket.

A is the correct answer. Under 28 U.S.C. §1447(d), 'an order remanding a case to the State court from which it was removed is not reviewable on appeal[ . . . ] .' In Thermtron Products, Inc. v. Hermansdorfer, 423 U.S. 336 (1976), the Supreme Court held that under §1447(d), a federal court's order remanding a case to state court is barred from appellate review only if the remand is due to reasons listed in §1447, such as a remand due to lack of subject matter jurisdiction noted in §1447(c). In Carlsbad Technology, Inc. v. HIF Bio, Inc., 556 U.S. 635 (2009), the Supreme Court held that a district court's decision not to exercise supplemental jurisdiction over a state law claim is not a dismissal for lack of subject matter jurisdiction for which appellate review is barred. The unanimous opinion stated that the district court merely exercised its discretion to dismiss a state law claim for which it had supplemental jurisdiction, and therefore its decision was not based on lack of subject matter jurisdiction, which would have precluded federal appellate court review. Therefore, A is correct and Answer C is incorrect. Answer B is not the best choice because it does not address the main issue under these facts, which is whether the court of appeals has jurisdiction to hear this appeal. Further, the district court's decision to remand is discretionary and would be reviewed by the circuit court with deference under the abuse of discretion standard. The correctness of the decision to remand is not a matter of law and would not be subject to de novo review. Answer D is incorrect because it incorrectly states the law regarding the district court's authority with regard to subject matter jurisdiction. Generally speaking, district courts do not have discretion to decline to hear cases over which they have original subject matter jurisdiction. In certain circumstances federal courts have discretion in determining whether to remand state law claims over which they have exercised supplemental jurisdiction, their power is stated too broadly in this answer choice.

The defendant/purchaser, a Vermont citizen, bought a television from the plaintiff/retailer, an Illinois corporation. The written sales agreement provides that the purchaser designated the company president's secretary to receive service of process for the purchaser in any action brought by the retailer under the terms of this agreement. When the purchaser failed to make a payment, the retailer sued him for breach of contract in federal district court and served the company president's secretary. The secretary, who was unknown to the defendant, mailed a copy of the service to the defendant's home and the defendant received it. Is this service proper under the Federal Rules of Civil Procedure? (A) Yes, because the defendant agreed to it by signing the contract and received the process. (B) No, because this is a form contract and the service provision was in small type. (C) No, because the secretary was unknown to the defendant and was the plaintiff's employee. (D) Yes, because the secretary was paid for being a recipient of service.

A is the correct answer. Under Fed. R. Civ. P. 4(e)(2)(C), service can be made upon an agent authorized by appointment to receive service of process. The Supreme Court held that a service provision in a form contract can constitute an effective appointment of an agent for service of process even if the recipient is unknown to the defendant. The only requirement of the job is to transfer service to the defendant and if that job is fulfilled, the Court ruled, the service is effective. That occurred here and so the service is proper under Rule 4(e)(2)(C). Answers B and C are incorrect because the defects alluded in the answers are irrelevant under these circumstances. Answer D is incorrect because being paid for being the recipient is irrelevant since a service provision in a form contract can constitute an effective appointment of an agent for service of process.

Jill went on a vacation to Europe with her two best friends, Lenny and Patty. Lenny is a resident of New York City, New York, and Patty is a resident of Chicago, Illinois. Jill had been a lifelong resident of Chicago but moved to Philadelphia, Pennsylvania, just before leaving for vacation. While they were in Paris, Lenny and Patty rented motorcycles and during their ride, they ran over Jill. Jill returned home and filed a civil action in federal court in Philadelphia asserting $200,000 tort claims against both Lenny and Patty. Neither Lenny nor Patty has ever been to Pennsylvania nor have either of them had any contact with the state. Does venue lie in the chosen forum? (A) No, because neither defendant resides there and the accident occurred in Paris. (B) Yes, because there is no other district in which venue would lie. (C) No, because New York law would govern the merits of the case. (D) Yes, because Jill is a resident of Philadelphia.

A is the correct answer. Venue in diversity cases is governed by 28 U.S.C. §1391. Venue is proper under this statute in a district in which any defendant resides if they all reside in the same state, a district in which a substantial part of the events giving rise to the claim occurred, or a district in which any defendant is subject to personal jurisdiction if there is no other district in the United States that would qualify under either of the first two tests. Neither of the defendants (NY, IL) resides in the chosen district (PA) and the events giving rise to the claim did not occur within that district (Paris, France). Can we then rely on the personal jurisdiction-based fall-back position? Only if there is no other federal district in which venue would lie under either of the first two tests. Since there is no district in the United States in which both defendants reside and there is no district in the United States where the events giving rise to the claim occurred, then the fall-back provision comes into play. However, Pennsylvania is not a suitable venue under the fall-back provision for venue as neither have been to Pennsylvania and neither have sufficient contacts in that forum in order to be subject to personal jurisdiction in PA. Answer B is incorrect. The fall-back provision states that venue will lie in a district in which "any defendant" is subject to personal jurisdiction. But neither defendant is subject to personal jurisdiction in Pennsylvania so venue would not lie in any district in that state. Since Lenny is subject to personal jurisdiction in New York and Patty is subject to personal jurisdiction in Illinois, venue would lie in either the Southern District of New York or the Northern District of Illinois. Answer C is incorrect because this factor is irrelevant to a venue determination. Venue in federal courts is governed by 28 U.S.C. §1391, not state law. Answer D is incorrect because the plaintiff's residence is irrelevant to a venue determination.

Erica, a resident of New York City, was vacationing in California when the taxi she was riding in collided with an automobile owned by Dave, a resident of San Francisco, California, and driven by Chris, a resident of Los Angeles, California. Erica brought a tort action seeking $300,000 in damages against both Dave and Chris in the federal district court for the Central District of California, which is located in Los Angeles. San Francisco is located within the Northern District of California. Does venue lie in the chosen forum? (A) Yes, because the plaintiff chose this forum. (B) No, because the plaintiff is a resident of New York City. (C) Yes, because Chris resides there. (D) No, because Dave resides in San Francisco, which is located in the Northern District of California.

Answer C is correct. Venue in diversity cases is governed by 28 U.S.C. §1391(a). Under §1391(b)(1) venue will lie in a federal judicial district in which any defendant resides as long as all defendants reside in the same state. Although the two defendants reside in different federal judicial districts (the Northern and Central Districts of California), they both reside in the State of California. Therefore, under § 1391(b)(1), venue would lie in either the Central (Los Angeles) or Northern (San Francisco) District. Answer A is incorrect because the plaintiff's choice of venue is not dispositive. Answer B is incorrect because residence of the plaintiff is irrelevant for venue purposes. Answer D is incorrect because, under § 1391(b)(1), only one of the defendants needs to reside in the chosen district if both defendants reside in the same multi-district state.


Ensembles d'études connexes

Qualified Retirement Plans Chapter Quiz

View Set

Business Law - Commercial - Chapter 28, 29, 30, 31, 32, 33, 34

View Set

(NR 328 WK 5) Chapter 31: The Child with Endocrine Dysfunction with PP slides

View Set

inquizitive 12 - monopolistic competition and advertising

View Set

Mult Chapter Handbook of Informatics Study Guide

View Set

Original Preamble-In Your Own Words

View Set